anaesthesia tmj pain orthognathic pathology reconstruction

220
2007 Oral and Maxillofacial Surgery Self Assessment Tool (OMSSAT) Administration Dates: April 14 - 28, 2007 The American Board of Oral and Maxillofacial Surgery

Upload: mohammed-qasim-al-watary

Post on 03-Dec-2015

11 views

Category:

Documents


5 download

DESCRIPTION

Great MCQ

TRANSCRIPT

Page 1: Anaesthesia TMJ Pain Orthognathic Pathology Reconstruction

2007 Oral and Maxillofacial Surgery

Self Assessment Tool (OMSSAT)

Administration Dates: April 14 - 28, 2007

The American Board of Oral and Maxillofacial Surgery

Page 2: Anaesthesia TMJ Pain Orthognathic Pathology Reconstruction

2006 - 2007 Oral and Maxillofacial Surgery Self Assessment Tool (OMSSAT) Committee

Chair - Patrick J. Louis

Anesthesia Section Editor – Jeffery D. Bennett Vincent Williams Patrick J. Vezeau Trevor Treasure David Todd Tony Petito Vasiliki Karlis Joseph Frielich Vince DiFabio Kevin Butterfield TMD/Pain Section Editor – Samuel McKenna Leon Assael John H. Campbell Louis Mercuri Stephen Milam H. Clifton Simmons III Alireza Sodeifi John Zuniga L. George Upton Michael Locke

Pathology/Reconstruction Section Editor – Eric Carlson Nabil Abaza Tara Aghaloo Jon Holmes Brian Schmidt Brent Ward Dale Baur Rui Fernandes Orthognathic/Cleft/Craniofacial Section Editor – Ramon Ruiz Sean Edwards Bernard J. Costello John Caccamese Paul S. Tiwana Patrick Ricalde Tinerfe J. Tejera Paul S. Tiwana Gregory Casey Jonathan Bailey

Page 3: Anaesthesia TMJ Pain Orthognathic Pathology Reconstruction

2007 Oral and Maxillofacial Surgery Self Assessment Tool (OMSSAT)

The American Board of Oral and Maxillofacial Surgery 3

1. What is the recommended maximum dose of 4% articaine 1:200,000 epinephrine for a 70 kg

adult?

A. 280 mgs

B. 350mgs

C. 420 mgs

D. 490 mgs

Answer: D Rationale:

Manufacture's recommended maximum dose is 7.0 mg/kg or 3.2 mg/lb. For children between the ages of 4 and 12 years, the manufacturer recommends a dose 5 mgs/kg or 2.27 mgs/lb.

Reference: Malamed SF: Handbook of Local Anethesia, 4th Edition. St. Louis, Mosby, 1997 p. 63-64

Page 4: Anaesthesia TMJ Pain Orthognathic Pathology Reconstruction

2007 Oral and Maxillofacial Surgery Self Assessment Tool (OMSSAT)

The American Board of Oral and Maxillofacial Surgery 4

2. Which of the following local anesthetics has the slowest onset time?

A. Lidocaine

B. Prilocaine

C. Bupivicaine

D. Mepivicaine

Answer: B Rationale:

The pKa of a local anesthetic determines its onset time. The closer the pKa of the anesthetic is the pH of tissue (7.4), the more rapid the onset time. The pKa of a local anesthetic is the pH at which equal concentrations of ionized and unionized forms exist.

Reference:

Malamed SF: Handbook of Local Anethesia, 4th Edition. St. Louis, Mosby, 1997 p. 49-73

Page 5: Anaesthesia TMJ Pain Orthognathic Pathology Reconstruction

2007 Oral and Maxillofacial Surgery Self Assessment Tool (OMSSAT)

The American Board of Oral and Maxillofacial Surgery 5

3. The lipid solubility of a local anesthetic determines its:

A. duration of anesthesia. B. onset time. C. potency. D. toxicity. Answer: C Rationale:

The lipid solubility of a local anesthetic appears to be related to its intrinsic potency. Increased lipid solubility permits the anesthetic to penetrate the nerve membrane (which is 90% lipid) more easily. This is reflected biologically in an increased potency of the anesthetic. Local anesthetics with greater lipid solubility produce more effective conduction blockade at lower concentrations (lower percentage solutions or smaller volumes deposited) than the less lipid soluble solutions. Onset time is related to the pKa of the anesthetic. The degree of protein binding will determine the duration of the local anesthetic.

Reference:

Malamed SF: Handbook of Local Anethesia, 4th Edition. St. Louis, Mosby, 1997 p. 20

Page 6: Anaesthesia TMJ Pain Orthognathic Pathology Reconstruction

2007 Oral and Maxillofacial Surgery Self Assessment Tool (OMSSAT)

The American Board of Oral and Maxillofacial Surgery 6

4. A 36 year-old atopic male presents for the extraction of a tooth. He has a past history of an anaphylactic reaction when undergoing a previous dental procedure under local anesthesia. Which of the following solutions would be best to use in this individual for future dental procedures?

A. Lidocaine 2% with epinephrine 1:100,000 (multidose vial) B. Prilocaine 4% plain (dental cartridge) C. Mepivacaine 3% (multidose vial) D. Bupivacaine 0.5% with epinephrine 1:200,000 (dental cartridge) Answer: B Rationale:

With a history of atopy, one can assume that the greatest number of potential allergens should be eliminated from any medications administered. Multi-dose vials of local anesthetics often contain preservatives such as parabens which are allergenic; this is not the case with single use dental cartridges. The presence of a vasoconstrictor usually is accompanied by an antioxidant such as a bisulfite which also may be allergenic, and should be avoided in cases of atopy. Therefore, a non-epinephrine containing solution without preservatives would be most indicated: prilocaine 4% in a dental cartridge.

Reference:

Malamed S: Handbook of Local Anesthesia. Mosby, St. Louis, 1997

Page 7: Anaesthesia TMJ Pain Orthognathic Pathology Reconstruction

2007 Oral and Maxillofacial Surgery Self Assessment Tool (OMSSAT)

The American Board of Oral and Maxillofacial Surgery 7

5. Which of the following is the cardiovascular manifestation of lidocaine toxicity?

A. Bradycardia B. Premature ventricular contractions (PVC’s) C. Prolonged QT interval D. Hypertension Answer: A Rationale:

Lidocaine has a depressor effect on the myocardium. Lidocaine toxicity causes sinus bradycardia because lidocaine increases the effective refractory period relative to the action potential duration and lowers cardiac automaticity. The bradycardia is followed by impaired contractility, massive peripheral vasodilation, hypotension and possible cardiac arrest. Lidocaine may be used to treat PVC's and lidocaine toxicity produces hypotension, not hypertension.

Reference:

Malamed SF: Handbook of Local Anesthesia, 4th Edition. St. Louis, Mosby, 1997 p. 269-270

Page 8: Anaesthesia TMJ Pain Orthognathic Pathology Reconstruction

2007 Oral and Maxillofacial Surgery Self Assessment Tool (OMSSAT)

The American Board of Oral and Maxillofacial Surgery 8

6. Prolonged muscle relaxation can result from the concomitant use of succinylcholine and which of the following local anesthetics?

A. Bupivicaine B. Procaine C. Mepivicaine D. Articaine Answer: B Rationale:

Procaine is an ester local anesthetic and metabolized in the blood by plasma cholinesterase. Succinylcholine is a depolarizing muscle relaxant that also requires plasma cholinesterase for hydrolysis. Prolonged apnea or paralysis may result form the concomitant use of these drugs. Bupivicaine, mepivicaine and articaine are all amides and thus metabolized in the liver.

Reference: Malamed SF: Handbook of Local Anesthesia, 4th Edition. St. Louis, Mosby, 1997 p. 35

Page 9: Anaesthesia TMJ Pain Orthognathic Pathology Reconstruction

2007 Oral and Maxillofacial Surgery Self Assessment Tool (OMSSAT)

The American Board of Oral and Maxillofacial Surgery 9

7. Geriatric increases in anesthetic sensitivity is most closely associated with:

A. a decrease in the number of neurons is compensated for by a increased cerebral metabolic rate.

B. an increase in cerebral metabolic rate, which is unrelated to cerebral blood flow. C. a decrease in levels of neurotransmitters and receptors in different regions of the brain. D. a decrease in the number of neurons, which is related to an increase in cerebral blood

flow. Answer: C Rationale:

In the elderly, there is a reduction in the number of neurons; this is matched by an decrease in the cerebral metabolic rate. The cerebral metabolic rate is directly related to the cerebral blood flow. The decrease in the neurons and neurotransmitters is related to a decrease in the cerebral blood flow.

Reference:

Cole, D.J. and Schlunt, M.: Adult Perioperative Anesthesia 2004. Philadelphia, Mosby. Pp.249 and 466. Power, I. and Kam, P.: Physiology for the Anaesthetist. 2001. London, Arnold, pp42-44. Stoelting, R.K. and Dierdorf, S. F.: Anesthesia and Co-Existing Disease. 4th Edition. 2002. Philadelphia, Churchill Livingstone, pp. 238-9.

Page 10: Anaesthesia TMJ Pain Orthognathic Pathology Reconstruction

2007 Oral and Maxillofacial Surgery Self Assessment Tool (OMSSAT)

The American Board of Oral and Maxillofacial Surgery 10

8. The efficiency of gaseous exchange in the elderly decreases as a result of:

A. a decrease in the closing volume. B. a reduced alveolar surface area. C. a decreased alveolar capillary membrane thickness. D. a decreased V/Q ratio. Answer: B Rationale:

In the elderly there is an increase in the closing volume, an increase in the alveolar capillary membrane thickness and an increase in the V/Q (ventilation/perfusion) ratio. The gaseous exchange is decreased because of reduced alveolar surface area.

Reference:

Power, I. and Kam, P.: Principles of Physiology for the Anaesthetist 2001. London, Oxford Press. pp. 367-8 and 73-92. Cole, D.J. and Schlunt, M.: Adult Perioperative Anesthesia. 2004. Philadelphia, Mosby. p. 467.

Page 11: Anaesthesia TMJ Pain Orthognathic Pathology Reconstruction

2007 Oral and Maxillofacial Surgery Self Assessment Tool (OMSSAT)

The American Board of Oral and Maxillofacial Surgery 11

9. When comparing a morbidly obese patient to a non-obese patient, which of the following statements is correct?

A. Oxygen consumption is higher in the non-obese patient. B. Functional residual capacity is the same. C. Time to desaturation with a period of apnea is the same. D. Positioning may diminish pulmonary reserve more in the obese patient. Answer: D Rationale:

Patients who are morbidly obese have changes in pulmonary, cardiovascular, gastrointestinal, and metabolic systems. Patients who are morbidly obese have increased minute ventilation at rest to meet the metabolic needs of the increased tissue mass. Changes in lung volumes at rest include reduced FRC, vital capacity, and total lung capacity. Closing volume is unchanged and reduced FRC can result in lung volumes below closing capacity in normal tidal ventilation. Anesthesia compounds these problems with greater reductions in FRC in obese patients compared to nonobese patients of the same age. As a result, obese patient's ability to tolerate periods of apnea is reduced. Patient positioning aggravates these changes in lung volumes and contributes to poor respiratory reserve in obese patients. Reverse Trendelenberg is the most optimal position for lung volumes whereas supine position and Trendelenberg are worst in terms of safe apnea periods and recovery time.

Reference:

Stoelting RK, Dierdorf SF. Anesthesia and Co-Existing Disease, 2002. Todd DW. Anesthetic Considerations for the Obese and Morbidly Obese Oral and Oral and Maxillofacial Surgery Patient. J Oral and Maxillofacial Surgery 63:1348-1353, 2005.

Page 12: Anaesthesia TMJ Pain Orthognathic Pathology Reconstruction

2007 Oral and Maxillofacial Surgery Self Assessment Tool (OMSSAT)

The American Board of Oral and Maxillofacial Surgery 12

10. In the elderly, differences in drug response include a/an:

A. increase in MAC. B. decreased rate of hepatic glucuronidation of morphine. C. lowered induction dose of thiopental. D. shorter recovery time to the normal ventilatory response with fentanyl. Answer: C Rationale:

The elderly require a lower dose of thiopental for the induction of anesthesia. With increasing age, the MAC for inhalation anesthetics decreases. The recovery of normal ventilatory drive after fentanyl is delayed. Regarding the rate of hepatic synthetic reactions: glucuronidation of morphine is also unchanged in the elderly but the rate of hepatic oxidative and reductive reactions are decreased with an increase in age.

Reference:

Longnecker, DE et al.:Principles and Practice of Anesthesiology. 2nd Edition, 1997. St. Louis: Mosby. pp. 481-2. Calvey, T.N. and Williams, N.E.: Pharmacology for Anaesthetists 4th Edition 2001, London, Blackwell Science pp 106-112. Cole, D.J. and Schlunt, M.: Adult Perioperative Anesthesia. 2004. Philadelphia, Mosby, pp.468-470.

Page 13: Anaesthesia TMJ Pain Orthognathic Pathology Reconstruction

2007 Oral and Maxillofacial Surgery Self Assessment Tool (OMSSAT)

The American Board of Oral and Maxillofacial Surgery 13

11. Which of the following agents is associated with the highest incidence of nausea and vomiting in the post-operative period?

A. Etomidate B. Propofol C. Ketamine D. Clonidine Answer: A Rationale:

Etomidate is associated with a high incidence of nausea and vomiting. Although ketamine can cause nausea and vomiting the incidence is much lower. Propofol has antiemetic effects at higher dosages. Clonidine has a low incidence of post operative nausea and may be beneficial in the treatment of cyclical vomiting syndrome.

Reference:

Miller RD (ed) Anesthesia 4th edition. New York, Churchill Livingstone, 1994; p268 Stoelting RK: Pharmacology and Physiology in Anesthetic Practice, 2nd Edition. Philadelphia, JB Lippincott, 1991.

Page 14: Anaesthesia TMJ Pain Orthognathic Pathology Reconstruction

2007 Oral and Maxillofacial Surgery Self Assessment Tool (OMSSAT)

The American Board of Oral and Maxillofacial Surgery 14

12. Which drug should be avoided in the patient with pre-existing renal dysfunction?

A. Ketorolac (Toradol) B. Midazolam (Versed) C. Methohexital (Brevital) D. Droperidol (Inapsine) Answer: A Rationale:

Barbiturates and benzodiazepines both reduce glomerular filtration rate but can be used in patients with renal failure. Doses need to be reduced and the anesthetic medication titrated to effect. Opioids, such as morphine and meperidine have active metabolites that are renally excreted and require cautious administration. The glucoronidated morphine compound is more potent than morphine itself. Droperidol, although, not generally used in outpatient anesthesia currently produces an �-adrenergic blocking effect that preserves renal blood flow and hemodynamics. Ketorolac or other NSAIDs are prostaglandin inhibitors that interfere with the prostaglandin mediated renal vasodilatation and has been shown to produce acute renal failure.

Reference:

Altee John L. Complications in Anesthesia. WB Saunders 1999 Roizen MF, Fleisher LA. Essence of Anesthesia Practice. WB Saunders 1997

Page 15: Anaesthesia TMJ Pain Orthognathic Pathology Reconstruction

2007 Oral and Maxillofacial Surgery Self Assessment Tool (OMSSAT)

The American Board of Oral and Maxillofacial Surgery 15

13. A 22 year-old female presents for removal of her third molars under deep sedation. She has a history of Wolff-Parkinson-White Syndrome (WPW). Midazolam, fentanyl and propofol are administered, and she develops atrial fibrillation consistent with the re-entry phenomenon of WPW. The most appropriate medication to treat this problem is:

A. adenosine.

B. diltiazem.

C. esmolol.

D. amiodarone.

Answer: D Rationale:

Drugs such as adenosine, calcium channel blockers, and beta blockers can cause a paradoxical increase in the ventricular response to the rapid atrial impulses of atrial fibrillation. This increase in ventricular response occurs because these agents slow or block conduction through the AV node and in some instances may facilitate conduction to the ventricle via the accessory pathway. The treatment of choice for a atrial fibrillation associated with Wolff-Parkinson-White syndrome is direct cardioversion, which if inappropriate, is then followed preferentially by amiodarone.

Reference:

ACLS: Principles and Practice, Pp. 328, American Heart Association 2003 GE Morgan et al, Clinical Anesthesiology, pp. 385, Lange/McGraw-Hill, 2002

Page 16: Anaesthesia TMJ Pain Orthognathic Pathology Reconstruction

2007 Oral and Maxillofacial Surgery Self Assessment Tool (OMSSAT)

The American Board of Oral and Maxillofacial Surgery 16

14. A 25 year-old female is sedated with nitrous oxide and intravenous ketamine. In the recovery area the patient is noted to be hallucinating. This could possibly have been prevented by concomitant use of

A. scopolamine. B. phenothiazenes. C. propofol. D. droperidol. Answer: C Rationale:

Ketamine delirium may be prevented by concomitant use of a benzodiazepine or propofol. The other 3 answers may also be responsible for post operative / emergent excitement.

Reference:

Firestone, Lebowitz and Cook: Clinical Anesthesia Procedures of the Massachusetts General Hospital , Third edition, P. 497 Barash, Cullen and Stoelting, Handbook of Clinical Anesthesia, Fourth edition, P.864

Page 17: Anaesthesia TMJ Pain Orthognathic Pathology Reconstruction

2007 Oral and Maxillofacial Surgery Self Assessment Tool (OMSSAT)

The American Board of Oral and Maxillofacial Surgery 17

15. A 65 year-old 45 kg frail cardiac transplant patient presents for extractions. Local anesthesia with mepivicaine 3% 270 mg and sedation with midazolam 1.25 mg i.v. with a 50% mixture of nitrous oxide and oxygen is used. The patient becomes hypotensive and bradycardic with a narrow complex QRS complex. What drug or intervention would you use in the management of this patient?

A. Flumazenil 4 mg IV push B. Fluid bolus of 500 mg normal saline C. Dopamine 200 µg/min IV infusion D. Atropine 0.5mg IV push Answer: C Rationale:

The cardiac transplant patient's chronic problems may include diffuse accelerated atherosclerosis leading to generally non-anginal myocardial ischemia and decreased cardiac output. Additionally, the chronotropic response is markedly decreased in the denervated transplanted heart. This patient is suffering from symptomatic bradycardia. One contributing factor could be the local anesthetic. Although the dosage of mepivicaine administered is within the upper acceptable recommended guidelines the practitioner must take into consideration the patient. Cardiovascularly compromised patients may be on several drugs that interact with intraoperatively administered medications. It would not be uncommon for a patient with this history to be taking an additional local anesthetic type dysrhythmic drug acting on her myocardium (such as procainamide.) Additionally, a local anesthetic injection into or near the pterygoid plexus can cause a precipitous rise in local anesthetic blood levels, especially with a lack of vasoconstrictor. The appropriate first step would be external pacing if available. If this is not available, a dopamine infusion titrated between 5-20 µg/kg/min would give both alpha and beta effects to raise heart rate and blood pressure. Since this patient's transplanted sinus node is denervated, atropine would not reverse her bradycardia. Flumazenil should be given in incremental doses of 0.05-0.1 mg slowly. For a frail patient, a 500 ml fluid bolus might significantly increase cardiac preload and further decrease cardiac output.

Reference:

ACLS Provider manual, 2001 American Heart Association, pp145-155 Weinberg, GL: Basic Science Review of Anesthesiology McGraw-Hill, New York, 1997 p 25

Page 18: Anaesthesia TMJ Pain Orthognathic Pathology Reconstruction

2007 Oral and Maxillofacial Surgery Self Assessment Tool (OMSSAT)

The American Board of Oral and Maxillofacial Surgery 18

16. Which of the following agents can be used to reverse the effects of dexmedetomidine?

A. Flumazenil B. Narcan C. Atipamezole D. Atropine Answer: C Rationale:

Flumazenil and Narcan are used to reverse effects of benzodiazepines and narcotics, respectively. Atropine is an antimuscarinic and does not reverse deximedetomidine. Atipamezole is an alpha2-adrenoceptor antagonist with an imidazole structure. It rapidly reverses sedation/anesthesia induced by alpha2-adrenoceptor agonists. In humans, atipamezole at doses up to 30 mg produces no cardiovascular or subjective side effects, while at a high dose (100 mg) it produced subjective symptoms, such as motor restlessness, and an increase in blood pressure.

Reference:

Jones JG, Taylor PM. Receptor specific reversible sedation: dangers of vascular effects. Anesthesiology 1999;90: 1489-1490. Scheinin H, Aantaa R, et al. Reversal of the sedative and sympatholytic effects of dexmedetomidine with a specific alpha 2 adrenoceptor antagonist atipamezole: a pharmacodynamic and kinetic study in healthy volunteers. Anesthesiology 1998;89:574-584.

Page 19: Anaesthesia TMJ Pain Orthognathic Pathology Reconstruction

2007 Oral and Maxillofacial Surgery Self Assessment Tool (OMSSAT)

The American Board of Oral and Maxillofacial Surgery 19

17. A 40 year-old male Type II diabetic arrives at your office for the extraction of multiple teeth under intravenous sedation. He is combative and disoriented. He has been NPO for six hours. He took his diabetic medications (acarbose and glyburide) this morning. A finger stick glucose is 50. What is the next appropriate step?

A. Fruit juice PO B. Intramuscular 50% Dextrose C. Intramuscular glucagon D. Dextrose tablet dissolved sublingually Answer: C Rationale:

Hypoglycemia after fasting while on oral hypoglycemics is a concern in office anesthesia settings, and patients must understand that they should avoid immediate preoperative hypoglycemic medications in that setting. This patient's medications include a second generation sulfonylurea (glyburide) which increases endogenous insulin release; and an α-glucosidase inhibitor (acarbose) which blocks intestinal breakdown of fructose and dextrose into absorbable glucose. In this case intake of fructose-containing juice is not indicated. Although one could consider oral administration of glucose containing tablets or solution, oral administration in a disoriented patient is also relatively contraindicated. Intramuscular administration of the very hyperosmotic 50% dextrose would cause extreme muscle irritation; its use should be limited to intravenous administration in a large vein. Dextrose is not absorbed significantly through sublingual epithelium. Glucagon 1 mg IM would be indicated. This would be absorbed and cause hepatic glycogenolysis and gluconeogenesis, raising blood glucose levels in about 15 minutes. Glucagon may cause significant nausea.

Reference:

Sarasin D: Ambulatory anesthetic management of the patient with diabetes. Oral Maxilofac Surg Clin N Amer 11(4): 589-99, 1999 Washington Manual of Medical Therapeutics, 30th ed, Liippincott, Philadelphia, 2001 pp463-66

Page 20: Anaesthesia TMJ Pain Orthognathic Pathology Reconstruction

2007 Oral and Maxillofacial Surgery Self Assessment Tool (OMSSAT)

The American Board of Oral and Maxillofacial Surgery 20

18. What effect will ketamine have on the degree of regurgitation in a patient with mitral valve prolapse with regurgitation?

A. Increase B. Decrease C. No effect D. Variable Answer: A Rationale:

Ketamine is discouraged in patients with mitral valve prolapse with regurgitation, due to its sympathomimetic actions. It will increase vascular resistance and worsen regurgitant flow.

Reference:

Waxman K, Shoemaker WC, Lippman M: Cardiovascular effects of anesthetic induction with ketamine. Anesth Analg 1980; 59:355-8. White PF, Way WL, Trevor AJ: Ketamine: Its pharmacology and therapeutic uses. Anesth 1982; 56:119-36.

Page 21: Anaesthesia TMJ Pain Orthognathic Pathology Reconstruction

2007 Oral and Maxillofacial Surgery Self Assessment Tool (OMSSAT)

The American Board of Oral and Maxillofacial Surgery 21

19. Barbiturates have which of the following effects on the myocardium?

A. Directly sensitize the myocardium to arrhythmias B. Directly increase myocardial contractility C. Indirectly increase heart rate by inducing venodilation D. Indirectly increase myocardial contractility Answer: C Rationale:

Barbiturates have no effect on myocardial sensitization. They decrease myocardial contractility. Reflex tachycardia is common after an induction dose of barbiturate to compensate for the vasodilatation.

Reference:

Miller RD (ed) Anesthesia 4th edition. New York, Churchill Linignstone, 1994; p238 Lieblich SE. Methohexital Versus Propofol of Outpatient Anesthesia Part 1: Methohexatal is Superior. JOMS 58:811-815, 1995

Page 22: Anaesthesia TMJ Pain Orthognathic Pathology Reconstruction

2007 Oral and Maxillofacial Surgery Self Assessment Tool (OMSSAT)

The American Board of Oral and Maxillofacial Surgery 22

20. Ketamine’s direct effect on the heart is:

A. chronotropic depression.

B. chronotropic stimulation.

C. inotropic depression.

D. inotropic stimulation.

Answer: C Rationale:

Ketamine direct action on the myocardium is a negative inotropic effect. Its centrally mediated sympathetic responses (indirect activation of the sympathetic nervous system) usually override the depression. Ketamine causes an increase in circulating catecholamines, especially norepinephrine, by inhibiting reuptake at postganglionic sympathetic neurons.

Reference:

Reich DL, Silvay G. Ketamine: an update on the first twenty-five years of clinical experience. Can J Anaesth 1989;36:186–97. Hirota K, Lambert DG. Ketamine: its mechanism(s) of action and unusual clinical uses. Br J Anaesth 1996; 77:441–4.

Page 23: Anaesthesia TMJ Pain Orthognathic Pathology Reconstruction

2007 Oral and Maxillofacial Surgery Self Assessment Tool (OMSSAT)

The American Board of Oral and Maxillofacial Surgery 23

21. Which of the following is a side effect associated with etomidate?

A. Decreased venous return and myocardial contractility B. Intra-arterial injection causing nerve injury and gangrene C. Adrenal suppression lasting at least 6 hours D. Triggering of porphyria in susceptible individuals Answer: C Rationale:

Etomidate maintains hemodynamic stability and has little effect on the heart. Barbiturates are known to have severe effects associated with intra-arterial injection. This group is also responsible for porphyria in susceptible individuals. Etomindate causes adrenal suppression and steroid administration may be necessary in patients already having adrenal axis suppression.

Reference:

Miller RD (ed) Anesthesia 4th edition. New York, Churchill Livingstone, 1994; p268 Dembo JB. Methohexital Versus Propofol of Outpatient Anesthesia Part II: Propofol is Superior. JOMS 58:816-820, 1995

Page 24: Anaesthesia TMJ Pain Orthognathic Pathology Reconstruction

2007 Oral and Maxillofacial Surgery Self Assessment Tool (OMSSAT)

The American Board of Oral and Maxillofacial Surgery 24

22. Which of the following neuromuscular blockers requires little if any dosage .change in the elderly patient?

A. Pancuronium B. Vecuronium C. Cisatracurium D. Mivacurium

Answer: C Rationale:

Cisatracurium is the neuromuscular blocker that is metabolized by Hoffman degeneration and organ-independent elimination. This process is non-enzymatic and occurs spontaneously and thus needs little if any change in the routine dose in the elderly. Also the clinical effect is not prolonged. Pancuronium is dependent on renal function and excretion which decreases with age. Vecuronium is dependent on hepatic function which also decreases with age; and mivacurium is metabolized by plasma cholinesterase which also decreases with age. Because of the age dependent nature of metabolism for these three neuromuscular blockers, they all require a reduction in dosage for the elderly.

Reference:

Cole, D.J and Schlunt, M: Adult Perioperative Anesthesia. 2004. Philadelphia, Mosby, p. 471. Vickers, M.D. and Power, I.: Medicine for Anaesthetists. 4th Edition, 1999. London, Blackwell Science, p. 114

Page 25: Anaesthesia TMJ Pain Orthognathic Pathology Reconstruction

2007 Oral and Maxillofacial Surgery Self Assessment Tool (OMSSAT)

The American Board of Oral and Maxillofacial Surgery 25

23. A 72 year-old male presents for removal of tooth #3. His written medical history is significant for stable angina, congestive heart failure, type 2 diabetes mellitus, and coronary stent placed after myocardial infarction 3 months ago. He is taking metoprolol and furosemide. A history and physical examination reveals the following: no chest pain with mild exercise, reduced carotid pulses, audible 3rd and 4th heart sound, and inspiratory basilar rales. The most influential determinant of this patient’s perioperative cardiac risk status is: A. stable angina.

B. coronary stent placement following MI 3 months ago.

C. congestive heart failure.

D. diabetes mellitus.

Answer: C Rationale:

This patient has signs of acute congestive heart failure. Physical signs of left ventricular CHF include: reduced carotid pulsations, diffuse and laterally displaced apical impulse, palpable and audible 3rd and 4th heart sounds, accentuated pulmonic 2nd sound, inspiratory basilar rales, and right-sided pleural effusion. There are several factors which contribute to determining a patient's overall preoperative cardiac risk. As shown in the ACC/AHA 2002 Guidelines Update on Perioperative Cardiovascular Evaluation for Noncardiac Surgery and Goldman et al. (Multifactorial index of cardiac risk in non-cardiac surgical procedures. N Engl J med 1977;297:845-50.), the preoperative factor which had the highest predictive value for an adverse cardiac event is the patient's acute congestive heart failure.

Reference:

ACC/AHA 2002 Guidelines Update on Perioperative Cardiovascular Evaluation for Noncardiac Surgery . Circulation. March 5,2002 Goldman et al.: Multifactorial index of cardiac risk in non-cardiac surgical procedures. N Engl J med 1977;297:845-50.

Page 26: Anaesthesia TMJ Pain Orthognathic Pathology Reconstruction

2007 Oral and Maxillofacial Surgery Self Assessment Tool (OMSSAT)

The American Board of Oral and Maxillofacial Surgery 26

24. In a patient with interstitial lung disease (ILD) why would one expect the FEV1/FVC ratio to be normal?

A. Vital capacity is reduced in ILD B. FEV1 is normal in a restrictive pattern C. ILD does not alter PFT’s D. ILD affects only the diffusing capacity of CO2 Answer: A Rationale:

In ILD a known or unknown agent stimulates macrophage induced alveolitis. This results in a loss of functioning alveolar capillary units and an accumulation of fibroblasts. The accumulation of collagen in the alveolar interstitium produces a thickened, scarred and eventually small lung. The FEV/FVC ratio is typically normal in ILD due to a tendency for patients to have small lung volumes. Small lung volumes (VC – vital capacity, TLC – total lung capacity) will be noted on both PFT examination and Chest radiographs. While FEV1 is decreased in ILD, the FVC is decreased at the same time resulting in a normal FEV/FVC ratio.

Reference:

Braunwald, E, et al: Interstitial Lung Disease: Harrison's Principles of Internal Medicine. McGraw-Hill Co. Alex, CG and Tobin, MJ: Assessment of Pulmonary Function in Critically Ill Patients, in Textbook of Critical Care, Shoemaker, WC. 4th Edition. W B Saunders Co. 2000.

Page 27: Anaesthesia TMJ Pain Orthognathic Pathology Reconstruction

2007 Oral and Maxillofacial Surgery Self Assessment Tool (OMSSAT)

The American Board of Oral and Maxillofacial Surgery 27

25. Which effect is seen with propofol?

A. Elevation of intracranial pressure B. Increases in intraocular pressure C. Potentiation of neuromuscular blockade D. Increases in bronchodilation Answer: D Rationale:

Propofol has a direct smooth muscle effect on the bronchi, causing bronchodilation. Propofol decreases intracranial and intraocular pressure, and it does not potentiate neuromuscular blockade.

Reference:

Miller RD (ed) Anesthesia 4th edition. New York, Churchill Livingstone, 1994; p270-272 Dembo JB. Methohexital versus Propofol of Outpatient Anesthesia Part II: Propofol is Superior. JOMS 58:816-820, 1995

Page 28: Anaesthesia TMJ Pain Orthognathic Pathology Reconstruction

2007 Oral and Maxillofacial Surgery Self Assessment Tool (OMSSAT)

The American Board of Oral and Maxillofacial Surgery 28

26. Which drug must be used with caution in a patient with a history of epilepsy?

A. Propofol B. Fentanyl C. Dexmedetomidine D. Methohexital Answer: D Rationale:

Methohexital can cause seizure activity; and therefore is contraindicated in patients with temporal lobe seizures. The other three drugs are not associated with seizure-like activity.

Reference:

Cote, Charles J., Pediatric Anesthesia, pg. 2375 in Miller's Anesthesia, Sixth Edition, Volume 2, by Elsevier Inc. Stoelting, Robert K. MD, Dierdorf, Stephen F, MD, Chapter 17, Anesthesia and Coexisting Disease, Fourth Edition, pg. 284,

Page 29: Anaesthesia TMJ Pain Orthognathic Pathology Reconstruction

2007 Oral and Maxillofacial Surgery Self Assessment Tool (OMSSAT)

The American Board of Oral and Maxillofacial Surgery 29

27. A 61 year-old patient presents for removal of tooth #2, and general anesthesia He has aortic regurgitation. The perioperative management of this patient includes which of the following?

A. Relative hypovolemia B. Negative inotropic agents C. Increase afterload D. Positive chronotropic agents Answer: D Rationale:

The perioperative goals of managing aortic regurgitation are to promote forward flow by decreasing afterload and to maintain a normal to slightly increased heart rate.

Reference:

Goldman et al. (Multifactorial index of cardiac risk in non-cardiac surgical procedures. N Engl J med 1977;297:845-50. DE Longnecker, FL Murphy, Introduction to Anesthesia, 9th ed., pp. 291, Saunders, 1997 GE Morgan et al, Clinical Anesthesiology, pp. 414, Lange/McGraw-Hill, 2002

Page 30: Anaesthesia TMJ Pain Orthognathic Pathology Reconstruction

2007 Oral and Maxillofacial Surgery Self Assessment Tool (OMSSAT)

The American Board of Oral and Maxillofacial Surgery 30

28. A 49-year-old patient presents for removal tooth #15 under general anesthesia. His clinical

examination is significant for a IV/VI holosystolic murmur heard at the apical area and radiates to left axilla. The perioperative management should include which of the following?

A. Decrease intravascular volume B. Decrease myocardial contractility C Increase peripheral vascular resistance D Increase heart rate Answer: D Rationale:

A holosystolic murmur heard at the apex and radiating to the left axilla is consistent with a mitral regurgitation. The perioperative goals of managing mitral regurgitation are to promote forward flow by decreasing afterload and to maintain a normal to slightly increased heart rate.

Reference:

DE Longnecker, FL Murphy, Introduction to Anesthesia, 9th ed., pp. 291, Saunders, 1997 GE Morgan et al, Clinical Anesthesiology, pp. 414, Lange/McGraw-Hill, 2002

Page 31: Anaesthesia TMJ Pain Orthognathic Pathology Reconstruction

2007 Oral and Maxillofacial Surgery Self Assessment Tool (OMSSAT)

The American Board of Oral and Maxillofacial Surgery 31

29. An 18 year-old with Cerebral Palsy (CP) requires intubation for a general anesthetic. Succinylcholine is contraindicated in this patient secondary to

A. structural similarity to acetylcholine. B. potential for a significant increase in circulating levels of potassium. C. deficiency of pseudocholinesterase. D. fasciculation and post-operative muscle pain. Answer: B Rationale:

Circulating potassium increases significantly in patients with CP following administration of succinylcholine and may lead to lethal arrhythmias.

Reference:

Firestone, Lebowitz and Cook: Clinical Anesthesia Procedures of the Massachusetts General Hospital , Third edition, P. 171 Barash, Cullen and Stoelting, Handbook of Clinical Anesthesia, Fourth edition, P.585

Page 32: Anaesthesia TMJ Pain Orthognathic Pathology Reconstruction

2007 Oral and Maxillofacial Surgery Self Assessment Tool (OMSSAT)

The American Board of Oral and Maxillofacial Surgery 32

30. A patient with which condition is least likely to be an aspiration risk during an outpatient non-intubated general anesthetic?

A. Systemic lupus erythematosus B. Parkinson’s disease C. Myasthenia gravis D. Diabetes mellitus Answer: A Rationale:

Parkinson's disease, myasthenia gravis, a transection of the vertebral column above T4, and diabetes mellitus all increase the risk of aspiration. Parkinson's disease and myasthenia gravis impair the patient's ability to control their airway. Parkinson’s' disease is a neurodegenerative disease characterized by a loss of dopaminergic neurons in the substantia nigra of the basal ganglia. Clinical signs and symptoms include a resting tremor, rigidity, postural instability and bradykinesia. Myasthenia gravis is an autoimmune disease of the neuromuscular junction. Clinical symptoms include fatigue, weakness of the striated muscles, diplopia, inspiratory muscle weakness, and bulbar weakness with impaired ability to handle secretions and swallow. The diabetic patient may have autonomic dysfunction resulting in gastroparesis. Frequently gastroparesis is undiagnosed and asymptomatic. This increases the risk of gastric aspiration. The patient with systemic lupus erythematosus may present anesthetic airway and pulmonary concerns such as reduced TMJ range of motion, decreased arytenoid movement, diaphragmatic dysfunction, pulmonary infiltrates and reduced PFTs. However, the patient is not at increased risk for aspiration.

Reference:

Altee John L. Complications in Anesthesia. WB Saunders 1999 Roizen MF, Fleisher LA. Essence of Anesthesia Practice. WB Saunders 1997

Page 33: Anaesthesia TMJ Pain Orthognathic Pathology Reconstruction

2007 Oral and Maxillofacial Surgery Self Assessment Tool (OMSSAT)

The American Board of Oral and Maxillofacial Surgery 33

31. The use of succinylcholine is acceptable in which patient population?

A. Behcet’s muscular dystrophy B. Four month old spinal cord injury C. Multiple sclerosis D. Myasthenia gravis Answer: D Rationale:

Myasthenia gravis (MG) is a neuromuscular disease that is characterized by weakness and fatigability of skeletal muscles. The MG patient is more sensitive to the use of nondepolarizing relaxants. However, because there are fewer functional receptors the MG patient may demonstrate increased resistance to depolarizing muscular relaxants. While many anesthesiologists may prefer to avoid neuromuscular blocking agents, succinylcholine (in higher doses) can be used in the MG patient and provide satisfactory intubating conditions. Succinylcholine may precipitate hyperkalemia in muscular dystrophy, spinal cord injuries less than 6 to 8 months and MS.

Reference:

Altee John L. Complications in Anesthesia. WB Saunders 1999 Roizen MF, Fleisher LA. Essence of Anesthesia Practice. WB Saunders 1997

Page 34: Anaesthesia TMJ Pain Orthognathic Pathology Reconstruction

2007 Oral and Maxillofacial Surgery Self Assessment Tool (OMSSAT)

The American Board of Oral and Maxillofacial Surgery 34

32. A 17 year-old asthmatic with a preoperative FEV1/FVC of 85% requires which preoperative treatment prior to induction of general anesthesia?

A. Ipratropium MDI B. Nebulized Racemic Epinephrine C. Advair MDI D. No treatment indicated Answer: D Rationale:

No treatment is required for a normal FEV1/FVC. Reference:

Firestone, Lebowitz and Cook: Clinical Anesthesia Procedures of the Massachusetts General Hospital , Third edition, P. 35-7 Barash, Cullen and Stoelting, Handbook of Clinical Anesthesia, Fourth edition, P. 419-22

Page 35: Anaesthesia TMJ Pain Orthognathic Pathology Reconstruction

2007 Oral and Maxillofacial Surgery Self Assessment Tool (OMSSAT)

The American Board of Oral and Maxillofacial Surgery 35

33. A 52 year-old obese patient with a hiatal hernia requires induction of general anesthesia. Which premedicant could be considered to minimize the patient’s risk of respiratory problems?

A. Glycopyrolate B. Amitriptyline C. Metoclopramide D. Meperidine Answer: C Rationale:

Metoclopramide would increase gastric emptying and increase esophageal sphincter tone. This would help decrease the risk of aspiration. All the other choices decrease esophageal sphincter tone and would increase the risk of aspiration.

Reference:

Firestone, Lebowitz and Cook: Clinical Anesthesia Procedures of the Massachusetts General Hospital , Third edition, P. 624-5 Barash, Cullen and Stoelting, Handbook of Clinical Anesthesia, Fourth edition, P. 549

Page 36: Anaesthesia TMJ Pain Orthognathic Pathology Reconstruction

2007 Oral and Maxillofacial Surgery Self Assessment Tool (OMSSAT)

The American Board of Oral and Maxillofacial Surgery 36

34. Which of the following statements is correct regarding the LMA (laryngeal mask airway?

A. The classic LMA when in correct position protects against aspiration of stomach contents

B. Insufflation of the stomach can occur with excess pressure exerted through the LMA. C. Administration of emergency drugs via a LMA or endotracheal tube has equivalent

success rates. D. A cuffless LMA is available for young children to minimize potential soft tissue

potential ischemia. Answer: B Rationale:

The laryngeal mask airway is an airway device that is placed in the hypopharynx above the opening of the larynx. The LMA does not protect against aspiration of stomach contents and insufflation of the stomach can be expected with pressures above 20 cm of water. The black orientation line faces cephalad when in correct position. Delivery of emergency drugs through a LMA are less successful (approximately 20% successful administration) than via an endotracheal tube. All sizes of the LMA have a cuff that allows for seating of the LMA in the hypopharynx.

Reference:

The LMA Manual. The Laryngeal Mask Airway Company, LTD. Todd DW. Use of the LMA for Outpatient General Anesthetics, Pro. J Oral and Maxillofac Surg, 61:2003.

Page 37: Anaesthesia TMJ Pain Orthognathic Pathology Reconstruction

2007 Oral and Maxillofacial Surgery Self Assessment Tool (OMSSAT)

The American Board of Oral and Maxillofacial Surgery 37

35. A 7 year-old male needs a general anesthetic and intubation for an elective surgical procedure. He presents with malaise, a productive cough and thick nasal discharge. How long will you wait to reschedule the procedure?

A. No delay necessary B. 7 to 10 days C. 2 to 3 weeks D. 1 ½ to 2 months Answer: D Rationale:

It is likely that this child has an upper respiratory infection (URI). Signs of an URI include fever, fatigue, loss of appetite, productive cough, and thick nasal discharge. Children with URI have an irritable airway and are at increased risk for laryngospasm, bronchospasm, postintubation croup, pneumonia, and episodes of desaturation. Bronchial hyperreactivity may last 4 to 6 weeks after an URI; it is recommended to delay treatment for at least 6 weeks. If urgent or emergency surgery is necessary a LMA may reduce complications associated with an irritable airway.

Reference:

Cote, Charles J., Pediatric Anesthesia, pg. 2381 - 2382 in Miller's Anesthesia, Sixth Edition, Volume 2, by Elsevier Inc. Dembo, Jeffrey B., Pediatric Consideration in Office Anesthesia, pg. 108, OMS Knowledge Update, Vol. 1, Part 1, August 1994

Page 38: Anaesthesia TMJ Pain Orthognathic Pathology Reconstruction

2007 Oral and Maxillofacial Surgery Self Assessment Tool (OMSSAT)

The American Board of Oral and Maxillofacial Surgery 38

36. Which mode of mechanical ventilation is described as allowing the patient to trigger a breath; but can cause respiratory alkalosis when the patient is tachypneic?

A. Assist control ventilation B. Synchronized intermittent mandatory ventilation C. Pressure controlled ventilation D. Controlled mechanical ventilation Answer: A Rationale:

Mechanical ventilation may be either volume or pressure controlled. Volume controlled ventilation consists of CMV(controlled mechanical ventilation), ACV (assist controlled ventilation), IMV (intermittent controlled ventilation), SIMV (synchronized intermittent controlled ventilation). Pressure controlled ventilation is either PSV (pressure support ventilation) which is patient triggered or PCV (pressure controlled ventilation) which is ventilator triggered. CMV is most commonly used during general anesthesia in the neuromuscularly blocked patient. The various other settings can be used for ventilatory support and weaning. The ACV mode requires that the patient trigger a breath. When the patient triggers a breath the ventilator delivers a preset volume. The disadvantage of the mode is that if the patient is tachypneic excessive volume and thus a respiratory alkalosis will develop. Alternatively, IMV allows the patient to breath spontaneously and delivers positive pressure ventilation at a preset volume and rate to ensure oxygenation and ventilation. SIMV prevents PVP (positive pressure ventilation) during a spontaneous breath.

Reference:

Miller, Stoelting, Basics in Anaesthesia

Page 39: Anaesthesia TMJ Pain Orthognathic Pathology Reconstruction

2007 Oral and Maxillofacial Surgery Self Assessment Tool (OMSSAT)

The American Board of Oral and Maxillofacial Surgery 39

37. A 64 year-old female presents for removal of tooth #30 under general anesthesia. Her past medical history is significant for rheumatic heart disease, and subsequent mitral valve stenosis. The preoperative management of this patient would include which of the following?

A. Decrease intravascular volume B. Increase intravascular volume C. Maintain a slower heart rate D. Maintain a faster heart rate

Answer: C Rationale:

The principal hemodynamic goals of managing patients with mitral stenosis are to maintain a slower to normal sinus rhythm and to avoid tachycardia, large increases in cardiac output, and both hypovolemia and fluid overload by judicious fluid therapy.

Reference:

DE Longnecker, FL Murphy, Introduction to Anesthesia, 9th ed., pp. 290, Saunders, 1997 GE Morgan et al, Clinical Anesthesiology, pp. 411, Lange/McGraw-Hill, 2002

Page 40: Anaesthesia TMJ Pain Orthognathic Pathology Reconstruction

2007 Oral and Maxillofacial Surgery Self Assessment Tool (OMSSAT)

The American Board of Oral and Maxillofacial Surgery 40

38. Twenty four hours after an ultralight general anesthesia procedure for third molar removal using intravenous fentanyl, methohexital, and nasal nitrous oxide/oxygen, your 40 year-old female patient calls complaining of intense abdominal pain in all quadrants and muscle weakness. Over the next several days, confusion develops and the urine turns dark. After successful medical management, the patient relates a family history of “severe reactions” to sulfonamide antibiotics, including prolonged hospitalizations. Which of the following agents would be safest to use on this patient in the future?

A. Morphine B. Ketamine C. Isoflurane D. Diazepam Answer: A Rationale:

The symptoms are classic for acute intermittent porphyria, an autosomal dominant condition leading to a deficiency in uroporphyrinogen synthetase activity, causing an accumulation of uroporphobilogen, which is excreted in the urine turning it a dark color. Classic symptoms of an acute attack include intense abdominal pain, motor weakness, (usually starting proximally in the upper limbs) and confusion/agitation. These are due to nervous system dysfunction and demylenization. Diagnosis is by family history, increased levels of urinary porphobilogen, and deficient uroporphyrogen synthetase in the red blood cells. Treatment for an acute episode is supportive, including withdrawal of precipitating agents, carbohydrate support, hydration, and careful administration of hematin. Many agents have been implemented as potential triggering agents for acute intermittent porphyria. The most commonly cited are barbiturates. Morphine, fentanyl and its conjoiners, and nitrous oxide are considered safe. Other anesthetic agents implicated as causative or exacerbative agents for AIP include most potent inhalation volatile anesthetics, benzodiazepines, ketamine, etomidate, meperidine, and lidocaine.

Reference:

Weinberg GL (ed;): Basic Science Review of Anesthesiology, McGraw-Hill, New York, 1997 pp64-66 Benumof JL (ed.) Anesthesia and Uncommon Diseases, WB Saunders, Philadelphia, 1998 pp 162-163

Page 41: Anaesthesia TMJ Pain Orthognathic Pathology Reconstruction

2007 Oral and Maxillofacial Surgery Self Assessment Tool (OMSSAT)

The American Board of Oral and Maxillofacial Surgery 41

39. Thirty minutes after extubating a 6 year old asthmatic male patient in your office you notice the child to be in respiratory distress characterized by high pitched coarse sounds occurring during inspiration. What is the most likely diagnosis and treatment?

A. Partial laryngospasm initially managed with positive pressure ventilation B. Bronchospasm initially managed with albuterol inhaler C. Postextubation stridor initially managed with racemic epinephrine D. ost-obstructive pulmonary edema initially managed with supplemental oxygen Answer: C Rationale:

Postextubation stridor is a result of laryngeal inflammation reducing the airway DIMENSION. In a young child it may be secondary to irritation of the endotracheal tube infringing on the narrowest part of the child's airway – being the cricoid cartilage. Maneuvers to avoid such include using a cuffless tube and ensuring that there is an air leak around the tube. Movement of the patient's head during surgery can cause a displacement of the tube superiorly (essentially extubating the patient) or inferiorly (irritating the mucosa around the region of the cricoid cartilage). The risk of the child developing postextubation stridor persists for up to 24 hours after the extubation. Aerosolized 2.25% racemic epinephrine produces mucosal vasoconstriction and reduces laryngeal edema. The patient has a history of asthma and could have developed a bronchospastic event post-operatively. However, the clinic presentation was an inspiratory sound where bronchospasm would more likely demonstrate an expiratory wheeze. Laryngospasm is also a possibility but there is nothing in the history to put the patient at risk for a laryngospasm 30 minutes postextubation and it should not be highest on your differential.

Reference:

Altee John L. Complications in Anesthesia. WB Saunders 1999 Roizen MF, Fleisher LA. Essence of Anesthesia Practice. WB Saunders 1997

Page 42: Anaesthesia TMJ Pain Orthognathic Pathology Reconstruction

2007 Oral and Maxillofacial Surgery Self Assessment Tool (OMSSAT)

The American Board of Oral and Maxillofacial Surgery 42

40. Four days after discharge after a bimaxillary osteotomy, a 16 year-old female returns to your office complaining of malaise. You note considerable pitting edema of the lower extremities and rales on chest auscultation. Review of the anesthesia record shows approximately 1000 ml of blood loss and induced hypotension to a mean arterial pressure of 60 during the maxillary downfracture. Surgical time was 7 hours. No blood products were administered. Which of the following would you expect to find?

A. Serum osmolality 250 mOsm/L B. Serum creatinine 0.9 mg/dL C. Intense diruesis D. Hypokalemia Answer: A Rationale:

This patient's clinical presentation is consistent with acute intrinsic renal failure due to intraoperative hypoperfusion. Renal ischemia brought about by loss of red cell mass and hypotension in this prolonged procedure caused glomerular and tubular damage. The classic initial presentation of acute renal failure is oliguria and fluid retention, causing a decreased serum osmolality (normal = about 275mOsm/L.) Retention of nitrogenous waste products by lack of renal excretion would cause increases in serum creatinine (normal = 0.5-1.2 mg//dL.) In the early phase of ARF, typically an oliguric phase occurs for the first 1 to 2 weeks. If there is a renal recovery, intense dieresis usually follows this. Early ARF also generally shows metabolic acidosis and subsequent hyperkalemia. Treatment of ARF includes intense fluid and electrolyte management, avoiding nephrotoxic medications, dialysis if necessary, and prevention of infection (which is more common during the diuretic phase of ARF.)

Reference:

Washington Manual of Medical Therapeutics, 30th ed, Liippincott, Philadelphia, 2001 pp258-263 Fauci et al (eds.) Harrison's Principles of Internal Medicine 14th Ed, McGraw-Hill, New York, 1998 pp1511-1513

Page 43: Anaesthesia TMJ Pain Orthognathic Pathology Reconstruction

2007 Oral and Maxillofacial Surgery Self Assessment Tool (OMSSAT)

The American Board of Oral and Maxillofacial Surgery 43

41. A patient develops STATUS asthmaticus after an anesthetic and requires emergent endotracheal intubation and mechanical ventilation for respiratory failure. Once transferred to the ICU, it is noted that your patient has a depressed cardiac output, hypotension and hypercapnia. What is the likely explanation for this constellation of findings?

A. Secondary bacterial pneumonia B. Dynamic hyperinflation by intrinsic positive end-expiratory pressure (air trapping) C. Beta agonist toxicity D. Inadequate fluid resuscitation Answer: B Rationale:

This constellation of findings is seen with intrinsic/auto PEEP due to dynamic airway collapse (“air trapping”). The FRC is increased by recruiting more alveoli and by increasing alveolar size. Intrinsic (auto) PEEP occurs when the inspiratory/expiratory time ratio approaches 1:1. Hyperinflation of the thoracic cavity results in increased intra-thoracic pressure and subsequently a decrease in cardiac output. Ideally, the I/E ventilator ratio should be maintained at 1:3 or 1:4 for obstructive pulmonary patients.

Reference:

Ninane V, et al: Intrinsic PEEP in patients with chronic obstructive lung disease: Role of expiratory muscles. Am Rev Respir Dis 148:1037, 1993. Alex, CG and Tobin, MJ: Assessment of Pulmonary Function in Critically Ill Patients, in Textbook of Critical Care, Shoemaker, WC. 4th Edition. W B Saunders Co. 2000.

Page 44: Anaesthesia TMJ Pain Orthognathic Pathology Reconstruction

2007 Oral and Maxillofacial Surgery Self Assessment Tool (OMSSAT)

The American Board of Oral and Maxillofacial Surgery 44

42. While under general anesthesia for a maxillo-mandibular advancement procedure, an obstructive sleep apnea patient is exhibiting significant hypotension. Pre-operatively, the patient’s baseline blood pressure was 170/90. On pre-operative physical examination, the patient had jugular venous distention and a para-sternal PMI. In this circumstance what is the most appropriate management for hypotension?

A. Intravenous fluid bolus B. Trendelenberg positioning C. Intravenous alpha-1 blockade D. Intravenous vasopressor agent Answer: D Rationale:

This OSA patient has physical signs of pulmonary hypertension. The most appropriate manner to treat anesthetic induced hypotension without exacerbating right heart failure is an IV vasopressor agent (e.g. phenylephrine). Fluid bolus may exacerbate right heart failure and positioning has little effect on anesthesia induced hypotension. Alpha blockade may prevent the physiologic responses during efforts to correct the hypotension.

Reference:

Ronderos, J and Boyd, G: Anesthetic Considerations for Obstructive Sleep Apnea. Oral and Maxillofacial Clinics of North America: Vol. 7, No. 2 1995, Hess, ML and Sibbald, WS: Applied Cardiovascular Physiology in the Critically Ill. Textbook of Critical Care, 4th Edition 2000, Chapter 89: 1001

Page 45: Anaesthesia TMJ Pain Orthognathic Pathology Reconstruction

2007 Oral and Maxillofacial Surgery Self Assessment Tool (OMSSAT)

The American Board of Oral and Maxillofacial Surgery 45

43. A 68 year-old male with stable atherosclerotic heart disease develops significant hypotension, BP 75/35 with a heart rate of 110 following induction of general anesthesia. What would be the most appropriate first line pharmacotherapy?

A. Atropine B. Phenylephrine C. Ephedrine D. Epinephrine Answer: B Rationale:

Phenylephrine is an alpha agonist. It will increase SVR and may cause a reflex bradycardia, which would be beneficial for cardiac function. Ephedrine and epinephrine would both increase heart rate due to their beta stimulation and given this patients medical history that would not be desirable. Atropine is an anticholinergic drug, would increase heart rate and compromise the ischemic heart.

Reference:

Firestone, Lebowitz and Cook: Clinical Anesthesia Procedures of the Massachusetts General Hospital , Third edition, P. 634-5 Barash, Cullen and Stoelting, Handbook of Clinical Anesthesia, Fourth edition, P.871

Page 46: Anaesthesia TMJ Pain Orthognathic Pathology Reconstruction

2007 Oral and Maxillofacial Surgery Self Assessment Tool (OMSSAT)

The American Board of Oral and Maxillofacial Surgery 46

44. Initial acute management of intraoperative bronchospasm under inhalational general anesthesia includes which of the following?

A. Decadron IV

B. Reducing the depth of anesthesia C. Albuterol MDI through the endotrachial tube D. Epinephrine IV Answer: C Rationale:

Intraoperative bronchospasm should be first treated by confirming that there is no mechanical obstruction of the tracheal tube, tube placement is correct and adequate depth of anesthesia is present. The initial treatment consists of the administration of a �-agonist such as albuterol. Epinephrine should be reserved for a severe bronchospasm refractory to initial �-agonist therapy because of the potential for severe adverse effects. Steroids are not helpful in the acute management and muscle paralysis will not improve the situation. Differential diagnosis should also include pneumothorax, pulmonary edema, pulmonary embolus, and pulmonary aspiration.

Reference:

Stoelting RK, Dierdorf SF. Anesthesia and Co-Existing Disease, Churchill Livingstone, 2002. AAOMS Office Emergency Manual, AAOMS, Chicago 2004.

Page 47: Anaesthesia TMJ Pain Orthognathic Pathology Reconstruction

2007 Oral and Maxillofacial Surgery Self Assessment Tool (OMSSAT)

The American Board of Oral and Maxillofacial Surgery 47

45. The patient is a 72-year old male who lives alone and has recently started to exhibit signs of memory loss. He is otherwise healthy except for multiple medications including: levothyroxine (thyroid replacement), vytorin (hypercholesterelemia) and hydrochlorothiazide (diuretic). He is cleared for surgery and anesthesia for multiple extractions and mandibular tori removal. Intraoperatively, he develops a significant tachycardia and severe hypotension. A fluid challenge and reduction in anesthetic agents do not alter these conditions. The most likely diagnosis is:

A. fluid overload. B. malignant hyperthermia. C. thyrotoxicosis factitia. D. pheochromocytoma. Answer: C Rationale:

Thyroid storm is an abrupt exacerbation of hyperthyroidism caused by sudden release of thyroid hormone (T4) into circulation by accident or an otherwise clinical presentation. Clinical symptoms mimic malignant hyperthermia. There is tachycardia, dehydration, hyperthermia and shock. This would be high on the list of possible causes due to the sudden loss of memory, the use of multiple medications and the solitary situation.

Reference:

Stoelting, R.K. and Dierdorf, S.F: Anesthesia and Co-Existing Disease. 4th Edition, 2002. Philadelphia, Churchill Livingstone. Pp. 415-7 Beers, M. and Berkow, R.: The Merck Manual. 17th Edition, 1999. Whitehouse Station, Merck Research Laboratories. P. 89. Cole, D.J. and Schlunt, M.: Adult Perioperative Anesthesia 2004. Philadelphia, Mosby pp.71-73

Page 48: Anaesthesia TMJ Pain Orthognathic Pathology Reconstruction

2007 Oral and Maxillofacial Surgery Self Assessment Tool (OMSSAT)

The American Board of Oral and Maxillofacial Surgery 48

46. In children, blood pressure is determined almost entirely by which of one of the following physiologic parameters?

A. Stoke Volume

B. Myocardial Contractility

C. Systemic Vascular Resistance

D. Heart Rate

Answer: D Rationale:

A child's required cardiac output can be double that of an adult; it is related to an increased metabolic rate and increased oxygen consumption. Blood pressure is dependent on stroke volume, myocardial contractility, systemic vascular resistance and heart rate. In the child, stroke volume, myocardial contractility, or systemic vascular resistance are relatively invariable. Blood pressure is almost entirely rate dependent. Since children show a tendency toward vagal response to many stimuli; monitoring of the heart rate is critical. Bradycardia in children invariably leads to hypotension.

Reference:

Dembo, Jeffrey B., Pediatric Anesthesia pg.837 in Oral & Maxillofacial Surgery Clinics of North America. W.B. Saunders Co., Nov. 1992. Kelly, John P., Dembo, Jeffrey,B ., in Anesthesia Section, OMS Know ledge Update, Vol. 1 Part 1, August 1994, Pg. 42

Page 49: Anaesthesia TMJ Pain Orthognathic Pathology Reconstruction

2007 Oral and Maxillofacial Surgery Self Assessment Tool (OMSSAT)

The American Board of Oral and Maxillofacial Surgery 49

47. In the infant and young child, the narrowest portion of the larynx is located at which anatomical position?

A. Vocal cords

B. Cricoid cartilage

C. Thyroid cartilage

D. Valecula

Answer: B Rationale:

In infants or young children, the narrowest portion of the larynx is at the cricoid cartilage. In a child, an endotracheal tube might pass easily through the vocal cords but not through the subglottic region. The cricoid is the only complete ring of cartilage in the laryngotracheobronchial tree and is therefore nonexepandable. A tight fitting endotracheal tube that compresses the tracheal mucosa at this level may cause edema and result in increased airway resistance at the time of extubation. For this reason, uncuffed endotracheal tubes are usually preferred for infants or young children. As the child matures (age 10 to 12 years of age), the cricoid and thyroid cartilages have grown, eliminating the narrowing of the subglottic area and angulation of the vocal cords.

Reference:

Cote, Charles J., Pediatric Anesthesia, pg. 2376 in Miller's Anesthesia, Sixth Edition, Volume 2, by Elsevier Inc.

Page 50: Anaesthesia TMJ Pain Orthognathic Pathology Reconstruction

2007 Oral and Maxillofacial Surgery Self Assessment Tool (OMSSAT)

The American Board of Oral and Maxillofacial Surgery 50

48. While under anesthesia for an iliac crest bone graft a patient is noted to become suddenly tachycardic, hypotensive and hypoxic. The capnography reveals decreasing end-tidal CO2.The procedure is terminated. A 12-lead EKG is obtained that reveals new findings of right-axis deviation, peaked T-waves and an incomplete right bundle-branch block. The most likely diagnosis for this situation is:

A. right to left cardiac shunting. B. malignant hyperthermia. C. pulmonary embolism. D. pulmonary edema secondary to left ventricular failure. Answer: C Rationale:

Massive pulmonary embolism/ cor pulmonale are associated with right ventricular failure, arterial hypoxemia and decreasing end-tidal CO2. The V/Q mismatch results in areas of wasted pulmonary ventilation. Conversely, a right to left shunt is an extreme V/Q abnormality when there is perfusion and no ventilation at all. A decrease in CO2 on intra-operative capnography suggests decreased lung perfusion. The EKG will rule out acute AMI and show right heart strain.

Reference:

Dehring, DJ, Arens JF: Pulmonary Thromboembolism: Disease recognition and patient management. Anesthesiology 73:146, 1990 Divekan, VM et al. Pulmonary embolism during anesthesia: Case Report. Can Anesthes Soc J 28:277, 1981

Page 51: Anaesthesia TMJ Pain Orthognathic Pathology Reconstruction

2007 Oral and Maxillofacial Surgery Self Assessment Tool (OMSSAT)

The American Board of Oral and Maxillofacial Surgery 51

49. Which of the following is contraindicated in management of cardiac arrhythmias during a malignant hyperthermia episode?

A. Procainamide B. Cardizem C. Regular insulin and D50 D. Sodium bicarbonate Answer: B Rationale:

Etiologic treatment of an MH episode requires dantrolene at an initial dose of 2-3 mg/kg. Cardiac dysrrhythmias should be treated by procainamide at an initial dose of 100mg IV. Management of hyperkalemia to correct the arrhythmia can be achieved with regular insulin 10 units IV together with 1 amp of D50, as well as sodium bicarbonate to help drive potassium intracellularly. Calcium channel blockers such as cardizem are contraindicated because they can cause severe myocardial depression in the presence of dantrolene.

Reference:

MHAUS, Emergency Therapy for MH Crisis Stoelting RK, Dierdorf SF. Anesthesia and Coexisting Disease, Churchill- Livingston, 2002.

Page 52: Anaesthesia TMJ Pain Orthognathic Pathology Reconstruction

2007 Oral and Maxillofacial Surgery Self Assessment Tool (OMSSAT)

The American Board of Oral and Maxillofacial Surgery 52

50. In a patient who undergoes mandibular advancement, the ratio of soft tissue changes to hard tissue changes at pogonion would most likely be which of the following?

A. 1:1

B. 1:2

C. 2:3

D. 1:4

Answer: A Rationale:

During mandibular orthognathic surgery, the overlying soft tissues follow osseous changes closely. Generally, the soft tissues follow bony movements in a 1:1 ratio. The exception is the lower lip which relies heavily on the position of the incisor teeth and can also be affected by the position of the upper lip.

Reference:

Betts NJ, Dowd KF. Soft tissue changes associated with orthognathic surgery. Atlas Oral Maxillofac Surg Clin North Am 8: 13-38, 2000.

Page 53: Anaesthesia TMJ Pain Orthognathic Pathology Reconstruction

2007 Oral and Maxillofacial Surgery Self Assessment Tool (OMSSAT)

The American Board of Oral and Maxillofacial Surgery 53

51. In a distraction osteogenesis procedure, the consolidation phase of treatment extends until which of the following stages of bone healing?

A. Hematoma formation and inflammation B. Soft callus formation C. Hard callus formation D. Bony maturation Answer: C Rationale:

Bony healing after a fracture or osteotomy consists of four histologically distinct stages: an inflammatory phase, soft callus formation, hard callus formation, and bony maturation/remodeling. During soft callus formation, fibrovascular structures bridge the osteotomized bone segments and there is recruitment of fibroblasts and mesenchymal stem cells within the fracture zone. It is this flexible soft callus which is lengthened via gradual traction during the subsequent distraction phase. Once the desired bony lengthening has been achieved, the distraction appliance is left in place and serves as a stabilizing device while the regenerate bone (soft callus) matures and calcifies forming a hard callus. Most distraction protocols recommend a consolidation/healing phase of at least 6 to 8 weeks.

Reference:

Crago C.A., Proffit W.R., and Ruiz R.L. Maxillofacial Distraction Osteogenesis. Pp. 357-393. In: Proffit W.R., White R.P., and Sarver D.M. (Eds) Contemporary Treatment of Dentofacial Deformity. Philadelphia. Mosby. 2003.

Page 54: Anaesthesia TMJ Pain Orthognathic Pathology Reconstruction

2007 Oral and Maxillofacial Surgery Self Assessment Tool (OMSSAT)

The American Board of Oral and Maxillofacial Surgery 54

52. A patient with an underlying diagnosis of Hemifacial Microsomia presents with severe unilateral hypoplasia of the ascending ramus, condyle, and glenoid fossa. There is little to no translational movement of the affected condyle. This mandibular-TMJ deformity is consistent with which skeletal type hemifacial microsomia?

A. Type I

B. Type IIa

C. Type IIb

D. Type III

Answer: C Rationale:

Patients with Hemifacial Microsomia (oculoauriculovertebral spectrum) will present with varying degrees of mandibular and temporomandibular joint hypoplasia as a primary finding in the condition. Kaban has refined a classification system useful in defining the degree of mandibular deformity in these patients. The specific type of mandibular/TMJ deformity present is a critical factor in deciding upon the specific reconstructive techniques that will be employed. The following is a brief synopsis of each Kaban type: Type I: All skeletal components (glenoid fossa, condyle, ascending ramus) are present with a mild degree of hypoplasia. Normal function is present. Type IIa: All of the skeletal components demonstrate a moderate degree of hypoplasia. While present, the condyle may appear to be malpositioned so that it is anterior and medial to the contralateral (normal) side. Function is affected, but remains satisfactory. � Type IIb: There is moderate to severe hypoplasia of the glenoid fossa and the condyle-ramus complex. Despite an abnormal and severely hypoplastic condyle, many patients will still have a working joint and an actual “stop” where the condylar segment seats against the skull base. Most patients will demonstrate function that is limited to simple rotation of the condyle without any translational movements. � Type III: This is the most severe form of mandibular hypoplasia with complete absence of the condyle-ramus complex and the glenoid fossa. The affected side of the mandible has no working articulation against the skull base.

Reference:

Kaban LB, Moses MH, Mulliken JB. Correction of hemifacial microsomia in the growing child: A follow-up study. Cleft Palate J 23 (Suppl 1) 50, 1986. Kaban LB. Congenital Abnormalities of the Temporomandibular Joint. In: Kaban LB and Troulis MJ (Eds). Pediatric Oral and Maxillofacial Surgery. Pp. 302-339. Elsevier, Philadelphia, 2004.

Page 55: Anaesthesia TMJ Pain Orthognathic Pathology Reconstruction

2007 Oral and Maxillofacial Surgery Self Assessment Tool (OMSSAT)

The American Board of Oral and Maxillofacial Surgery 55

53. What musculature must be re-approximated as a distinct layer during soft tissue closure following a transoral genioplasty procedure?

A. Genioglossus

B. Mentalis

C. Digastric

D. Platysma

Answer: B Rationale:

When a transoral approach to the bony chin is utilized, the mentalis musculature is routinely disrupted. These are paired muscles that insert on each side of the bony midline and are separated by a small fat pad. During closure, adequate repair of the mentalis muscles is critical in order to avoid a “witches chin” deformity.

Reference:

Posnick JC. Craniofacial and Maxillofacial Surgery in Children and Young Adults. Ed. J. C. Posnick. W. B Saunders, Philadelphia.

Page 56: Anaesthesia TMJ Pain Orthognathic Pathology Reconstruction

2007 Oral and Maxillofacial Surgery Self Assessment Tool (OMSSAT)

The American Board of Oral and Maxillofacial Surgery 56

54. Which of the following is a consideration when performing sagittal split osteotomy for mandibular advancement in children compared to adults?

A. The lingula and inferior alveolar foramina are located in a more inferior and anterior

position within the ramus.

B. The sagittal bone cuts should be positioned more medial.

C. The propensity for “greenstick” fracture of the inferior border of the mandible is greater.

D. Simultaneous removal of partially developed third molar teeth is not possible.

Answer: C Rationale:

The lingula and inferior alveolar foramina are located in a more superior and posterior position in this age group. This has technical implications in determining the vertical placement of the medial osteotomy of the ramus. If the medial bone cut is positioned high on the ramus, then injury to the nerve is avoided, but the risk of unfavorable split (i.e. buccal plate fracture) increases. In children, the sagittal component of the osteotomy design should be placed as far laterally as possible in order to avoid injury to the developing teeth. Children will have a higher propensity for “greenstick” fracture along the inferior border of the mandible. Their bone is more cancellous in nature and this often results in a longer area of fracture along the inferior border of the mandible. Developing third molars can be removed after the mandibular ramus has been split. In cases where the third molar teeth are only partially developed, they can still be enucleated while the proximal and distal segments are separated

Reference:

Bell WH: Mandibular Advancement in Children Special Considerations. In Bell WH, Modern Practice in Orthognathic and Reconstructive Surgery. Vol. 3, 1992, WB Saunders. pp. 2516

Page 57: Anaesthesia TMJ Pain Orthognathic Pathology Reconstruction

2007 Oral and Maxillofacial Surgery Self Assessment Tool (OMSSAT)

The American Board of Oral and Maxillofacial Surgery 57

55. Which type of osteotomy procedure allows only a limited amount of mandibular advancement?

A. Bilateral saggittal split osteotomies

B. Inverted “L” osteotomies with bone grafts

C. Distraction osteogenesis after complete corticotomies

D. Trans-oral vertical ramus osteotomies

Answer: D Rationale:

If any significant forward movement of the mandible is undertaken using trans-oral vertical ramus osteotomies, a large gap between the proximal and distal segments will result. The clinical applications of vertical ramus osteotomy procedures are limited primarily to mandibular set-back procedures and correction of relatively minor lower facial asymmetries. All other choices are reasonable approaches to a large mandibular advancement.

Reference:

Bell WH, Proffit WR, White RP, (eds.) Surgical correction of dentofacial deformities. Philadelphia: WB Saunders; 1980.

Page 58: Anaesthesia TMJ Pain Orthognathic Pathology Reconstruction

2007 Oral and Maxillofacial Surgery Self Assessment Tool (OMSSAT)

The American Board of Oral and Maxillofacial Surgery 58

56. A 7 year-old patient with Hemifacial Microsomia demonstrates complete absence of the ipsilateral condyle and the entire ascending ramus. There is a severe deficiency of posterior facial height on the affected side and a large maxillary cant. The patient has undergone an initial phase of interceptive orthodontic treatment and is ready for surgery. Which of the following is the most predictable surgical treatment?

A. Distraction osteogenesis of the affected side of the mandible

B. Bilateral distraction osteogenesis of the mandible

C. Maxillary osteotomy for leveling of the occlusal plane

D. Costochondral graft for mandibular reconstruction

Answer: D Rationale:

The patient described in this question has a Kaban Type III mandibular deformity with the complete absence of the condyle-ramus complex and glenoid fossa. The reconstructive approach for these patients consists of an initial phase of mandibular reconstruction using a costchondral graft for the construction of missing skeletal components (e.g. condyle-ascending ramus). This is typically carried out at approximately ages 6 to 12 years. A second stage of surgery is undertaken as the patient approaches skeletal maturity and typically consists of maxillomandibular surgery in order to normalize the occlusion and position of the lower facial skeleton. Often, placement of a rib graft on the affected side is combined with a contralateral sagittal split osteotomy on the unaffected side. This allows for better three-dimensional repositioning of the distal segment of the mandible. At the same time, a posterior open bite on the affected side is created and maintained during the postoperative healing period with the use of an occlusal splint. This occlusal splint is then gradually reduced in order to facilitate dentoalveolar development of the maxilla on the affected side. Distraction osteogenesis is a technique that allows for the lengthening of a skeletal part. In the type III deformity, there is no role for distraction osteogenesis since the condyle-ramus complex is congenitally missing and these anatomic parts can not be distracted into existence.

Reference:

Posnick JC. Hemifacial Microsomia: Evaluation and Treatment. In: Posnick JC (Ed). Craniofacial and Maxillofacial Surgery in Children and Young Adults. Pp: 419-445. W.B. Saunders. Philadelphia, 2000.

Page 59: Anaesthesia TMJ Pain Orthognathic Pathology Reconstruction

2007 Oral and Maxillofacial Surgery Self Assessment Tool (OMSSAT)

The American Board of Oral and Maxillofacial Surgery 59

57. An 3 year-old undergoes bilateral mandibular osteotomies with application of internal distraction appliances. Surgery is followed by a 2 day latency phase. The distraction appliances are then activated 0.75 mm four times daily for a total of 8 days and a consolidation phase of 8 weeks follows. Upon removal of the distraction appliances, bilateral mandibular nonunions are noted. Which of the following is the most likely cause of this complication?

A. Inadequate length of time for the latency phase

B. The rate and rhythm of the distraction phase

C. Inadequate rigidity of the device during consolidation phase

D. Excessive soft tissue resistance during the advancement Answer: B Rationale:

In the above example, the most likely cause of the nonunions is the rapid rate of lengthening during the distraction phase. Most distraction protocols advocate a maximum daily rate of distraction of 1 mm total. In addition, many authors advocate a “rhythm” of distraction whereby the 1 mm total movement is divided into four incremental movements of 0.25 mm. This allows for gradual lengthening of the bony segments while avoiding excessive disruption of the bony regenerate (soft callus) and reduces soft tissue pain associated with activation of the device. In the above example, the patient's rate of distraction was approximately 3 mm per day and a large movement (24 mm) resulted in a relatively short period of 8 days. Excessive lengthening over a short period may cause stretching of the soft callus with a thinned, hour-glass shaped bony regenerate or a fibrous nonunion. Although the duration of the latency phase (2 days) described is shorter than is typical, the likelihood that it would have a negative consequence in a young child is very low. Average latency phase length is usually 5 to 7 days and allows for adequate formation of a soft callus before active distraction is initiated. In young children, bone healing occurs much faster and little or no latency phase is required.

Reference:

Crago C.A., Proffit W.R., and Ruiz R.L. Maxillofacial Distraction Osteogenesis. Pp. 357-393. In: Proffit W.R., White R.P., and Sarver D.M. (Eds), Contemporary Treatment of Dentofacial Deformity. Philadelphia. Mosby. 2003.

Page 60: Anaesthesia TMJ Pain Orthognathic Pathology Reconstruction

2007 Oral and Maxillofacial Surgery Self Assessment Tool (OMSSAT)

The American Board of Oral and Maxillofacial Surgery 60

58. A patient undergoes bilateral sagittal split osteotomies for mandibular advancement and the distal segment if placed into the final surgical splint. After elimination of bony interferences, a bony gap remains between the anterior extent of the proximal and distal segments of the mandible. If lag-screw fixation is applied in this area, which of the following will occur?

A. There will be anterior displacement of the distal segment.

B. An anterior open bite will result

C. The mandibular condyles will be displaced medially

D. The mandibular condyles will be displaced laterally Answer: D Rationale:

Changes in intercondylar width may occur as a result of sagittal split osteotomy. As the distal segment is advanced, there is often the creation of a gap at the anterior-superior extent of the osteotomy site. If the segments are compressed together at the time of rigid fixation placement, there is a fulcrum based on the distal segment and lateral displacement of the condyle. Several techniques have been described in order to avoid unfavorable condylar displacement. These include the use of bicortical position screws, placement of bone shim between the proximal and distal segments, recontouring of the proximal/lingual aspect of the distal segment, and addition of a midline/symphyseal osteotomy. Use of lag screw fixation will compress the segments and actually cause lateral condylar displacement. Alternatively, lag screws may be utilized only when at least one position screw is placed first in order to maintain the anterior osteotomy gap.

Reference:

Tucker M.R., Frost D.E., and Terry B.C. Mandibular Surgery. In: Tucker M.R., Terry B.C., White R.P., and VanSickels J.E. (Eds), Rigid Fixation for Maxillofacial Surgery. J.B. Lippincott, New York, Pp. 251-295, 1991.

Page 61: Anaesthesia TMJ Pain Orthognathic Pathology Reconstruction

2007 Oral and Maxillofacial Surgery Self Assessment Tool (OMSSAT)

The American Board of Oral and Maxillofacial Surgery 61

59. A patient with unilateral failure of hardware (loss of fixation) one week following bilateral sagittal split osteotomies of the mandible may present with which of the following physical findings?

A. Unilateral open bite

B. Coincident maxillary and mandibular midlines

C. Deviation of the mandible on extreme opening

D. Increased lower facial height Answer: A Rationale:

Loss of fixation on one side of a bilateral sagittal split osteotomy typically causes a contralateral open bite, deviation of the mandible towards the side of the hardware failure, and lower facial asymmetry. The open bite occurs typically occurs as ramus height is lost and an early occlusal contact occurs. Midline shift occurs with loss of sagittal projection after fixation loss.

Reference:

Kaban L, et al. Complications in Oral and Maxillofacial Surgery. Complications of Orthognathic Surgery. WB Saunders; Philadelphia, Pennsylvania.

Page 62: Anaesthesia TMJ Pain Orthognathic Pathology Reconstruction

2007 Oral and Maxillofacial Surgery Self Assessment Tool (OMSSAT)

The American Board of Oral and Maxillofacial Surgery 62

60. Which of the following statements about nasal changes following Le Fort I osteotomy with maxillary advancement and impaction is true?

A. Nasolabial angle will increase

B. Alar base width will decrease

C. Dorsal hump will be less noticeable

D. Nasal tip rotation will be unaffected Answer: C Rationale:

Movement of the maxilla directly affects the lower nasal anatomy and the nasal septum. When the maxilla is advanced and/or superiorly repositioned, the alar base width will increase, nasolabial angle will decrease, and increased superior rotation of the nasal tip is noted. As a result of these changes, the prominence of any dorsal hump will be reduced by the surrounding changes of the lower nasal dorsum.

Reference:

O'Ryan F, Schendel S. Nasal anatomy and maxillary surgery III. Surgical techniques for correction of nasal deformities in patients undergoing maxillary surgery. Int J Adult Ortho Orthogn Surg 4;157, 1989.

Page 63: Anaesthesia TMJ Pain Orthognathic Pathology Reconstruction

2007 Oral and Maxillofacial Surgery Self Assessment Tool (OMSSAT)

The American Board of Oral and Maxillofacial Surgery 63

61. Which of the following statements is correct regarding the use of distraction osteogenesis compared to conventional orthognathic surgical techniques for maxillary advancements of 5 millimeters or less?

A. Sagittal stability has been proven to be superior

B. Vertical stability has been proven to be superior

C. Perioperative morbidity is less

D. Long-term stability is similar Answer: D Rationale:

Distraction osteogenesis has not been proven to provide significant advantages when one looks at post-operative stability. Similar results have been seen when compared with studies done after traditional orthognathic surgery. There is no difference in orthodontic finishing requirements, and additional compensation may be required when compared with traditional techniques. During the consolidation phase, appliances (either internal or external) are kept in place for several months.

Reference:

Turvey TA, Ruiz RL, and Costello BJ. Surgical Correction of Midface Deficiency in Cleft Lip and Palate Malformation. Oral and Maxillofacial Surgery Clinics of North America: Secondary Cleft Surgery. Philadelphia: W.B. Saunders, 14: 491-507, 2002

Page 64: Anaesthesia TMJ Pain Orthognathic Pathology Reconstruction

2007 Oral and Maxillofacial Surgery Self Assessment Tool (OMSSAT)

The American Board of Oral and Maxillofacial Surgery 64

62. Which statement regarding surgically assisted rapid palatal expansion (SARPE) is correct?

A. Complete downfracture of the maxilla is required in order to realize adequate maxillary mobility. B. The procedure creates more expansion within the anterior maxilla than the posterior maxilla. C. The procedure is indicated when the amount of expansion necessary is less than 6 mm total. D. The posterior maxilla may be adequately freed without the need for osteotomy in the lateral buttress region. Answer: B Rationale:

During surgically assisted rapid palatal expansion (SARPE), there is greater widening within the anterior maxilla than there is posteriorly. By contrast, a segmental osteotomy will produce more expansion posteriorly than anteriorly. This is the result of each procedure having a distinctive “hinge” pattern during the actual maxillary widening. The SARPE procedure is reserved for cases in which greater than 8 to 10 mm of widening is necessary. If segmental osteotomy is undertaken for widening of greater than 10 mm, the likelihood of soft tissue dehiscence, periodontal defects, and poor long term stability increases. The surgical approach requires that osteotomies be created at all of the points where potential resistance to lateral expansion will be encountered. These include the palatal suture, lateral buttresses, and pterygomaxillary junction. Complete maxillary downfracture is not required.

Reference:

Bailey LJ. Segmental Le Fort I osteotomy to effect palatal expansion. J Oral Maxillofac Surg 55:728-731, 1997.

Page 65: Anaesthesia TMJ Pain Orthognathic Pathology Reconstruction

2007 Oral and Maxillofacial Surgery Self Assessment Tool (OMSSAT)

The American Board of Oral and Maxillofacial Surgery 65

63. At the time of a LeFort I osteotomy, the width of the alar base of the nose can be controlled by which of the following?

A. Augmenting the piriform rim

B. Contouring the lateral extent of the alar cartilages

C. Performing a V-Y closure of the upper lip

D. Pexy of the transverse nasalis muscle Answer: D Rationale:

Widening of the nasal base occurs anytime the maxilla is repositioned anteriorly or superiorly. In order to avoid undesirable nasal widening, a cinch-suture is passed through the transverse nasalis muscle on each side of the alar base of the nose and tied. Some surgeons prefer to pass this suture through the caudal edge of the nasal septum or anterior nasal spine.

Reference:

Schendel SA, Williamson L. Muscle reorientation following superior repositioning of the maxilla. J Oral Maxillofac Surg 41: 235-40, 1983.

Page 66: Anaesthesia TMJ Pain Orthognathic Pathology Reconstruction

2007 Oral and Maxillofacial Surgery Self Assessment Tool (OMSSAT)

The American Board of Oral and Maxillofacial Surgery 66

64. The Steiner analysis is useful in evaluating which of the following?

A. Upper and lower incisor position

B. The degree of lip incompetence

C. Lower facial height to width ratio

D. Vertical maxillary position Answer: A Rationale:

The Steiner analysis is a cephalometric analysis useful in evaluating the degree of maxillary and mandibular incisor protrusion relative to the surrounding skeletal structures. The position of the maxillary and mandibular incisors is related to Nasion-A point (N-A) and Nasion-B point (N-B) lines using both angular and linear measurements.

Reference:

Proffit W.R., Sarver D.M. Diagnosis: Gathering and Organizing the Appropriate Information. Pp. 127-170. In: Proffit W.R., White R.P., and Sarver D.M. (Eds) Contemporary Treatment of Dentofacial Deformity. Philadelphia. Mosby. 2003.

Page 67: Anaesthesia TMJ Pain Orthognathic Pathology Reconstruction

2007 Oral and Maxillofacial Surgery Self Assessment Tool (OMSSAT)

The American Board of Oral and Maxillofacial Surgery 67

65. Which of the following two-jaw orthognathic surgical procedures demonstrates the greatest long-term skeletal stability?

A. Maxillary impaction with mandibular advancement

B. Maxillary advancement with mandibular advancement

C. Maxillary downgraft with mandibular setback

D. Maxillary transverse widening with mandibular setback

Answer: A Rationale:

Long-term data on skeletal stability following orthognathic surgery suggests that some procedures are more predictable than others. Overall, surgical procedures aimed at the correction of Class II problems tend to be extremely stable. These include mandibular advancement, superior and/or anterior maxillary repositioning, and combinations of the two. Skeletal stability following mandibular setback and combined procedures for the correction of Class III situations are less reliable. Treatment plans that include the transverse widening of the maxilla are the most problematic in terms of long term stability.

Reference:

Proffit WR, Turvey TA, Phillips C. Orthognathic Surgery: a hierarchy of stability. Int J Adult Orthod Orthogn Surg 11: 191-204, 1996.

Page 68: Anaesthesia TMJ Pain Orthognathic Pathology Reconstruction

2007 Oral and Maxillofacial Surgery Self Assessment Tool (OMSSAT)

The American Board of Oral and Maxillofacial Surgery 68

66. A 48 year-old patient presents with obstructive sleep apnea and a respiratory disturbance index (RDI) of 110. Diagnostic work-up is undertaken and reveals that the patient has signs of pulmonary hypertension, intermittent ventricular ectopy, and moderate to severe congestive heart failure (cor pulmonale). Which of the following is the most appropriate initial surgical treatment?

A. Septoplasty, turbinectomy

B. Uvulopalatopharyngoplasty

C. Tracheostomy

D. Maxillo-mandibular advancement Answer: C Rationale:

Tracheostomy remains an extremely effective treatment modality in patients with obstructive sleep apnea because it completely bypasses all of the levels of obstruction within the upper airway. Although permanent tracheostomy is an unattractive alterative for patients, its use as a temporary measure can produce marked and rapid improvements in sleep architecture and oxygenation. In the above clinical situation, the primary indication for tracheostomy as an initial therapy is the patient's strikingly abnormal RDI and the presence of severe cardio-pulmonary consequences associated with OSA. The presence of life-threatening cardiac conditions requires that the upper airway be bypassed. In this example, tracheostomy may be utilized as an interim treatment until other procedures (e.g. maxillomandibular advancement) may be carried out in order to improve the patient's upper airway.

Reference:

Tiner BD, Waite PD. Surgical and Nonsurgical Management of Obstructive Sleep Apnea. In: Miloro M, Ghali GE, Larsen PE, and Waite PD (Editors), Peterson's Principles of Oral and Maxillofacial Surgery, Second Edition. Pp. 1297-1313. BC Decker, Hamilton, 2004.

Page 69: Anaesthesia TMJ Pain Orthognathic Pathology Reconstruction

2007 Oral and Maxillofacial Surgery Self Assessment Tool (OMSSAT)

The American Board of Oral and Maxillofacial Surgery 69

67. Which of the following statements about cleft lip and palate is correct?

A. Overall prevalence is approximately 1 in 4000 live births. B. Orofacial clefting rates are lowest in Native-Americans. C. Isolated cleft palate is more common than cleft lip. D. A syndromic diagnosis is more common with isolated cleft palate. Answer: D Rationale:

Cleft lip with or without cleft palate is a common congenital malformation with an incidence of approximately 1 in 700 live births, but significant variation is encountered when different ethnic/racial populations are examined. African Americans have an incidence which is significantly lower than the general population while Asians and Native-Americans have the highest rates of birth prevalence. By contrast, isolated cleft palate has a lower overall incidence of approximately 1 in 2,000 live births with similar distribution among the different racial and ethnic populations. Of note is that children with isolated cleft palate are almost five-times more likely to also have some other underlying syndromic condition.

Reference:

Costello BJ, Ruiz RL. Cleft Lip and Palate: Comprehensive Treatment Planning and Primary Repair. In: Miloro M, Ghali GE, Larsen PE, and Waite PD (Editors): Peterson's Principles of Oral and Maxillofacial Surgery, Second Edition. BC Decker. Hamilton. 2004. Pp: 839-858.

Page 70: Anaesthesia TMJ Pain Orthognathic Pathology Reconstruction

2007 Oral and Maxillofacial Surgery Self Assessment Tool (OMSSAT)

The American Board of Oral and Maxillofacial Surgery 70

68. The superiorly based pharyngeal myomucosal flap contains a portion of what muscle?

A. Superior constrictor

B. Palatopharyngeus

C. Palatoglossus

D. Tensor veli palatini Answer: A Rationale:

The pharyngeal flap is fashioned on the posterior pharyngeal wall by incising through mucosa and bluntly dissecting through superior constrictor until prevertebral fascia is identified. A superior pedicle is maintained for this finger-like, random pattern flap that is then sewn into the soft palate.

Reference:

Costello BJ, Ruiz RL. Reconstruction of Cleft Lip and Palate: Secondary Procedures. In Peterson's Principles of Oral and Maxillofacial Surgery, ed 2. Miloro M, Larsen P, Ghali GE, and Waite P eds. Hamilton: Decker 2004:871-886.

Page 71: Anaesthesia TMJ Pain Orthognathic Pathology Reconstruction

2007 Oral and Maxillofacial Surgery Self Assessment Tool (OMSSAT)

The American Board of Oral and Maxillofacial Surgery 71

69. The diagnosis of velopharyngeal insufficiency is best established by which of the following?

A. Perceptual evaluation by a speech language pathologist B. Fiberoptic nasopharyngoscopy C. Videofluoroscopy D. Nasometry Answer: A Rationale:

The determination of the presence or absence of velopharyngeal dysfunction is made during the perceptual speech evaluation performed by a speech language pathologist. During this examination, the speech pathologist relies on auditory and visual input perceived during spontaneous speech and during the production of standard phrases/words designed to test velopharyngeal function. Nasopharyngoscopy and videofluoroscopy are instruments employed to further study individuals diagnosed with velopharyngeal dysfunction. These provide information regarding the cause and the magnitude of the dysfunction. The information gained therein may be used to select subsequent therapy and tailor it according to each individual's functional anatomy. Nasometry is a physiologic assessment tool that provides indirect, quantitative measurements of nasal, relative to oral, acoustic energy transmission. Its utility is in helping quantify dysfunction and perhaps as a biorationale device for training proper velopharyngeal function.

Reference:

Marsh JL. The evaluation and management of velopharyngeal dysfunction. Clin Plast Surg 2004;31:261-269.

Page 72: Anaesthesia TMJ Pain Orthognathic Pathology Reconstruction

2007 Oral and Maxillofacial Surgery Self Assessment Tool (OMSSAT)

The American Board of Oral and Maxillofacial Surgery 72

70. What physical finding should alert the surgeon to the possibility of a submucous cleft palate?

A. Alveolar notch

B. Nasolabial fistula

C. Enlarged adenoid pad

D. Bifid uvula Answer: D Rationale:

As described by Calnan, the classic clinical findings seen when a patient has a submucous cleft palate are a triad of bifid uvula, hard palate bony notch, and separation along the median raphe of the soft palate especially during elevation of the velum. When a submucous cleft palate is present, the levator muscle is clefted and inserts abnormally into the posterior edge of the hard palate. The primary functional concern related to submucous cleft palate is the possibility that the patient will develop velopharyngeal insufficiency and hypernasal speech as with other cleft palate patients.

Reference:

Calnan, J. Submucous cleft palate. Br J Plast Surg. 1954; 6:264-82. Ruiz RL, Costello BJ. Reconstruction of Cleft Lip and Palte: Secondary Procedures. In: Miloro M, Ghali GE, Larsen PE, and Waite PD (Editors): Peterson's Principles of Oral and Maxillofacial Surgery, Second Edition. Pp: 871-886. BC Decker. Hamilton, Ontario, 2004.

Page 73: Anaesthesia TMJ Pain Orthognathic Pathology Reconstruction

2007 Oral and Maxillofacial Surgery Self Assessment Tool (OMSSAT)

The American Board of Oral and Maxillofacial Surgery 73

71. The repair of a submucous cleft palate is indicated:

A. after 3 or more episodes of otitis media during a 6 month period.

B. at the time of the diagnosis.

C. only in the presence of velopharyngeal dysfunction.

D. in the presence of obstructive sleep apnea. Answer: C Rationale:

The lone indication for repair of a submucous cleft palate is the presence of velopharyngeal dysfunction. While middle ear disease has long been associated with clefting of the secondary palate it's incidence in the submucous cleft palate population may only be slightly higher compared with t he general population. Otitis media with effusion is managed with myringotomy and tube placement It has been estimated that only 11% of children diagnosed with a submucous cleft will go on to develop velopharyngeal dysfunction. As such, repair at the time of diagnosis would represent overtreatment.

There is no direct association of submucous cleft palate and obstructive sleep apnea.

Reference:

Gosain AK, Conley SF, Marks S, Larson DL. Submucous Cleft Palate: Diagnostic Methods and Outcomes of Surgical Treatment.

Page 74: Anaesthesia TMJ Pain Orthognathic Pathology Reconstruction

2007 Oral and Maxillofacial Surgery Self Assessment Tool (OMSSAT)

The American Board of Oral and Maxillofacial Surgery 74

72. Phase I orthodontic treatment in the patient with a bilateral cleft lip and palate prior to maxillary bone graft surgery is primarily intended to:

A. collapse the cleft segments in order to obtain a class I molar relationship.

B. correct transverse maxillary dimension and align the pre-maxilla for symmetry.

C. expedite the exfoliation of remaining anterior primary teeth.

D. expand the cleft segments until a Class I molar relationship is obtained. Answer: B Rationale:

Surgical reconstruction of the child with cleft lip and palate requires familiarity with orthodontic management especially when considering sequencing of the procedures. Phase I orthodontic care of the child with bilateral cleft lip and palate should consist of maxillary transverse expansion to reduce cross-bites and align the pre-maxilla for position and symmetry. This will allow proper bone graft placement and consolidation of the premaxilla in the most appropriate position for function and aesthetics.

Reference:

Vig K, Turvey TA, Fonseca RJ: Orthodontic and Surgical Considerations in Bone Grafting in the Cleft Maxilla and Palate. In Facial Clefts and Craniosynostosis Principles and Management. Turvey et al. Saunders, Philadelphia, 1996. pg 396.

Page 75: Anaesthesia TMJ Pain Orthognathic Pathology Reconstruction

2007 Oral and Maxillofacial Surgery Self Assessment Tool (OMSSAT)

The American Board of Oral and Maxillofacial Surgery 75

73. Intravelar veloplasty during cleft palate repair is undertaken in order to reorient which aberrantly positioned muscle?

A. Salpingopharyngeus

B. Palatoglossus

C. Levator veli palatini

D. Tensor veli palatini Answer: C Rationale:

The term “intravelar veloplasty” was first used by Kriens in 1969 to describe a muscular reconstruction of the cleft palate. In particular, the levator veli palatini, which lie in an abnormal sagittal orientation and are aberrantly inserted at the posterior edge of the hard palate, are mobilized and re-oriented to lie transversely across the palate in a more normal orientation. The goal is to restore normal palatal function through the restitution of normal palatal functional anatomy. The salpingopharyngeous, palatoglossus and superior constrictor, though part of the velopharyngeal mechanism, are not directly involved in this muscular reconstruction. The tensor veli palatini muscle passes around the hamular process and is not part of the musculature that is re-directed during veloplasty.

Reference:

Sommerlad BC. A Technique for Cleft Palate Repair. Plast Reconstr Surg 2003;112:1542-1548.

Page 76: Anaesthesia TMJ Pain Orthognathic Pathology Reconstruction

2007 Oral and Maxillofacial Surgery Self Assessment Tool (OMSSAT)

The American Board of Oral and Maxillofacial Surgery 76

74. When raising a superiorly based pharyngeal flap, dissection is carried to, but not through, which anatomic structure?

A. Superior pharyngeal constrictor muscle

B. Palatopharyngeous muscle

C. Pre-vertebral fascia

D. Pre-tracheal fascial Answer: C Rationale:

The superiorly based pharyngeal flap recruits tissue from the posterior wall of the pharynx and inserts it within the nasals side of the soft palate. The procedure involves the elevation of a superiorly based myomucosal flap (includes mucosa and superior pharyngeal constrictor muscle) off of the underlying pre-vertebral fascia. During the dissection, vertical incisions are made and blunt dissection is utilized to reach a glistening, distinct fascial layer. The soft palate is then split with dissection of the oral, nasal , and muscular layers. The flap is then inset into the nasal side closure and the oral mucosa is closed over the raw surface.

Reference:

Costello BJ, Turvey TA. Velopharyngeal insufficiency in patients with cleft palate. Oral Maxillofacial Surg Clin N Am 14 (2002) 539-551.

Page 77: Anaesthesia TMJ Pain Orthognathic Pathology Reconstruction

2007 Oral and Maxillofacial Surgery Self Assessment Tool (OMSSAT)

The American Board of Oral and Maxillofacial Surgery 77

75. One-stage repair of a cleft of the secondary palate is generally undertaken at what age?

A. Prior to 3 months of age

B. 3-6 months of age

C. 10-18 months of age

D. 3-5 years of Answer: C Rationale:

Cleft palate repair is generally undertaken between the ages of 10 and 18 months, The choice of timing is a balance between optimizing facial growth and the development of normal speech patterns. Repair prior to this age range has little impact on speech outcomes but does impact negatively on midfacial growth. Repair is then chosen to coincide with the generation of early speech such that a child who is not yet speaking need not undergo palatal repair with any sense of urgency. Delay of palatal repair beyond 18 months is generally felt to be detrimental to speech outcomes.

Reference:

LaRossa, D. The State of the Art in Cleft Palate Surgery. Cleft Craniofac J 2000;37:225-228.

Page 78: Anaesthesia TMJ Pain Orthognathic Pathology Reconstruction

2007 Oral and Maxillofacial Surgery Self Assessment Tool (OMSSAT)

The American Board of Oral and Maxillofacial Surgery 78

76. A 17 year-old male with a history of a repaired left unilateral cleft lip and palate presents with a large Class III skeletal malocclusion. He is scheduled to undergo maxillary advancement via a LeFort I osteotomy. The anticipated incidence of postsurgical velopharyngeal insufficiency 6 months post surgery would be approximately:

A. 1%.

B. 15%.

C. 50%.

D. 75%. Answer: B Rationale:

Deterioration in velopharyngeal function after maxillary advancement surgery in patients with cleft palates is a common occurrence. With the passage of time, a spontaneous recovery is observed such that, at 6 months, only approximately 15% will exhibit a persistent deterioration in their velopharyngeal function.

Reference:

Janulewicz JJ, Costello BJ, Buckley MJ, et al. The Effects of LeFort I Osteotomies on Velopharyngeal and Speech Functions in Cleft Patients. J Oral Maxillofac Surg 2004;62:308-314.

Page 79: Anaesthesia TMJ Pain Orthognathic Pathology Reconstruction

2007 Oral and Maxillofacial Surgery Self Assessment Tool (OMSSAT)

The American Board of Oral and Maxillofacial Surgery 79

77. During the primary repair of a cleft palate utilizing a two-flap technique, the greater palatine neurovascular bundle is transected during elevation of the left palatal flap. What is the next step in the surgical management of this patient?

A. Abort the procedure and reposition the flap.

B. Proceed with the palatoplasty procedure.

C. Proceed but without any further elevation of the flap.

D. Attempt re-anastamosis of the vessel. Answer: B Rationale:

Two-flap palatoplasty techniques involve the elevation of full-thickness mucoperiosteal flaps on each side of the cleft defect for oral side closure. After the nasal mucosa is closed, these soft tissue flaps are sutured together in the midline for closure of the cleft defect. During the initial dissection and elevation of the flaps, the greater palatine neurovascular bundles are identified and protected. The result is that the axial soft tissue flaps are raised based upon the blood supply of the greater palatine arteries bilaterally. If the greater palatine artery is injured or cauterized, surgery may proceed without any modifications. This is because of the abundant blood supply provided by the palatal soft tissue pedicle. Injury to the greater palatine artery simply converts the palatal flap's blood supply from an “axial” pattern to a “random” pattern (i.e. not based on one specific arterial supply).

Reference:

Bardach J: Two-flap palatoplasty: Bardach's technique. Operative Techniques in Plastic and Reconstructive Surgery, Vol 2. No 4(November), 1995: pp 211-214

Page 80: Anaesthesia TMJ Pain Orthognathic Pathology Reconstruction

2007 Oral and Maxillofacial Surgery Self Assessment Tool (OMSSAT)

The American Board of Oral and Maxillofacial Surgery 80

78. Initial management of the patient who develops velopharyngeal incompetence after LeFort I advancement surgery should be which of the following?

A. Double opposing Z-plasty (Furlow) palatoplasty

B. V-Y palatoplasty pushback surgery (Veau-Wardhill-Kilner Procedure)

C. Superiorly based pharyngeal flap

D. Observation for six months to one year Answer: D Rationale:

While a deterioration in velopharyngeal function is a very common manifestation after maxillary advancement surgery in the cleft palate population, the long term incidence is only approximately 18%. Because of the demonstrated recovery potential, it is most prudent to delay any interventions until sufficient time has passed to allow for spontaneous return of function. A Furlow conversion, VY pushback and superiorly based pharyngeal flap are all surgical modalities employed in the surgical management of velopharyngeal insufficiency. Their use may be indicated in those where the velopharyngeal function fails to improve but again, only after waiting 6 months to 1 year to allow for spontaneous recovery.

Reference:

Janulewicz JJ, Costello BJ, Buckley MJ, et al. The Effects of LeFort I Osteotomies on Velopharyngeal and Speech Functions in Cleft Patients. J Oral Maxillofac Surg 2004;62:308-314.

Page 81: Anaesthesia TMJ Pain Orthognathic Pathology Reconstruction

2007 Oral and Maxillofacial Surgery Self Assessment Tool (OMSSAT)

The American Board of Oral and Maxillofacial Surgery 81

79. In a patient with cleft lip and palate, which of the following treatment modalities may cause growth restriction of the maxilla?

A. Lip adhesion procedure

B. Lip taping

C. Naso-alveolar molding devices

D. Pin retained orthopedic devices Answer: D Rationale:

All of the above listed treatment modalities are frequently utilized prior to definitive cleft lip repair in order to idealize the position of the maxillary dentoalveolar segments, improve the position of the alveolus and nasal base, and reduce the width of the cleft defect. The lip adhesion procedure involves the creation of a small “repair” of the cleft lip defect by attaching only the mucosa and lip skin without undertaking the repair of the orbicularis oris muscle. Lip adhesion is performed in order to convert a large complete cleft defect into a narrower, incomplete cleft defect. After several months, the patient is returned to the operating room and the definitive lip repair procedure (including the orbicularis muscle) is undertaken. Routine application of elastic bands and adhesive strips across the cleft defect may also help narrow the defect as well without the need for surgical intervention. Naso-alveolar molding devices are composed of a passive plate that fits over the maxillary segments and have attached wire and acrylic components for use in the molding of the nasal complex and lip taping. Unlike the other techniques listed, the use of pin-retained orthopedic devices (Latham appliance) involves more aggressive repositioning of the maxillary segments. Unfortunately, the use of these pin-retained devices with active orthopedic movement of the skeletal components is associated with maxillary growth restriction and a significant increase in the likelihood that orthognathic surgery will be necessary in the teenage years. The most popular theory to explain this association is that the aggressive skeletal movements produce disruption with hemorrhage at nasopalatine and maxillary suture regions.

Reference:

Berkowitz S. The comparison of treatment results in complete cleft lip/palate using conservative approach vs Millard-Latham PSOT procedure. Semin Orthod 2: 169, 1996.

Page 82: Anaesthesia TMJ Pain Orthognathic Pathology Reconstruction

2007 Oral and Maxillofacial Surgery Self Assessment Tool (OMSSAT)

The American Board of Oral and Maxillofacial Surgery 82

80. In a case of single-suture craniosynostosis, in what direction is cranial growth arrested?

A. Parallel to the affected suture B. Perpendicular to the affected suture C. Parallel to the open sutures D. Perpendicular to the open sutures Answer: B Rationale:

The term “craniosynostosis” refers to the premature fusion of one or more of the cranial vault sutures. The result is fusion of the bones adjacent to the suture and arrested sutural growth of the adjacent bones. The classic theory, known as Virchow's law, states that premature fusion of a cranial suture results in limited development of the skull perpendicular to the fused suture and a compensatory overgrowth at the remaining open sutures.

Reference:

Nonsyndromic craniosynostosis: diagnosis and management. Ruiz, et.al. Oral and Maxillofacial Clinics of North America: Craniofacial Update.

Page 83: Anaesthesia TMJ Pain Orthognathic Pathology Reconstruction

2007 Oral and Maxillofacial Surgery Self Assessment Tool (OMSSAT)

The American Board of Oral and Maxillofacial Surgery 83

81. Most forms of craniofacial dysostosis syndromes (e.g. Crouzon, Apert, Saethre-Chotzen) demonstrate which of the following inheritance patterns?

A. X-linked recessive

B. X-linked dominant

C. Autosomal dominant

D. Autosomal recessive Answer: C Rationale:

The craniofacial dysostosis syndromes are inherited forms of craniosynostosis. Patients are born with more extensive sutural involvement which includes premature fusion of cranial vault sutures, but also affects the sutures of the mid-facial structures. This results in a typical morphological pattern that includes a total mid-face deficiency, proptosis, and a severe Angle Class III malocclusion.

Reference:

Chapter 6, Genetic etiologies of craniosynostosis by Ethylin Wang Jabs, MD. Understanding craniofacial anomalies, the etiopathogenesis of craniosynostosis and facial clefting. Mooney and Siegel, Willey-Liss, New York, 2002.

Page 84: Anaesthesia TMJ Pain Orthognathic Pathology Reconstruction

2007 Oral and Maxillofacial Surgery Self Assessment Tool (OMSSAT)

The American Board of Oral and Maxillofacial Surgery 84

82. This CT image of a 3 month old male demonstrates which of the following conditions?

A. Metopic synostosis

B. Coronal synostosis

C. Sagittal synostosis

D. Lambdoid synostosis Answer: C Rationale:

The CT scan image shown above reveals sagittal suture craniosynostosis. The sagittal suture is absent and there is a classic scaphocephalic skeletal deformity. This is characterized by a narrow cranial vault in the bi-temporal and bi-parietal dimensions and elongation of the cranial vault in the antero-posterior dimension. This craniofacial deformity is the result of both arrested development at the site of the affected suture (e.g. sagittal) and an abnormal compensatory overgrowth at the site of the remaining open sutures (e.g. coronal and lambdoid).

Reference:

Posnick JC, Lin KY, Chen P, et al. Sagittal Synostosis: Quantitative assessment of presenting deformity and surgical results based on CT scans. Plast Reconstr Surg 92:1015, 1993.

Page 85: Anaesthesia TMJ Pain Orthognathic Pathology Reconstruction

2007 Oral and Maxillofacial Surgery Self Assessment Tool (OMSSAT)

The American Board of Oral and Maxillofacial Surgery 85

83. In diagnosing a child with a posterior cranial vault asymmetry characterized by unilateral flattening, benign positional skull molding must be differentiated from which type of craniosynostosis?

A. Lambdoid synostosis

B. Metopic synostosis

C. Sagittal synostosis

D. Bicoronal synostosis Answer: A Rationale:

When a lambdoid suture is absent, there is arrested skeletal growth across the affected occipital and parietal bones, which also results in significant flattening or posterior plagiocephaly, but that is where the similarities to positional plagiocephaly end. Nonsynostotic Positional versus Lambdoid Synostosis Plagiocephaly Occipitoparietal flattening Occipitoparietal flattening Ipsilateral frontal bossing Ipsilateral frontal flattening Forward displacement of Posterior displacement of the ipsilateral ear the ipsilateral ear No bony ridging Bony ridging along involved suture region

Reference:

From Positional plagiocephaly: evaluation and management. John Caccamese, DMD, MD, et.al. Oral and Maxillofacial Clinics of North America: Craniofacial Update.

Page 86: Anaesthesia TMJ Pain Orthognathic Pathology Reconstruction

2007 Oral and Maxillofacial Surgery Self Assessment Tool (OMSSAT)

The American Board of Oral and Maxillofacial Surgery 86

84. A 4 year-old child is seen for extraction of a several carious teeth. Her medical history reveals a repaired ventricular septal defect. On exam, you note a bifid uvula and hypernasal, poorly intelligible speech. She has normal vision and the remainder of her exam is otherwise unremarkable. Which of the following is the most likely diagnosis?

A. Van der Woude syndrome

B. Stickler syndrome

C. Nager syndrome

D. Velocardiofacial syndrome Answer: D Rationale:

Velocardiofacial syndrome was first described by Shprintzen and colleagues in 1978 as a syndrome comprising clefts of the palate either overt or submucous, congenital heart defects, typical facies and learning difficulties. The hypernasal speech and bifid uvula are suggestive of a submucous cleft palate, which is a feature of the syndrome. Van der Woude syndrome displays an autosomal dominant pattern of inheritance and consists of lower lip pits and clefts of the lip and/or palate. Stickler syndrome, also known as Hereditary Arthro-Ophthalmopathy, consists of a flat facies with a short nose, epicanthal folds and anteverted nares, a cleft palate, spondyloepiphyseal dysplasia and high grade myopia. Nager syndrome is characterized by radial limb hypoplasia, malar hypoplasia with down-slanting palpebral fissures, ear defects and occasionally a cleft palate

Reference:

Smith's Patterns of Recognizable Human Malformation, ed. 6. Jones ed. Philadelphia: Saunders 2005:976pp.

Page 87: Anaesthesia TMJ Pain Orthognathic Pathology Reconstruction

2007 Oral and Maxillofacial Surgery Self Assessment Tool (OMSSAT)

The American Board of Oral and Maxillofacial Surgery 87

85. Which cranial vault suture(s) is most commonly involved in Crouzon Syndrome?

A. The lambdoid suture(s) B. The sagittal suture C. The coronal suture(s) D. The metopic suture Answer: C Rationale:

The craniosynostosis of Crouzon, Apert, and Pfeiffer syndromes involve sites of pathogenesis either unilaterally or bilaterally along the calvarial articulations of the coronal ring. Most commonly, patients affected with one of these craniofacial dysostosis syndromes will present with bilateral involvement of the coronal sutures.

Reference:

Chapter 16, Facial dysmorphology in the craniosynostoses: clinical implications by Katherine W.L. Vig, BDS, MS, Dorth. Understanding craniofacial anomalies, the etiopathogenesis of craniosynostosis and facial clefting. Mooney and Siegel, Willey-Liss, New York, 2002.

Page 88: Anaesthesia TMJ Pain Orthognathic Pathology Reconstruction

2007 Oral and Maxillofacial Surgery Self Assessment Tool (OMSSAT)

The American Board of Oral and Maxillofacial Surgery 88

86. In which disorder is absence of the frontal sinus most commonly noted?

A. Apert Syndrome

B. Carpenter Syndrome

C. Cystic Fibrosis

D. Neurofibromatosis Answer: C Rationale:

Knowledge of how certain genetic disorders modify the craniofacial skeleton has an impact on directed surgical intervention. In cystic fibrosis, congenital absence of the frontal sinus is a noted feature of the disease. Patients with a diagnosis of cystic fibrosis are no more or less likely to sustain maxillofacial trauma than those of their age matched peers and are more susceptible to severe maxillofacial infections than that of the general population because of chemotherapeutic regimens and disorders of mucous regulation.

Reference:

Boat TF: Cystic Fibrosis. In Nelson Textbook of Pediatrics. Behrman, Kliegman, Jensen eds. Saunders, Philadelphia, 2000. pg 1315.

Page 89: Anaesthesia TMJ Pain Orthognathic Pathology Reconstruction

2007 Oral and Maxillofacial Surgery Self Assessment Tool (OMSSAT)

The American Board of Oral and Maxillofacial Surgery 89

87. The clinical term trigonocephaly is used to describe which form of craniosynostosis?

A. Metopic

B. Sagittal

C. Lambdoid

D. Coronal Answer: A Rationale:

The term trigonocephaly (e.g. triangular-head) refers to a specific morphological pattern that may be the result of metopic suture craniosynostosis. In patients with absence of the metopic suture, there is a lack of skeletal growth perpendicular to the metopic suture which runs down the center of the forehead. At the same time, there is an increased, “compensatory,” growth at the cranial vault sutures that remain open. This results in the characteristic trigonocephalic deformity which includes a triangular-shaped forehead, hypotelorism, and horizontally retrusive lateral orbital rims.

Reference:

From Chapter 2, Terminology and classification of craniosynostosis by Marilyn Jones, MD. Understanding craniofacial anomalies, the etiopathogenesis of craniosynostosis and facial clefting. Mooney and Siegel, Willey-Liss, New York, 2002.

Page 90: Anaesthesia TMJ Pain Orthognathic Pathology Reconstruction

2007 Oral and Maxillofacial Surgery Self Assessment Tool (OMSSAT)

The American Board of Oral and Maxillofacial Surgery 90

88. In a patient with Treacher Collins Syndrome, the reconstruction of zygomatic and orbital deformities should be delayed until what age?

A. 1 to 2 years

B. 3 to 4 years

C. 8 to 10 years

D. 4 to 17 years Answer: C Rationale:

Children with Treacher Collins Syndrome require multiple stages of skeletal reconstruction which include repair of the orbitozygomatic deformities and definitive orthognathic surgery. Reconstruction of the orbital hypoplasia and missing zygomatic arches is typically undertaken with the use of cranial bone grafts. Whenever possible, orbitozygomatic reconstruction should be delayed until the orbital structures themselves are at or approaching skeletal maturity. By seven years of age, the cranio-orbitozygomatic skeleton is almost completely done growing. This makes late childhood the ideal time for construction of orbital and zygomatic components. If surgical correction of the orbital-zygomatic deformity is undertaken during infancy or early childhood, then subsequent growth may adversely effect the outcome.

Reference:

Waitzmann AA, Posnick JC, Armstrong D, et al. Craniofacial skeletal measurements based on computed tomography: II. Normal values and growth trends. Cleft Palate Craniofacial Journal 29:118, 1992.

Page 91: Anaesthesia TMJ Pain Orthognathic Pathology Reconstruction

2007 Oral and Maxillofacial Surgery Self Assessment Tool (OMSSAT)

The American Board of Oral and Maxillofacial Surgery 91

89. This CT scan image is of a 6 month old infant. What is the most appropriate course of treatment for the patient?

A. Observation with no intervention

B. Reshaping of the posterior two-thirds of the cranial vault

C. Reshaping of the anterior cranial vault and fronto-orbital advancement

D. Non-surgical treatment with a custom cranial orthotic band Answer: C Rationale:

The CT Scan demonstrates that the right coronal suture is congenitally missing confirming the diagnosis of unilateral coronal craniosynostosis. Absence of growth perpendicular to this suture causes a characteristic craniofacial deformity that is characterized by orbital dystopia, shortening of the anterior cranial base on the affected side, and flattening of the forehead. The treatment for this patient is designed to release the affected suture so that normal brain growth can occur and increased intracranial pressure is avoided. At the same time, the dysmorphic skeletal components are reshaped into a more anatomically correct position and contour. Reconstruction typically requires, bifrontal craniotomy for access, fronto-orbital osteotomies with reshaping, and reshaping of the anterior cranial vault.

Reference:

Turvey TA, Gudeman SK. Nonsyndromic Craniosynostosis. In: Turvey TA, Vig KWL, and Fonseca RJ (Editors): Facial Clefts and Craniosynostosis: Principles and Management. Pp: 596-629, W.B. Saunders. Philadelphia, 1996.

Page 92: Anaesthesia TMJ Pain Orthognathic Pathology Reconstruction

2007 Oral and Maxillofacial Surgery Self Assessment Tool (OMSSAT)

The American Board of Oral and Maxillofacial Surgery 92

90. During craniofacial surgery, a venous air embolism is suspected by the surgeon. Which of the following maneuvers would be most helpful in controlling the entry of air into the venous circulation?

A. Raising the patient’s head above the level of the heart

B. Limiting or avoiding irrigation of the involved surgical field

C. Application of bone wax to cut bone edges

D. Administration of a nitrous oxide – oxygen mixture Answer: C Rationale:

The cranial vault has a large number of diploic and emissary venous channels. These channels within the bone can be opened during a variety of surgical procedures including split-thickness cranial bone harvest, skull base procedures, and any reconstructive craniofacial operation that includes a craniotomy for access. As these venous channels are opened, air may travel to the right atrium. Because 35% of the pediatric population has a patent foramen ovale, the paradoxic passage of air from the right to left heart is possible and this air may then enter the cerebral and/or coronary circulation. When air embolus is suspected during a craniofacial procedure, rapid control of the air entry portal is critical. Several technical maneuvers are useful in eliminating the entry of air into the cranial venous circulation. These include cessation of surgery, rapid lowering of the head below the level of the heart, copious irrigation of the involved field, manual compression of the venous channel, and the use of bone wax on cut bone surfaces. Nitrous oxide should be avoided and 100% oxygen administered.

Reference:

Greensmith AL, Meara JG, and Holmes AD. Complications related to cranial vault surgery. Oral Maxillofacial Surg Clin N Am 16 (2004) 465-473.

Page 93: Anaesthesia TMJ Pain Orthognathic Pathology Reconstruction

2007 Oral and Maxillofacial Surgery Self Assessment Tool (OMSSAT)

The American Board of Oral and Maxillofacial Surgery 93

91. When assessing the efficacy of myofascial trigger points injections what substance should be used as a control?

A. Xylocaine B. Sterile water C. Dry needling of the trigger point D. Normal saline Answer: D Rationale:

Normal saline is used as the control in studies of efficacy of trigger point injection solutions.

Reference:

Byrn C, Olsson J, Falkheden L, Lindh M, Hosterey U, Fogelberg MN, Linder LE, Bundertorp O. Subcutaneous sterile water injections for chronic neck and shoulder pain following whiplash injuries. Lancet, 1993;341:449-52.

Page 94: Anaesthesia TMJ Pain Orthognathic Pathology Reconstruction

2007 Oral and Maxillofacial Surgery Self Assessment Tool (OMSSAT)

The American Board of Oral and Maxillofacial Surgery 94

92. Which of the following classes of drugs may provide benefits of both analgesia and reduction of bruxism in the myofascial pain patient?

A. Monoamine oxidase inhibitors B. Tricyclic antidepressants C. Nonsteroidal anti-inflammatory medications D. Selective serotonin reuptake inhibitors Answer: B Rationale:

Tricyclic antidepressants, including specifically amitriptyline, have been suggested to be effective in alleviating pain in chronic pain syndromes and in reducing bruxism by an unknown mechanism. None of the distractors have been shown to decrease bruxism. SSRI's may increase bruxism.

Reference:

Karlis, V and Glickman, R. Nonsurgical management of temporomandibular disorders. In Miloro, M (ed). Peterson's Principles of Oral and Maxillofacial Surgery, 2nd ed. 2004 Hamilton: BC Decker p. 953.

Page 95: Anaesthesia TMJ Pain Orthognathic Pathology Reconstruction

2007 Oral and Maxillofacial Surgery Self Assessment Tool (OMSSAT)

The American Board of Oral and Maxillofacial Surgery 95

93. Comparison of outcomes one year after arthroscopy, discectomy, modified condylotomy, and disc repositioning surgery for TMJ internal derangement is most correctly summarized by which of the following statements?

A. Superior pain relief and diet improvement after arthroscopy B. Statistically indistinguishable pain relief and diet improvement C. Superior pain relief and improved diet after disc repositioning surgery D. Statistically indistinguishable range of motion Answer: B Rationale:

A prospective investigation comparing outcomes one year after arthroscopy, discectomy, condylotomy, and disc repositioning for internal derangement showed no statistically significant differences in pain reduction or diet improvement. Greater improvement in contralateral range of motion occurred with condylotomy and arthroscopy, presumably due to intracapsular scarring following discectomy and disc repositioning.

Reference:

Hall HD, Indresano TA, Kirk WS, et al. Prospective multicenter comparison of 4 temporomandibular joint operations. J Oral Maxillofac Surg 2005;63:1174-1179

Page 96: Anaesthesia TMJ Pain Orthognathic Pathology Reconstruction

2007 Oral and Maxillofacial Surgery Self Assessment Tool (OMSSAT)

The American Board of Oral and Maxillofacial Surgery 96

94. Which statement most accurately describes the TMJ disc?

A. Convex superior surface, concave inferior surface B. Composed primarily of Type-2 collagen C. Biconcave cross-sectional morphology D. Composed of hyaline cartilage Answer: C Rationale:

In cross section the disc has a biconcave shape. This shape enhances load distribution between the surfaces of the condyle and temporal bone. The disc is composed of fibrous connective tissue. The disc has little or no innervation.

Reference:

Mohamed,S:Developmental and Clinical Anatomy and Physiology of the Temporomandibular Joint. In Fonseca RJ, Bays RA, Quinn PD (eds):Oral and Maxillofacial Surgery Vol 4. 2000, WB Saunders, Philadelphia, p 8.

Page 97: Anaesthesia TMJ Pain Orthognathic Pathology Reconstruction

2007 Oral and Maxillofacial Surgery Self Assessment Tool (OMSSAT)

The American Board of Oral and Maxillofacial Surgery 97

95. Which of the following best describes the correct sequence of structures encountered superior to the zygomatic arch with a preauricular approach to the temporomandibular superior joint space?

A. Skin, superficial deep temporal fascia, temporoparietal fascia, subgaleal fascia, capsule B. Skin, subgaleal fascia, temporoparietal fascia, temporalis fascia, capsule C. Skin, temporoparietal fascia, superficial deep temporalis fascia, subgaleal fascia, capsule D. Skin, temporoparietal fascia, subgaleal fascia, temporalis fascia, capsule Answer: D Rationale:

Temporoparietal fascia is continuous superiorly with galea. Subgaleal fascia is a discrete fascial layer deep to temporoparietal fascia. The layer immediately deep to subgaleal fascia is temporalis fascia. Temporalis fascia is immediately superficial to joint capsule.

Reference:

Politi M, Toro C, Cian R, Costa, F, et al. The deep subfascial approach to the temporomandibular joint. J oral Maxillofac Surg 2003;62:1097-1102. Ellis EE, Fide MF. Surgical Approaches to the facial skeleton. Williams and Wilkins, Baltimore, 1995 p.167-911.

Page 98: Anaesthesia TMJ Pain Orthognathic Pathology Reconstruction

2007 Oral and Maxillofacial Surgery Self Assessment Tool (OMSSAT)

The American Board of Oral and Maxillofacial Surgery 98

96. The auriculotemporal nerve:

A. originates from 2 roots surrounding the anterior deep temporal artery. B. innervates the stapedius muscle. C. innervates the lateral pterygoid muscle. D. has communications with the otic ganglion. Answer: D Rationale:

The auriculotemporal nerve is one of several nerves provides sensation to the TMJ and a portion of the ear. Its origin is by two roots that surround the middle meningeal artery. The stapedius muscle is innervated by the facial nerve. The lateral pterygoid is innervated by pterygoid branches form the 3rd division of the trigeminal nerve. Auriculotemporal contributions to the otic ganglion supply afferent innervation to the parotid gland.

Reference:

Mohamed,S: Developmental and Clinical Anatomy and Physiology of the Temporomandibular Joint. In Fonseca RJ, Bays RA, Quinn PD (eds): Oral and Maxillofacial Surgery Vol 4, 2000, WB Saunders, Philadelphia, p. 14.

Page 99: Anaesthesia TMJ Pain Orthognathic Pathology Reconstruction

2007 Oral and Maxillofacial Surgery Self Assessment Tool (OMSSAT)

The American Board of Oral and Maxillofacial Surgery 99

97. What statement is true about internal derangement of the TMJ?

A. Disc morphology is more important than disc position. B. Disc mobility is more important than disc position. C. Disc position is more important than disc mobility. D. Disc morphology is more important than disc mobility. Answer: B Rationale:

Disc displacement may follow or precede alteration in the surface characteristics of the joint, although contemporary literature has discounted the importance of both disc displacement and the value of surgical disc repositioning. Impaired disc mobility is more closely related to alterations in the internal milieu of the joint and altered joint mechanics than is disc position. Disc displacement, though a marker of internal derangement, is probably not as important as altered disc mobility

Reference:

Dolwick, MF: Temporomandibular Joint Disc Displacement: A Clinical Perspective. In, Sessle, BJ; Bryant, PS; Dionne, RA (Eds): Temporomandibular Disorders and Related Pain Conditions. Progress in Pain Research and Management. Vol 4 1995 IASP Press. Seattle. pp.79 – 87. Mercuri, LG and Laskin, DM: Indications for Surgical Treatment of Internal Derangements of the TMJ. Oral Maxillofac Surg Clin N Am 1994;6:223.

Page 100: Anaesthesia TMJ Pain Orthognathic Pathology Reconstruction

2007 Oral and Maxillofacial Surgery Self Assessment Tool (OMSSAT)

The American Board of Oral and Maxillofacial Surgery 100

98. The temporal branch of the seventh cranial nerve is located in which soft tissue plane when approaching the TMJ via a preauricular approach?

A. Within the superficial temporal fat pad B. Within the subcutaneous fat C. Deep surface of the temporalis fascia D. Deep surface of the temporoparietal fascia Answer: D Rationale: The temporal branch of the facial nerve is most commonly found on the deep surface of the temporoparietal fascia and superficial to the temporalis fascia.

Reference: Ellis, E, Zide, M: Surgical Approaches to the Facial Skeleton, Baltimore, 1995, Williams and Wilkins, pp. 167-168

Page 101: Anaesthesia TMJ Pain Orthognathic Pathology Reconstruction

2007 Oral and Maxillofacial Surgery Self Assessment Tool (OMSSAT)

The American Board of Oral and Maxillofacial Surgery 101

99. In the preauricular approach to the TM joint what best describes the location of temporal branch of the facial nerve?

A. Deep to temporalis fascia, 8-35 mm anterior to the external auditory canal B. Deep to temporoparietal fascia, 8-35 mm anterior to the external auditory canal C. Deep to temporoparietal fascia, mean 3.5 cm anterior to the external auditory canal D. Deep to temporalis fascia, mean 3.5 cm anterior to the external auditory canal Answer: B Rationale:

The main trunk of facial nerve exits from the skull at the stylomastoid foramen and subsequently enters the parotid gland. The temporal branch of the facial nerve emerges from the parotid gland and crosses the zygoma deep to or within the temporoparietal fascia to innervate the frontalis muscle in the forehead. The temporal branch crosses the zygomatic arch 8-35 mm (mean 2.0cm) anterior to the most anterior portion of the external auditory canal.

Reference:

Quinn P: Color atlas of temporomandibular joint surgery. 1988 Mosby St. Louis pps. 30-31. Ellis E, Zide: Surgical approach to the facial skeleton. 1995 Lippincott Williams & Wilkins, Baltimore pps. 163-185. Al-Kayat A, Bramley P: Modified pre-auricular approach to the temporomandibular joint and malar arch. Br J Oral Surg 1980;17:91.

Page 102: Anaesthesia TMJ Pain Orthognathic Pathology Reconstruction

2007 Oral and Maxillofacial Surgery Self Assessment Tool (OMSSAT)

The American Board of Oral and Maxillofacial Surgery 102

100. MR image generation is a function of which mechanism?

A. Detection of tissue positron emission B. Detection of tissue features as a radiofrequency signal C. Detection of tissue gamma irradiation D. Detection of ionizing radiation which has traversed the tissue KEY: B Answer: B Rationale:

Choice A describes Positron emission tomography (PET). It is a nuclear medicine imaging technique which produces a three dimensional image or map of functional processes in the body. PET is a valuable technique for some diseases and disorders, because it is possible to target the radio-chemicals used for particular bodily functions. Choice C Describes Bone Scan. A bone scan is a nuclear medicine study to detect bone abnormalities. The patient is injected with a small amount of radioactive material and then scanned with a Gamma camera, a device sensitive to the radiation emitted by the injected material. Several gamma-emitting radioisotopes are used, one of which is technetium-99m. When administered to a patient, a gamma camera can be used to form an image of the radioisotope's distribution by detecting the gamma radiation emitted. Such a technique can be employed to diagnose a wide range of conditions (e.g. spread of cancer to the bones). Choice D Describes X-ray. X-rays are primarily used for diagnostic medical imaging and crystallography. X-rays are a form of ionizing radiation and as such can be dangerous. Ionizing radiation is a type of particle radiation in which an individual particle (for example, a photon, electron, or helium nucleus) carries enough energy to ionize an atom or molecule (that is, to completely remove an electron from its orbit). If the individual particles do not carry this amount of energy, it is essentially impossible for even a large flood of particles to cause ionization. These ionizations, if enough occur, can be very destructive to living tissue. By far, the most significant source of man-made radiation exposure to the general public is from medical procedures, such as diagnostic X-rays, nuclear medicine, and radiation therapy. Some of the major radionuclides used are I-131, Tc-99, Co-60, Ir-192, Cs-137. These are rarely released into the environment.

Page 103: Anaesthesia TMJ Pain Orthognathic Pathology Reconstruction

2007 Oral and Maxillofacial Surgery Self Assessment Tool (OMSSAT)

The American Board of Oral and Maxillofacial Surgery 103

Reference:

Grainger & Allison's Diagnostic Radiology: A Textbook of Medical Imaging, 4th ed., Copyright © 2001 Churchill Livingstone, Inc.

Page 104: Anaesthesia TMJ Pain Orthognathic Pathology Reconstruction

2007 Oral and Maxillofacial Surgery Self Assessment Tool (OMSSAT)

The American Board of Oral and Maxillofacial Surgery 104

101. Low signal intensity on a T-1 weighted TMJ MRI is characteristic of which substance?

A. Joint effusion B. Marrow C. Cortical bone D. Fat Answer: C Rationale:

Magnetic resonance imaging (MRI) is a noninvasive method of mapping the internal structure of the body which completely avoids the use of ionizing radiation and appears to be without hazard. It employs radiofrequency (rf) radiation in the presence of carefully controlled magnetic fields in order to produce high quality cross-sectional images of the body in any plane. It portrays the distribution of hydrogen nuclei and parameters relating to their motion in water and lipids. The exposure of the tissue to a Radiofrequency (RF) radiation causes the nuclei in the lower energy state to jump to the higher energy state. MR Imaging is based on the observation of the relaxation that takes place after the RF pulse has stopped and subsequent return of the excited nuclei from the high energy to the low. In pure water, the T2 and T1 times are approximately identical. For biological material, the T2 time is considerably shorter than the T1 time. By varying imaging it is possible to weight the signal to produce T1-, T2- or PD-weighted (proton density) images. From a medical perspective, it means that MR Imaging can provide multiple channels to observe the same anatomy. For instance in a brain image, white matter appears in a light grey in T1 and a dark grey in T2. Grey matter appears grey in both images. The Cerebro-Spinal Fluid (CSF) appears black in T1 and white in T2. The background of the image (air) appears black in both images. Fluid matters such as edema and water appear white (or whiter) on the T-2 image compared to T-1, while bone will appear darker.

Reference:

Grainger & Allison's Diagnostic Radiology: A Textbook of Medical Imaging, 4th ed., Copyright © 2001 Churchill Livingstone, Inc. .

Page 105: Anaesthesia TMJ Pain Orthognathic Pathology Reconstruction

2007 Oral and Maxillofacial Surgery Self Assessment Tool (OMSSAT)

The American Board of Oral and Maxillofacial Surgery 105

102. Which of the following structures lies in closest proximity to the medial aspect of the TMJ?

A. Carotid artery B. Middle meningeal artery C. Internal jugular vein D. 3rd division of Trigeminal nerve Answer: B Rationale:

The mean distance from the outer aspect of the zygomatic arch to the middle meningeal artery has been reported as 31 mm (range: 21-43 mm). This vessel is located slightly forward of the center (i.e., depth) of the glenoid fossa (mean: 2.4 mm). The mean distances from the outer aspect of the zygomatic arch to the carotid artery (37.5 mm; range: 29-48 mm), internal jugular vein (38.3 mm; range, 31-49 mm), and the third division of the trigeminal nerve (35 mm; 24-46 mm) were greater than that of the middle meningeal artery, but are nevertheless at risk for injury during TMJ surgery.

Reference:

Talebzadeh, N. Rosenstein, TP. Pogrel, MA. Anatomy of the structures medial to the temporomandibular joint. Oral Surg Oral Med Oral Pathol Oral Radiol Endod, 1999; 88): 674-78.

Page 106: Anaesthesia TMJ Pain Orthognathic Pathology Reconstruction

2007 Oral and Maxillofacial Surgery Self Assessment Tool (OMSSAT)

The American Board of Oral and Maxillofacial Surgery 106

103. Patients who undergo unilateral segmental mandibular resection with preservation of a short condylar segment can be expected to have what findings prior to delayed mandibular reconstruction:

A. painful disc displacement with the condyle out of the fossa. B. painless normal disc relationship with the condyle in the fossa. C. painless disc displacement with the condyle out of the fossa. D. painless normal disc relationship with the condyle out of the fossa. Answer: D Rationale:

Unilateral segmental mandibular resection does not affect the relationship between the disc and the condyle. Most condyles with a short condyle-ramus remnant will displace out of the fossa while retaining a normal relationship to the disc. Clinical symptoms of TMJ pain, TMJ noise, or muscular pain are not associated with partial mandibulectomy defects.

Reference:

Yoshiki H, Toshirou K, Kazutoshi N, Kanichi S: Magnetic resonance imaging findings and clinical symptoms in the temporomandibular joint in patients with mandibular continuity defects. J Oral and Maxillofac Surg 2000;50: 487-493.

Page 107: Anaesthesia TMJ Pain Orthognathic Pathology Reconstruction

2007 Oral and Maxillofacial Surgery Self Assessment Tool (OMSSAT)

The American Board of Oral and Maxillofacial Surgery 107

104. A TMJ MRI has been obtained on a symptomatic 19-year-old. Which clinical scenario is most compatible with this imaging study?

A. Bilateral clicking B. Recent cessation of clicking with deviation on opening C. Unilateral reciprocal clicking D. Bilateral crepitace

Page 108: Anaesthesia TMJ Pain Orthognathic Pathology Reconstruction

2007 Oral and Maxillofacial Surgery Self Assessment Tool (OMSSAT)

The American Board of Oral and Maxillofacial Surgery 108

Answer: B Rationale:

This MRI demonstrates unilateral disc displacement without reduction. Preservation of near normal disc morphology is consistent with recent progression to disc displacement without reduction. The contralateral joint disc position is normal. No clicking would be expected and, in fact, this patient stopped clicking and experienced a closed lock with deviation to the locked side 3 weeks prior to the MRI. The MRI demonstrates normal condylar morphology without evidence of arthrosis.

Reference:

Katzberg RW, et al. Magnetic resonance imaging of the temporomandibular joint meniscus. Ora Surg Oral Med Oral Pathol 1985;59:332. Anderson QN. Temporomandibular joint imaging: treatment planning. In: Fonseca RJ, Bays RA, Quinn PD (eds):Oral and Maxillofacial Surgery Vol 4. 2000, WB Saunders, Philadelphia, pp129-142.

Page 109: Anaesthesia TMJ Pain Orthognathic Pathology Reconstruction

2007 Oral and Maxillofacial Surgery Self Assessment Tool (OMSSAT)

The American Board of Oral and Maxillofacial Surgery 109

105. A 12 year-old male is noted to have his mandibular dental and symphysis midlines 3 mm to the right of his facial midline. Centric occlusion position approximates centric relation position. These findings are most consistent with:

A. myofascial pain dysfunction. B. internal derangement of the right TMJ. C. left lateral pterygoid spasm. D. internal derangement of the left TMJ. Answer: B Rationale:

Mandibular asymmetry in a growing child is often associated with an internal derangement on the short side. This association is associated with a small or deformed condyle and decreased vertical ramus height on the affected side.

Reference:

Schellhas KP, Pollei SR, Wilkes CH: Pediatric Internal Derangements of the Temporomandibular Joint: Effect on facial development. Am J Orthodont Dentofac Orthoped, 1993;104:51-59

Page 110: Anaesthesia TMJ Pain Orthognathic Pathology Reconstruction

2007 Oral and Maxillofacial Surgery Self Assessment Tool (OMSSAT)

The American Board of Oral and Maxillofacial Surgery 110

106. What is the most common configuration of TMJ disc displacement?

A. Medial B. Anteromedial C. Anterior D. Anterolateral Answer: C Rationale:

Of 58 consecutive TMJ patients imaged by 3-DIMENSIONal MRI 44/116 (38%) had anterior displacement of the TMJ disk. 23/116 (20%) had anterolateral displacement, 10/116 (9%) had medial displacement, and 10/116 (9%) had anteromedial displacement.

Reference:

Simmons HC, Gibbs SJ: Initial TMJ disk recapture with anterior repositioning appliances and relation to dental history. J Craniomandib Prac 1997;15:281-295.

Page 111: Anaesthesia TMJ Pain Orthognathic Pathology Reconstruction

2007 Oral and Maxillofacial Surgery Self Assessment Tool (OMSSAT)

The American Board of Oral and Maxillofacial Surgery 111

107. Which of the following skeletal jaw deformities is most likely associated with long-standing bilateral TMJ disc displacement without reduction?

A. High mandibular plane angle Class II malocclusion B. Asymmetric Class II malocclusion C. Low mandibular plane angle Class II malocclusion D. Class III malocclusion Answer: A Rationale:

TMJ internal derangement should be suspected in patients with mandibular deficiency, symmetrical and asymmetric Class II malocclusion, and vertical ramus deficiency with anterior open bite. Of the skeletal deformity options, high mandibular plane angle Class II patient is most likely to be associated with bilateral non-reducing disc displacements. Nickerson and Link reported that 11/11 open bite patients and 29/33 Class II patients had bilateral internal derangements.

Reference:

Link JL, Nickerson JW. Temporomandibular joint internal derangements in an orthognathic surgery population. Int J Adult Orthod Orthognath Surg 1992;7:161-9 Gidarakou IK, Tallents RH, Kyrkanides S, et al. Comparison of skeletal and dental morphology in asymptomatic volunteers and symptomatic patients with bilateral disc displacement with reduction. Angle Orthod 2002;72:541-46.

Page 112: Anaesthesia TMJ Pain Orthognathic Pathology Reconstruction

2007 Oral and Maxillofacial Surgery Self Assessment Tool (OMSSAT)

The American Board of Oral and Maxillofacial Surgery 112

108. Which of the following is the most common complaint in TMD patients?

A. Pain with opening B. Open lock C. Headache D. Tinnitus Answer: C Rationale:

Forty-eight consecutive TMD patients were asked 86 different symptoms on initial diagnostic visit. The occurrence of the symptoms was: Cephalalgia 94%, Painful to chew food 94%, Pain upon opening or closing mouth 90%, Open lock 13%.

Reference:

Simmons HC, Gibbs SJ: Anterior Repositioning Appliance Therapy for TMJ Disorders: Specific symptoms relieved and relationship to disk status on MRI. J Craniomandib Prac 2005;23:89-99

Page 113: Anaesthesia TMJ Pain Orthognathic Pathology Reconstruction

2007 Oral and Maxillofacial Surgery Self Assessment Tool (OMSSAT)

The American Board of Oral and Maxillofacial Surgery 113

109. Which of the following devices is an absolute contraindication to performing an MRI of the temporomandibular joint?

A. Cardiac pacemaker B. Titanium dental implants C. Orthodontic appliances D. Prosthetic knee joint Answer: A Rationale:

A cardiac pacemaker can malfunction when subjected to a strong magnetic field. The other choices are typically not affected by the magnetic field.

Reference:

Quinn, P: Color Atlas of Temporomandibular Joint Surgery, St. Louis, 1998, Mosby, p. 23.

Page 114: Anaesthesia TMJ Pain Orthognathic Pathology Reconstruction

2007 Oral and Maxillofacial Surgery Self Assessment Tool (OMSSAT)

The American Board of Oral and Maxillofacial Surgery 114

110. What tissue type demonstrates a bright signal on T1-weighted MR images?

A. Fat B. Muscle C. Bone D. Cartilage Answer: A Rationale:

Fat appears as a bright (white) signal on conventional T1 weighted MRI. Reference:

Cunningham, L et al. Magnetic resonance imaging of the head and neck. Atlas of the Oral and Maxillofacial Surgery Clinics of North America 2003;11:87-88.

Page 115: Anaesthesia TMJ Pain Orthognathic Pathology Reconstruction

2007 Oral and Maxillofacial Surgery Self Assessment Tool (OMSSAT)

The American Board of Oral and Maxillofacial Surgery 115

111. A 21 year-old patient with a two week history of painful limited opening (20 mm interincisal) presents with the following MRI images. What is the most likely diagnosis?

A. Acute TM joint effusion B. Disc displacement without reduction C. Disc displacement with reduction D. Lateral pterygoid fibrosis Answer: B

Page 116: Anaesthesia TMJ Pain Orthognathic Pathology Reconstruction

2007 Oral and Maxillofacial Surgery Self Assessment Tool (OMSSAT)

The American Board of Oral and Maxillofacial Surgery 116

Rationale:

The question presents a patient with acute onset of hypomobility. T1 weighted MRI images in closed and open mouth views are presented. The disc is displaced in the closed mouth view, and does not reduce in the open mouth image. Muscle fibrosis would not occur acutely. Joint effusion is best imaged with a T2 weighted MRI.

Reference:

Katzberg, RW. Temporomandibular joint imaging. Radiology 1989;170:297-307 Manzione, JV, Katzberg, RW, Tallents, RH. Magnetic resonance imaging of the temporomandibular joint. J Am Dent Assoc 1986;113:398-402.

Page 117: Anaesthesia TMJ Pain Orthognathic Pathology Reconstruction

2007 Oral and Maxillofacial Surgery Self Assessment Tool (OMSSAT)

The American Board of Oral and Maxillofacial Surgery 117

112. Which statement is true regarding TMJ magnetic resonance imaging technique? A. Sagittal images should be parallel to the long axis of the condyle B. Coronal images should be perpendicular to the long axis of the condyle C. Image should be acquired with surface coils D. Open mouth images should be acquired at an opening that does not produce an opening click Answer: C Rationale:

MRI of the TMJ image slice thickness should never be more than 3 mm. Sagittal images should be perpendicular and coronal images parallel to the condylar long axis. Open mouth images should be acquired at an opening that produces an opening click as this click may represent a reducing disc.

Reference:

Gibbs SJ, Simmons HC: A protocol for magnetic resonance imaging of the temporomandibular joint. J Craniomandib Prac 1998 16:236-241.

Page 118: Anaesthesia TMJ Pain Orthognathic Pathology Reconstruction

2007 Oral and Maxillofacial Surgery Self Assessment Tool (OMSSAT)

The American Board of Oral and Maxillofacial Surgery 118

113. A T2 weighted image of the temporomandibular joint best demonstrates which of the following joint abnormalities?

A. Superior joint space adhesions B. Perforation of the Temporomandibular Disc C. Arthrosis of the bony condyle D. Joint effusion Answer: D Rationale:

T2 weighted images best demonstrate structures with high water content. Joint effusion is best demonstrated by this modality.

Reference:

Quinn, P: Color Atlas of Temporomandibular Joint Surgery, St. Louis, 1998, Mosby, p. 22.

Page 119: Anaesthesia TMJ Pain Orthognathic Pathology Reconstruction

2007 Oral and Maxillofacial Surgery Self Assessment Tool (OMSSAT)

The American Board of Oral and Maxillofacial Surgery 119

114. What statement best describes a Wilkes Stage III TMJ internal derangement?

A. Non-reducing disk displacement-chronic B. Non-reducing disk displacement-subacute/acute C. Reducing disk displacement-chronic D. Reducing disk displacement-subacute/acute Answer: B Rationale:

In the Wilkes Classification of internal derangement stage III represents a non-reducing disk displacement that is acute or subacute. Early stage III internal derangements may be reducible with mandibular manipulation. Late stage III internal derangements represent permanent disc displacement.

Reference:

Bays, R: Surgery for Internal Derangement. In Fonseca, R (ed): Oral and Maxillofacial Surgery, Vol. 4. Philadelphia, WB Saunders, 2000, p 276.

Page 120: Anaesthesia TMJ Pain Orthognathic Pathology Reconstruction

2007 Oral and Maxillofacial Surgery Self Assessment Tool (OMSSAT)

The American Board of Oral and Maxillofacial Surgery 120

115. Which of the following is consistent with Wilkes Stage III internal derangement?

A. Painless reducing disc B. Painless nonreducing perforated disc C. Painful reducing disc D. Acute or subacute painful nonreducing disc Answer: D Rationale:

Stage III refers to disc displacement without reduction often associated with pain but prior to any hard tissue changes. A. Describes Stage I B. Describes Stage V C. Describes Stage II

Reference:

Bays RA :Surgery for Internal Derangement. In Fonseca RJ, Bays RA, Quinn PD (eds):Oral and Maxillofacial Surgery Vol 4. 2000, WB Saunders, Philadelphia ,p. 276.

Page 121: Anaesthesia TMJ Pain Orthognathic Pathology Reconstruction

2007 Oral and Maxillofacial Surgery Self Assessment Tool (OMSSAT)

The American Board of Oral and Maxillofacial Surgery 121

116. What results are expected 6 months after TMJ arthrocentesis for pain and limitation of motion from osteoarthritis?

A. Increased range of motion and decreased pain B. Unchanged range of motion and decreased pain C. Increased range of motion and unchanged pain D. Decreased range of motion and decreased pain Answer: A Rationale:

Arthrocentesis is a safe procedure that in many instances results in the osteoarthritic TMJs returning to a healthy functional state. Failure of arthrocentesis suggests that the painful limitation is most probably caused by changes such as fibrous adhesions or osteophytes that require surgical intervention for their removal. Of 38 TMJs treated with arthrocentesis, 26 joints responded favorably; pain and dysfunction scores were reduced from 9.86 ± 0.73 to 3.39 ± 0.76 and from 11.34 ± 0.66 to 3.4 ± 0.69, respectively (P < .001). Maximal mouth opening increased from 24.40 ± 2.70 mm to 43.20 ± 3.10 mm (P < .001). Lateral and protrusive jaw movements also increased.

Reference:

Nitzan D, Price: Arthrocentesis for osteoarthritis TMJ. J Oral Maxillofac Surg 2001; 59:1154.

Page 122: Anaesthesia TMJ Pain Orthognathic Pathology Reconstruction

2007 Oral and Maxillofacial Surgery Self Assessment Tool (OMSSAT)

The American Board of Oral and Maxillofacial Surgery 122

117. Regarding TMJ discectomy and dermal graft placement, which of the following is most correct?

A. With joint function, dermis assumes a biconcave shape. B. Dermis grafting prevents condylar erosion after discectomy. C. Dermis grafting has not been demonstrated superior to discectomy alone. D. Dermis grafting limits intraarticular adhesion formation. Answer: C Rationale:

There is little evidence that dermal graft placement produces results superior to discectomy, alone. Dermis does not assume a biconcave shape and does not prevent condylar erosion after discectomy. Dermal grafting does no prevent intraarticular adhesion formation.

Reference:

Bays RA :Surgery for Internal Derangement. In Fonseca RJ, Bays RA, Quinn PD (eds):Oral and Maxillofacial Surgery Temporomandibular Disorders. Philadelphia, WB Saunders, 2000 , Volume 4, p 292.

Page 123: Anaesthesia TMJ Pain Orthognathic Pathology Reconstruction

2007 Oral and Maxillofacial Surgery Self Assessment Tool (OMSSAT)

The American Board of Oral and Maxillofacial Surgery 123

118. The expected outcome after unilateral temporomandibular joint discectomy without disc replacement is:

A. open bite. B. loss of range of motion. C. subcondylar cyst formation. D. formation of a pseudo-disc. Answer: D Rationale:

MR imaging after discectomy without disc replacement has demonstrated formation of new tissue between the condyle and fossa. This tissue has been referred to as pseudo-disc because of its location. Loss of mandibular height, open bite, crepitation, loss of range of motion, though described in isolated cases, are not the usual outcome of discectomy. Accelerated condylar arthrosis has also been observed but with avoidance of early joint loading is not expected. Subcondylar cyst formation is a manifestation of osteoarthritis and not associated with discectomy.

Reference:

Susumu T, Tsuguo S, Masashi Y: Long-term magnetic resonance imaging after temporomandibular joint discectomy without replacement. J Oral Maxillofac Surg 2000;58:739-745.

Page 124: Anaesthesia TMJ Pain Orthognathic Pathology Reconstruction

2007 Oral and Maxillofacial Surgery Self Assessment Tool (OMSSAT)

The American Board of Oral and Maxillofacial Surgery 124

119. To protect the facial nerve in the preauricular approach to the temporomandibular joint, the incision through the fascia and periosteum in the area of the zygomatic arch should not be more than ________ cm from the anterior border of the external auditory canal.

A. .8 cm B. .5cm C. 1.5cm D. 2.2cm Answer: A Rationale:

The facial nerve crosses the zygomatic arch .8cm to 3.5cm anterior to the anterior border of the external auditory canal. Therefore, the incision through temporoparietal fascia and periosteum should not be more than be .8cm anterior to the anterior border of the external auditory canal.

Reference:

Ellis, E, Zide, M: Surgical Approaches to the Facial Skeleton, Baltimore, 1995, Williams and Wilkins, p. 165.

Page 125: Anaesthesia TMJ Pain Orthognathic Pathology Reconstruction

2007 Oral and Maxillofacial Surgery Self Assessment Tool (OMSSAT)

The American Board of Oral and Maxillofacial Surgery 125

120. Condylotomy for symptomatic internal derangement has the greatest positive clinical effect with regard to improvement in:

A. pre-existing malocclusion. B. position of a non-reducing disc. C. symptoms associated with Wilkes stage II and III internal derangement. D. symptoms associated with Wilkes stage IV and V internal derangement. Answer: C Rationale:

Pain and diet are significantly improved 3 years after modified condylotomy for internal derangement. Disc position is improved at least 70% of the time with disc displacement with reduction. Non-reducing disc displacement is unaffected by condylotomy. Late stage internal derangement joints with degenerative joint disease (Wilkes Stage IV, V) have less satisfactory pain relief after modified condylotomy. Although modified condylotomy can be applied simultaneously to the patient with Class III malocclusion and symptomatic internal derangement this is not well documented and a represents a more novel application of condylotomy.

Reference:

Hall HD, Navarro ZE, Gibbs SJ: Prospective study of modified condylotomy for treatment of nonreducing disc displacement. Oral Surg Oral Med Oral Pathol Oral Radiol Endod 2000;89:147-158. Hall HD, Navarro ZE, Gibbs SJ: One- and three-year prospective outcome study of modified condylotomy for treatment of reducing disc displacement. J Oral Maxillofac Surg 2000;58:7-17.

Page 126: Anaesthesia TMJ Pain Orthognathic Pathology Reconstruction

2007 Oral and Maxillofacial Surgery Self Assessment Tool (OMSSAT)

The American Board of Oral and Maxillofacial Surgery 126

121. Compared to no disc replacement, the main advantage of placing a dermis graft in conjunction with TMJ discectomy is:

A. enhanced pain relief. B. improved range of motion. C. decreased joint noise. D. decreased regressive condylar remodeling. Answer: C Rationale:

Interpositional dermis can decrease joint noise after discectomy. No clinical evidence of joint sounds such as crepitus was found in 33 of 35 operated joints evaluated an average of 2 years after discectomy with placement of a dermis interpositional graft. Dermis grafting has no effect on pain response, range of motion or the incidence of regressive condylar remodeling following discectomy.

Reference:

Dimitrioulis G: The use of dermis grafts after discectomy for internal derangement of the temporomandibular joint JOMS. J. Oral Maxillofac Surg 2005;63:75

Page 127: Anaesthesia TMJ Pain Orthognathic Pathology Reconstruction

2007 Oral and Maxillofacial Surgery Self Assessment Tool (OMSSAT)

The American Board of Oral and Maxillofacial Surgery 127

122. Which branch of cranial nerve VII is at greatest risk with the pre-auricular approach to the temporomandibular joint?

A. Buccal B. Zygomatic C. Cervical D. Temporal Answer: D Rationale:

The most commonly injured branch of the facial nerve is the temporal branch with preauricular approach to the TMJ. Less commonly, the zygomatic branch may be injured.

Reference:

Vallerand WP, Dolwick MF. Complication of temporomandibular joint surgery. Oral Maxillofac Clin North Am 1990;2:481-8.

Page 128: Anaesthesia TMJ Pain Orthognathic Pathology Reconstruction

2007 Oral and Maxillofacial Surgery Self Assessment Tool (OMSSAT)

The American Board of Oral and Maxillofacial Surgery 128

123. What statement describes a functional deficit associated with injury to the frontal branch of the facial nerve?

A. Lagophthalmos B. Exposure keratitis C. Brow ptosis D. Hypertropia Answer: C Rationale:

Injury to the frontal branch of the facial nerve causes frontalis weakness and brow ptosis. Lagophthalmos and exposure keratitis are associated with injury to the zygomatic branch of the facial nerve. Hypertropia refers to globe elevation from impaired globe depressor function, as may be seen in injury to the inferior division of the oculomotor nerve.

Reference:

Quinn P: Color atlas of temporomandibular joint surgery; 1998 Mosby St. Louis p.31

Page 129: Anaesthesia TMJ Pain Orthognathic Pathology Reconstruction

2007 Oral and Maxillofacial Surgery Self Assessment Tool (OMSSAT)

The American Board of Oral and Maxillofacial Surgery 129

124. During intra-articular TMJ surgery, preservation of the medial capsule, may avoid damage to which vascular structure? A. The internal maxillary artery B. The masseteric artery C. The deep temporal artery D. The middle meningeal artery Answer: D Rationale:

The middle meningeal artery is immediately medial to the temporomandibular joint. Preservation of the medial joint capsule will minimize injury to this vascular structure. Options a,b,c, are incorrect because none of these vessels course immediately adjacent to the medial portion of the TMJ.

Reference:

Mohamed,S:Developmental and Clinical Anatomy and Physiology of the Temporomandibular Joint. In Fonseca RJ, Bays RA, Quinn PD (eds):Oral and Maxillofacial Surgery Vol 4, 2000 WB Saunders, Philadelphia, p 4.

Page 130: Anaesthesia TMJ Pain Orthognathic Pathology Reconstruction

2007 Oral and Maxillofacial Surgery Self Assessment Tool (OMSSAT)

The American Board of Oral and Maxillofacial Surgery 130

125. Compared to discectomy, a more common complication of modified mandibular condylotomy is?

A. Change in occlusion B. Post operative joint noise C. Intraarticular adhesions D. Condylar arthrosis Answer: A Rationale:

Condylar “sag” and loss of ramus height can lead to a posterior occlusal prematurity on the side of a modified condylotomy. Occlusion changes following discectomy, if any, are mild and transient. Joint noise is more common after discectomy because of intraarticular scarring/adhesions.

Reference:

Bays RA :Surgery for Internal Derangement. In Fonseca RJ, Bays RA, Quinn PD (eds): Oral and Maxillofacial Surgery Vol 4, 2000, WB Saunders, Philadelphia, p. 297.

Page 131: Anaesthesia TMJ Pain Orthognathic Pathology Reconstruction

2007 Oral and Maxillofacial Surgery Self Assessment Tool (OMSSAT)

The American Board of Oral and Maxillofacial Surgery 131

126. During TMJ arthrotomy, what structure is most likely to be the source of bleeding during entry into the superior joint space?

A. Superior head of the lateral pterygoid muscle B. Inferior head of the lateral pterygoid muscle C. Retrodiscal tissue D. Posterior temporalis muscle Answer: C Rationale:

The retrodiscal tissue has a robust blood supply and can be encountered as the lateral capsule is incised and the joint space entered. a. and b. are incorrect because they are encountered at the very anterior and medial region of the joint. d. is incorrect because it is usually encountered before the joint capsule has been opened.

Reference:

Mohamed,S:Developmental and Clinical Anatomy and Physiology of the Temporomandibular Joint. In Fonseca RJ, Bays RA, Quinn PD (eds):Oral and Maxillofacial Surgery Vol 4 2000 WB Saunders, Philadelphia, p 6.

Page 132: Anaesthesia TMJ Pain Orthognathic Pathology Reconstruction

2007 Oral and Maxillofacial Surgery Self Assessment Tool (OMSSAT)

The American Board of Oral and Maxillofacial Surgery 132

127. During modified mandibular condylotomy surgery what is the maximum distance that the oscillating blade must extend medially beyond ramus at the level of the midsigmoid notch to place the internal maxillary artery at least risk of injury?

A. 20 mm

B. 15 mm

C. 10 mm

D. <5 mm

Answer: D Rationale:

The mean distance from the internal maxillary artery to the midsigmoid ramus is only 3.3 mm. In addition to the risk of injury to the internal maxillary artery, a branch of the internal maxillary artery, the masseteric artery, passes through the sigmoid notch to supply the masseter muscle. Both arteries are at risk during modified condylotomy.

Reference:

Fujimura K, Segami N, Kobayashi S. Anatomical study of the complications of intraoral vertico-sagittal ramus osteotomy. J Oral Maxillofac Surg 64:384-389, 2006

Page 133: Anaesthesia TMJ Pain Orthognathic Pathology Reconstruction

2007 Oral and Maxillofacial Surgery Self Assessment Tool (OMSSAT)

The American Board of Oral and Maxillofacial Surgery 133

128. During intra-articular TMJ surgery, preservation of the medial capsule, may avoid damage to which vascular structure?

A. The internal maxillary artery B. The masseteric artery C. The deep temporal artery D. The middle meningeal artery Answer: D Rationale:

The middle meningeal artery is immediately medial to the temporomandibular joint. Preservation of the medial joint capsule will minimize injury to this vascular structure. Options a,b,c, are incorrect because none of these vessels course immediately adjacent to the medial portion of the TMJ.

Reference:

Mohamed,S:Developmental and Clinical Anatomy and Physiology of the Temporomandibular Joint. In Fonseca RJ, Bays RA, Quinn PD (eds):Oral and Maxillofacial Surgery Vol 4, 2000 WB Saunders, Philadelphia, p 4.

Page 134: Anaesthesia TMJ Pain Orthognathic Pathology Reconstruction

2007 Oral and Maxillofacial Surgery Self Assessment Tool (OMSSAT)

The American Board of Oral and Maxillofacial Surgery 134

129. The surface of the mandibular condyle is covered with what material?

A. Hyaline cartilage B. Synovium C. Fibrous connective tissue D. Fibrocartilage Answer: D Rationale:

The TMJ articular surface of the condyle covered with fibrocartilage. Fibrocartilage is best suited to loading of the articular surface, while also allowing remodeling. A, B, and C do not form the covering of the condyle.

Reference:

Mohamed, S: Developmental and Clinical Anatomy and Physiology of the Temporomandibular Joint. In Fonseca RJ, Bays RA, Quinn PD (eds): Oral and Maxillofacial Surgery. Vol 4, 2000, WB Saunders, Philadelphia, p 8.

Page 135: Anaesthesia TMJ Pain Orthognathic Pathology Reconstruction

2007 Oral and Maxillofacial Surgery Self Assessment Tool (OMSSAT)

The American Board of Oral and Maxillofacial Surgery 135

130. Marked degenerative TMJ disease has been associated with which condition?

A. Multiple sclerosis B. Hemophilia C. Cystic fibrosis D. Grave’s disease Answer: B Rationale:

Hemarthrosis is a common consequence of moderate to severe coagulopathies. Recurrent bleeding into the temporomandibular joint may occur in afflicted individuals resulting in marked degradation of articular surfaces of the mandibular condyle and temporal bone, and fibrosis. The mechanism responsible for these degenerative changes involves the generation of free radicals from Fe++ released from degraded hemoglobin. Some patients with multiple sclerosis may complain of radiating, electric shock-like pain limited to trigeminal dermatomes. Though the clinical presentation is characteristic of trigeminal neuralgia, it is conceivable that this pain could be mistaken as TMJ arthralgia, particularly if the pain was felt in the dermatome of CN V3. However, multiple sclerosis is not associated with an increased incidence of degenerative TMJ disease. Likewise, there is no evidence that the prevalence of degenerative TMJ disease is higher in patients with either Grave's disease or cystic fibrosis.

Reference:

Kaneda T. Nagayama M. Ohmori M. Minato F. Nakajima J. Shikimori M. Hemarthrosis of the temporomandibular joint in a patient with hemophilia B: report of case. J Oral Surg 1979;37:513-4. Nishioka GJ. Van Sickels JE. Tilson HB. Hemophilic arthropathy of the temporomandibular joint: review of the literature, a case report, and discussion. Oral Surg, Oral Med, Oral Pathol 1988; 65:145-50.

Page 136: Anaesthesia TMJ Pain Orthognathic Pathology Reconstruction

2007 Oral and Maxillofacial Surgery Self Assessment Tool (OMSSAT)

The American Board of Oral and Maxillofacial Surgery 136

131. Which of the following molecules has been implicated in the pathogenesis of degenerative TMJ diseases and can be inhibited by tetracyclines?

A. Tumor necrosis factor alpha (TNF) B. Interleukin 1 beta (IL-1) C. Matrix metalloproteinase 1 (MMP1) D. Cathepsin D Answer: C Rationale:

Matrix metalloproteinases (MMPs) are enzymes which degrade molecules in the extracellular matrices (e.g., collagens, proteoglycans) of articular tissues of the TMJ. To date, four MMPs have been isolated from diseased human TMJs (MMP1, MMP2, MMP3, MMP9). These matrix degrading enzymes require zinc as a co-factor for activity. Tetracyclines inhibit these enzymes by chelation of zinc. Recent animal studies and limited clinical trials indicate that tetracyclines may limit progression of some degenerative TMJ diseases by inhibition of these matrix degrading enzymes. TNF and IL-1 are potent cytokines that have also been isolated from symptomatic human TMJs. These signaling molecules are believed to induce the synthesis of MMPs and other molecules that are involved in tissue degradation and inflammation. However, the activities of TNF and IL-1are not affected by tetracyclines. Cathepsin D is an intracellular endopeptidase that is involved in the intracellular degradation of molecules. Cathepsin D has been identified in synoviocytes of the TMJ. The activity of this enzyme is not affected by tetracyclines.

Reference:

Milam SB. Pathogenesis of degenerative temporomandibular joint arthritides. Odontology 2005;93):7-15. Zardeneta G. Milam SB. Lee T. Schmitz JP. Detection and preliminary characterization of matrix metalloproteinase activity in temporomandibular joint lavage fluid. International Journal of Oral & Maxillofacial Surgery. 1998;27:397-403. Haskin CL. Milam SB. Cameron IL. Pathogenesis of degenerative joint disease in the human temporomandibular joint. Critical Reviews in Oral Biology & Medicine. 1995;6:248-77. Kiyoshima T. Tsukuba T. Kido MA. Tashiro H. Yamamoto K. Tanaka T. Immunohistochemical localization of cathepsins B and D in the synovial lining cells of the normal rat temporomandibular joint. Archives of Oral Biology. 1993;38:357-9.

Page 137: Anaesthesia TMJ Pain Orthognathic Pathology Reconstruction

2007 Oral and Maxillofacial Surgery Self Assessment Tool (OMSSAT)

The American Board of Oral and Maxillofacial Surgery 137

132. A 46 year-old female presents with bilateral preauricular pain, restricted jaw movement and slight apertognathia. CT and MR images reveal marked erosions of the articular surfaces of both TMJs with mild joint effusions. Distal interphalangeal joint swelling and pitting fingernails (i.e., nail dystrophy) are observed on general examination. The patient has a negative rheumatoid factor, but is HLA-Bw38 and HLA-DR4 positive, anemic, and has a mildly elevated erythrocyte sedimentation rate. What is most likely diagnosis is?

A. Psoriatic arthritis B. Gout C. Anklyosing spondylitis D. Reiter syndrome Answer: A Rationale:

Psoriatic arthritis is an erosive polyarthritic disease occurring in patients with psoriasis (psoriatic skin lesions). Psoriasis is a genetically determined disease, and HLA-Bw38, HLA-DR4, and HLA-DR7 are genetic markers for psoriatic arthritis. The disease is slightly more prevalent in women. Several forms of the arthropathy appear to exist. Clinical findings suggestive of psoriatic arthritis include skin lesions, distal interphalangeal joint swelling, and pitting fingernails (i.e., nail dystrophy). This condition often results in an elevated ESR and anemia. Crystal deposition diseases are believed to be genetic or acquired metabolic disorders that result in the formation of crystals derived from uric acid (gout) or calcium pyrophosphate dihydrate (pseudogout) in tissues of affected joints. Gout is typically a slowly progressive monoarthritide (in initial phases) with restricted mandibular range of motion. Gout predominantly affects older men (peak incidence in 5th decade of life). Focal crystal deposits, termed tophi, are classically found in the helix or antihelix of the ear in chronic cases. These deposits normally appear as small papular masses that rarely ulcerate or become infected. Leukocytosis, elevated ESR, and elevated uric acid are commonly observed on serology. Apertognathia is also rare. CBC, ESR, uric acid and anti-nuclear antibody studies are typically within normal limits. Reiter syndrome is an asymmetrical inflammatory oligoarthritic condition consisting of painless mucocutaneous lesions, conjunctivitis, urethritis, and arthritis affecting predominantly young men (9:1-99:1 male to female ratio) who have recently experienced an infection (Chlamydia, Campylobactor, Shigella, Salmonella, HIV). Onset is typically 1-4 weeks after infection.

Reference:

Page 138: Anaesthesia TMJ Pain Orthognathic Pathology Reconstruction

2007 Oral and Maxillofacial Surgery Self Assessment Tool (OMSSAT)

The American Board of Oral and Maxillofacial Surgery 138

Espinoza LR. Cuellar ML. Silveira LH. Psoriatic arthritis. Current Opinion in Rheumatology.1992;4:470-8. Kononen M. Wolf J. Kilpinen E. Melartin E. Radiographic signs in the temporomandibular and hand joints in patients with psoriatic arthritis. Acta Odontologica Scandinavica. 1991;49):191-6. Barthelemy I. Karanas Y. Sannajust JP. Emering C. Mondie JM. Gout of the temporomandibular joint: pitfalls in diagnosis. Journal of Cranio-Maxillo-Facial Surgery. 2001;2):307-10. Bomalaski JS. Jimenez SA. Erosive arthritis of the temporomandibular joint in Reiter's syndrome. Journal of Rheumatology. 1984;11:400-2.

Page 139: Anaesthesia TMJ Pain Orthognathic Pathology Reconstruction

2007 Oral and Maxillofacial Surgery Self Assessment Tool (OMSSAT)

The American Board of Oral and Maxillofacial Surgery 139

133. This 29 year-old male presented with slowly progressing malocclusion and chin deviation. What is the most likely diagnosis based on CT images?

A. Osteochondroma B. Giant cell lesion C. Osteosarcoma D. Condylar hyperplasia Answer: A Rationale:

This represents a slow growing well-circumscribed benign lesion. Osteosarcoma would be poorly circumscribed and the margins of the native condyle would be indistinct. Giant cell lesions typically would present as a destructive radiolucency rather than a proliferative, radiopaque process. In condylar hyperplasia the native condyle is usually dramatically enlarged in all dimensions.

Reference:

Stanton, D Stewart, J: Tumors of the Temporomandibular Joint. In Fonseca, R (ed): Oral and Maxillofacial Surgery, Vol. 4. Philadelphia, WB Saunders, 2000, pp 369-373. Waldron, C: Bone Pathology. In Neville, C (ed): Oral and Maxillofacial Pathology. Philadelphia, WB Saunders, 1995, p 472.

Page 140: Anaesthesia TMJ Pain Orthognathic Pathology Reconstruction

2007 Oral and Maxillofacial Surgery Self Assessment Tool (OMSSAT)

The American Board of Oral and Maxillofacial Surgery 140

134. Select from the list the appropriate laboratory evaluations for a 12 year old girl with 8 weeks of intermittent daily fever, macular rash and swelling/pain involving the wrists, knees, ankles and TM joints:

ANA (antinuclear antibodies) CRP (C-Reactive protein) CBC (complete blood count) C-ANCA(cytoplasmic antineutrophil cytoplasmic antibody) RF(rheumatoid factor) ESR(erythrocyte sedimentation rate) LFTs(liver function tests) A. CBC, LFTs, CRP, ANA, RF, ESR B. LFTs, CRP, ANA, RF, ESR C. C-ANCA, CRP, ANA, RF, ESR D. C-ANCA, LFTs, CRP, ANA, RF, ESR Answer: A Rationale:

The clinical scenario suggests systemic juvenile rheumatoid arthritis (Still's disease). Systemic JRA is marked by fever, rash, hepato-splenomegally, anemia, leukocytosis, and any number of joints may be affected, including the TMJ. Laboratory studies should include complete blood count, liver function tests (e.g AST, ALT), erythrocyte sedimentation rate and CRP. Minor AST and ALT enzyme elevations are often present. The ESR is typically elevated in children with systemic onset JRA and may exceed 100 mm/hr; normal sedimentation rates are rare when the disease is active. The level of C-reactive protein, another acute phase reactant, is usually elevated as well. Screening for rheumatic diseases should include rheumatoid factor and antinuclear antibodies. Antinuclear antibodies and RF are rarely seen in systemic onset JRA. Their presence suggests alternative diagnoses. RF, for example, may be associated with Sjögren's syndrome or the early onset of adult type RA in teenagers. Both RF and ANA may be present in children with mixed connective tissue disease. However, these conditions rarely mimic systemic onset JRA. Anti-neutrophil cytoplasmic antibodies with a cytoplasmic staining pattern (c-ANCA) have been found to have a high degree of sensitivity and specificity for Wegener's granulomatosis and is not an appropriate test to order in this child.

Page 141: Anaesthesia TMJ Pain Orthognathic Pathology Reconstruction

2007 Oral and Maxillofacial Surgery Self Assessment Tool (OMSSAT)

The American Board of Oral and Maxillofacial Surgery 141

Reference: Rimon A, Zeharia A, Mimouni M, Mukamel M: Temporomandibular joint pain in pediatrics: the clinical approach and differential diagnosis; Pediatric Rheumatology Online, Volume 3, Number 3 2005. (http://www.pedrheumonlinejournal.org/may-june05/TMJ-Pain.htm) Cassidy, JT, Petty, RE. Juvenile rheumatoid arthritis. In: Cassidy, JT, Petty, RE (Eds) Textbook of Pediatric Rheumatology, 4th ed, 2001 W.B. Saunders Company, Philadelphia, p. 218.

Page 142: Anaesthesia TMJ Pain Orthognathic Pathology Reconstruction

2007 Oral and Maxillofacial Surgery Self Assessment Tool (OMSSAT)

The American Board of Oral and Maxillofacial Surgery 142

135. A 20 year-old female presented with a previous history of TMJ clicking, isolated left TM joint pain, and limited opening. Physical examination now reveals left pre-auricular tenderness and crepitus. You obtain the following radiograph. The most likely diagnosis is:

A. psoriatic arthritis.

B. avascular necrosis.

C. osteoarthrosis.

D. rheumatoid arthritis.

Answer: C Rationale:

The history of past TMJ clicking suggests a long-standing internal derangement. The radiographic changes are that of arthrosis, consistent with a previously clicking joint. Psoriatic and rheumatoid arthritis are both systemic arthropathies that involve multiple joints. Avascular necrosis may possibly be diagnosed my MR imaging but has no characteristic panoramic radiographic appearance.

Reference:

Silverstein K. Arthritis of the temporomandibular joint. In Fonseca, R (ed): Oral and Maxillofacial Surgery Vol. 4. Philadelphia, WB Saunders, 2000 p. 73-92.

Page 143: Anaesthesia TMJ Pain Orthognathic Pathology Reconstruction

2007 Oral and Maxillofacial Surgery Self Assessment Tool (OMSSAT)

The American Board of Oral and Maxillofacial Surgery 143

136. A healthy 65 year-old male presents with a chief complaint of generalized intermittent headache of 40 years duration. Over the past year his left TMJ is making a grinding noise when he eats. He denies any TMJ pain or locking. What is your clinical diagnosis from this patient’s history and image?

A. Avascular necrosis of the mandibular condyle B. Degenerative joint disease C. Synovial chondromatosis D. Rheumatoid arthritis Answer: B Rationale:

a. The concept of “avascular necrosis” of the TMJ condyle is unproven. b. This is a case of a patient with long standing “headache” unrelated to TMJ since he has no TMJ pain or dysfunction. He does however exhibit primary, age related DJD of the TMJ which if misdiagnosed can lead to unnecessary surgical intervention. c. The patient has no TMJ pain and dysfunction which would be hallmarks of chondromatosis along with joint noise. d. The patient has no history of systemic arthritis, TMJ pain or dysfunction. Involvement of the TMJ as the initial joint in systemic rheumatoid arthritis is rare. Unilateral TMJ rheumatoid is also rare.

Page 144: Anaesthesia TMJ Pain Orthognathic Pathology Reconstruction

2007 Oral and Maxillofacial Surgery Self Assessment Tool (OMSSAT)

The American Board of Oral and Maxillofacial Surgery 144

Reference: Dolwick, MF: Temporomandibular Joint Disc Displacement: A Clinical Perspective. In, Sessle, BJ; Bryant, PS; Dionne, RA (Eds): Temporomandibular Disorders and Related Pain Conditions. Progress in Pain Research and Management. Vol 4 1995,IASP Press. Seattle. pp.79 – 87. Milam, SB: Articular Disc Displacements and Degenerative Temporomandibular Joint Disease. In: Sessle, BJ; Bryant, PS; Dionne, RA (Eds): Temporomandibular Disorders and Related Pain Conditions. Progress in Pain Research and Management. Vol 4 1995 IASP Press. Seattle. Vol 4 pp.89 – 112.

Page 145: Anaesthesia TMJ Pain Orthognathic Pathology Reconstruction

2007 Oral and Maxillofacial Surgery Self Assessment Tool (OMSSAT)

The American Board of Oral and Maxillofacial Surgery 145

137. Which of the following is a prominent feature of type I condylar hyperplasia?

A. Ankylosis B. Deviation of chin to contralateral side C. open bite on ipsilateral side D. bilateral crossbite Answer: B Rationale:

Ankylosis and crossbite are not usually features of type I condylar hyperplasia. Open bite is a prominent feature of type II condylar hyperplasia. Deviation of the chin is a common feature of type I but not type II condylar hyperplasia.

Reference:

Stanton D, Stewart, J: Tumors of the Temporomandibular Joint. In Fonseca, R (ed): Oral and Maxillofacial Surgery Vol. 4. Philadelphia, WB Saunders, 2000, p 365.

Page 146: Anaesthesia TMJ Pain Orthognathic Pathology Reconstruction

2007 Oral and Maxillofacial Surgery Self Assessment Tool (OMSSAT)

The American Board of Oral and Maxillofacial Surgery 146

138. A 25 year-old female presents to the emergency department with a medial pole fracture of the right TMJ condyle. Treatment should consist of which of the following?

A. Open reduction and internal fixation followed by maxillomandibular fixation for 14

days B. Open reduction with internal fixation only C. Closed reduction with maxillomandibular fixation for 4 weeks to maintain proper

posterior vertical height. D. Medical management with soft diet only Answer: D Rationale:

The medial pole of the TMJ condyle by position does not function as a primary determinant of mandibular vertical height. As such, open or closed surgical approaches for such a fracture are unwarranted; and the injury is managed with soft diet and range of motion exercises.

Reference:

Sicher, H; and DuBrul, EL: Temporomandibular articulation. In, Oral Anatomy (6th Ed.) CV Mosby Co. St. Louis. pp. 160 - 191, 1975. Quinn, PD: Color Atlas of Temporomandibular Joint Surgery. CV Mosby, St. Louis, 1998. Assael, LA: Hard Tissue Trauma. In, Temporomandibular Joint Disorders - Diagnosis and Treatment. Kaplan, AS and Assael, LA (Eds) WB Saunders, Philadelphia, 1992. pp.224-237. Mercuri, LG: Temporomandibular Joint Disorders. In: Kwon, PH; and Laskin, DM (eds) Clinician' Manual of Oral and Maxillofacial Surgery 3rd ed. 2001 Quintessence. Chicago. Kobinson DA, Hohn FI: Traumatic injuries. In, Laskin DM, Greene CS, Hylander WL (eds): Temporomandibular Disorders: An Evidence-Based Approach to Diagnosis and Treatment. 2006 Quintessence. Chicago.

Page 147: Anaesthesia TMJ Pain Orthognathic Pathology Reconstruction

2007 Oral and Maxillofacial Surgery Self Assessment Tool (OMSSAT)

The American Board of Oral and Maxillofacial Surgery 147

139. A 25 year-old female presents to the emergency department with a medial pole fracture of the right TMJ condyle. Treatment should consist of which of the following?

A. Open reduction and internal fixation followed by maxillomandibular fixation for 14

days B. Open reduction with internal fixation only C. Closed reduction with maxillomandibular fixation for 4 weeks to maintain proper

posterior vertical height. D. Medical management with soft diet only Answer: D Rationale:

The medial pole of the TMJ condyle by position does not function as a primary determinant of mandibular vertical height. As such, open or closed surgical approaches for such a fracture are unwarranted; and the injury is managed with soft diet and range of motion exercises.

Reference:

Sicher, H; and DuBrul, EL: Temporomandibular articulation. In, Oral Anatomy (6th Ed.) CV Mosby Co. St. Louis. pp. 160 - 191, 1975. Quinn, PD: Color Atlas of Temporomandibular Joint Surgery. CV Mosby, St. Louis, 1998. Assael, LA: Hard Tissue Trauma. In, Temporomandibular Joint Disorders - Diagnosis and Treatment. Kaplan, AS and Assael, LA (Eds) WB Saunders, Philadelphia, 1992. pp.224-237. Mercuri, LG: Temporomandibular Joint Disorders. In: Kwon, PH; and Laskin, DM (eds) Clinician' Manual of Oral and Maxillofacial Surgery 3rd ed. 2001 Quintessence. Chicago. Kobinson DA, Hohn FI: Traumatic injuries. In, Laskin DM, Greene CS, Hylander WL (eds): Temporomandibular Disorders: An Evidence-Based Approach to Diagnosis and Treatment. 2006 Quintessence. Chicago.

Page 148: Anaesthesia TMJ Pain Orthognathic Pathology Reconstruction

2007 Oral and Maxillofacial Surgery Self Assessment Tool (OMSSAT)

The American Board of Oral and Maxillofacial Surgery 148

140. Which of the following ligaments restricts posterior movements of the mandibular condyle?

A. Stylomandibular ligament B. Sphenomandibular ligament C. Capsular ligament D. Lateral ligament Answer: D Rationale:

The lateral ligament, also known as the temporomandibular ligament, is a fan-shaped structure that runs obliquely in a posterior and caudal direction from the lateral aspect of the articular eminence to the posterior aspect of the mandibular condyle and lateral margin of the articular disk. The lateral ligament is composed of two elements: an outer oblique portion and an inner horizontal portion. The lateral ligament limits inferior and posterior displacement of the mandibular condyle, and to a limited extent the posterior displacement of the articular disk. The capsular ligament is a relatively thin structure originating from the rim of the glenoid fossa. This structure extends inferiorly to join the periosteum of the condylar process below the condylar head. The capsular ligament encloses the joint cavity, which is divided into a superior joint space and an inferior joint space by the articular disk. The stylomandibular ligament extends from the styloid process to the angle of the mandible. The sphenomandibular ligament, a remnant of the perichondrium of Meckel's cartilage, runs from the spine of the sphenoid bone to the mandibular lingula. The stylomandibular and sphenomandibular ligaments provide resistance to extreme anterior, lateral, and caudal displacements of the mandible, offering indirect support of the TMJ.

Reference:

Ten Cate, AR, Gross and micro anatomy. In: Zarb, GA, et al (eds) Temporomandibular Joint and Masticatory Muscle Disorders. 1994 Mosby, Munksgaard, p. 58.

Page 149: Anaesthesia TMJ Pain Orthognathic Pathology Reconstruction

2007 Oral and Maxillofacial Surgery Self Assessment Tool (OMSSAT)

The American Board of Oral and Maxillofacial Surgery 149

141. A 32 year-old patient presents with a history of increasing frequency of open lock. He now requires treatment in the local emergency room to be able to close his mouth. What intervention is most likely to provide long-term relief of his condition?

A. Injection of the lateral pterygoid muscle with botulinum toxin B. Eminectomy C. Intra-articular injection of a sclerosing agent D. Temporalis muscle scarification Answer: B Rationale:

Comprehensive literature review reveals that the of the interventions listed, only eminectomy is likely to provide long-term control of recurrent dislocation. Distractors: Botulinum toxin may be of benefit, but the literature is scanty and the effect temporary. Sclerosing agents have a disproportionately low success rate. Temporalis scarification does not have reported long-term follow-up.

Reference:

Shorey, CW and Campbell, JH. Temporomandibular joint dislocation. Oral Surg, Oral Med Oral Pathol Oral Diag Endod 2000;89:662-8

Page 150: Anaesthesia TMJ Pain Orthognathic Pathology Reconstruction

2007 Oral and Maxillofacial Surgery Self Assessment Tool (OMSSAT)

The American Board of Oral and Maxillofacial Surgery 150

142. Agenesis of the mandibular condyle can be the result of which of the following?

A. Malformation of the third branchial arch B. Damage to the stapedial artery during development in utero C. Folic acid deficiency D. Niacin deficiency Answer: B Rationale:

A. Agenesis of the mandibular condyle is also called “First Arch Syndrome” since the first, not the third, branchial arch is involved. B. This is the suspected etiology of hemifacial microsomia and related disorders, based on the work of Poswillo. C. and d. Vitamin deficiencies have nothing to do with this entity.

Reference:

Mercuri, LG: Temporomandibular Joint Disorders. In: Kwon, PH; and Laskin, DM (Eds): Clinician Manual of Oral and Maxillofacial Surgery 3rd ed. 2001 Quintessence Publishing Co. Chicago. Poswillo, D: The pathogenesis of Treacher-Collins Syndrome (mandibular-facial dysostosis). Br J Oral Surg 13:1, 1975.

Page 151: Anaesthesia TMJ Pain Orthognathic Pathology Reconstruction

2007 Oral and Maxillofacial Surgery Self Assessment Tool (OMSSAT)

The American Board of Oral and Maxillofacial Surgery 151

Troulis MJ, Kaban LB: Congenital and developmental anomalies. In, Laskin DM, Greene CS, Hylander WL (eds): Temporomandibular Disorders: An Evidence-Based Approach to Diagnosis and Treatment. 2006 Quintessence. Chicago.

Page 152: Anaesthesia TMJ Pain Orthognathic Pathology Reconstruction

2007 Oral and Maxillofacial Surgery Self Assessment Tool (OMSSAT)

The American Board of Oral and Maxillofacial Surgery 152

143. A 45 year-old male presents with recurrent left TMJ ankylosis and maximum interincisal opening of 10mm. 5 years ago he underwent closed reduction of a mandibular condyle fracture. One year after the closed reduction he underwent autogenous costochondral reconstruction for TM joint ankylosis. Subsequently he underwent repeat costochondral reconstruction for recurrent ankylosis. In this situation, which treatment demonstrates the lowest rate of recurrent ankylosis?

A. Gap arthroplasty with the insertion of a permanent silicone rubber block B. Gap arthroplasty with the insertion of a temporalis muscle rotation flap and autogenous

costochondral graft C. Gap arthroplasty with the insertion of autogenous fat graft D. Gap arthroplasty with the insertion of an alloplastic TMJ prosthesis Answer: D Rationale:

A. Silicone rubber implants are not a satisfactory material for use as a permanent interpositional implant in the TMJ. Silicone rubber has poor wear properties in load-bearing joints. B. Autogenous bone grafting for re-ankylosis of the TMJ has poor outcome. C. Autogenous fat grafting has been used successfully to prevent re-ankylosis of the TMJ, but not as a sole entity. D. Re-ankylosis of the TMJ is best managed by total TMJ alloplastic reconstruction and autogenous fat grafting to minimize heterotopic bone formation around the prosthesis.

Page 153: Anaesthesia TMJ Pain Orthognathic Pathology Reconstruction

2007 Oral and Maxillofacial Surgery Self Assessment Tool (OMSSAT)

The American Board of Oral and Maxillofacial Surgery 153

Reference: Mercuri, LG: Temporomandibular Joint Disorders. In: Kwon, PH; and Laskin, DM (eds). Clinician Manual of Oral and Maxillofacial Surgery 3rd ed. 2001 Quintessence Publishing Co. Chicago. Anonymous.Recommendations for Management of Patients With Temporomandibular joint Implants. J Oral Maxillofac Surg 1993; 51:1164-72. Mercuri LG, Wolford LM, Sanders B, et al: Long-term follow-up of the CAD/CAM patient fitted total temporomandibular joint reconstruction system. J Oral Maxillofac Surg 2002;60:1440. Mercuri LG: TMJ Concepts Patient Fitted Total TMJ Reconstruction Prostheses. Oral Maxillofacial Surg Clinics North Am 2000;12:73.

Page 154: Anaesthesia TMJ Pain Orthognathic Pathology Reconstruction

2007 Oral and Maxillofacial Surgery Self Assessment Tool (OMSSAT)

The American Board of Oral and Maxillofacial Surgery 154

144. A 7 year-old child with a history of facial trauma at age 5 presents with oral opening limited to 4 mm and left-sided mandibular deviation. The surgical intervention most likely to provide functional and esthetic rehabilitation into adulthood is:

A. alloplastic total joint reconstruction.

B. alloplastic condylar replacement.

C. gap arthroplasty with auricular cartilage graft.

D. autogenous costochondral graft.

Answer: D Rationale:

The history and image indicate bony ankylosis of the temporomandibular joint in a growing child. When reconstructing the temporomandibular joint in children, the surgeon must provide both functional anatomy and an environment in which growth may occur. Donor sites with growth potential are preferred. Alloplastic total or partial joint reconstruction or gap arthroplasty will not support growth.

Reference:

American Association of Oral and Maxillofacial Surgeons. Parameters and Pathways: Clinical Practice Guidelines for Oral and Maxillofacial Surgery. J Oral Maxillofac Surg, Version 3.0, TMJ-6, 2001

Page 155: Anaesthesia TMJ Pain Orthognathic Pathology Reconstruction

2007 Oral and Maxillofacial Surgery Self Assessment Tool (OMSSAT)

The American Board of Oral and Maxillofacial Surgery 155

145. During surgical removal of an ankylosed condyle brisk arterial bleeding is encountered and

localized to the region medial to the lateral pterygoid muscle. Which terminal branch of the carotid artery is the most likely source of the bleeding?

A. Internal maxillary artery B. Anterior deep temporal artery C. Masseteric artery D. Middle meningeal artery Answer: D Rationale:

The middle meningeal artery is a branch of the internal maxillary artery, coursing medially from the internal maxillary artery ascending between the sphenomandibular ligament and the lateral pterygoid muscle, passing through the foramen spinosum. The average distance from the glenoid fossa to the middle meningeal artery is 2.4 mm. The close proximity of the middle meningeal artery to the TMJ places it at risk during complex procedures that require dissection medical to the mandibular condyle.

Reference:

Cillo JE, Sinn D, Truelson JM. Management of middle meningeal and superficial temporoal artery hemorrhage from total temporomandibular joint replacement surgery with a gelatin-based hemostatic agent. J Craniofac Surg 2005;16:309-12

Page 156: Anaesthesia TMJ Pain Orthognathic Pathology Reconstruction

2007 Oral and Maxillofacial Surgery Self Assessment Tool (OMSSAT)

The American Board of Oral and Maxillofacial Surgery 156

146. A 60 year-old post-menopausal female presents to the emergency department with the complaint of transient but sudden, severe unilateral headache. Her past medical history is significant for hypertension, tobacco and alcohol abuse. Her neurological examination is normal. This clinical history is most compatible with which diagnosis.

A. variant migraine. B. cerebral vasospasm. C. giant cell arteritis. D. subarachnoid hemorrhage. Answer: D Rationale:

A retrospective analysis of patients with subarachnoid hemorrhage (SAH) suggests that minor episodes with sudden headache may precede rupture of an aneurysm. Up to 40% of patient will experience a sentinel headache a week or more prior to rupture of a cerebral aneurysm. This woman's history is compatible with sentinel headache for a “leaking” aneurysm. Risk factors for the development of SAH are 1) smoking, 2)hypertension, and 3)alcohol, 4)estrogen depletion. Variant migraine includes such findings as hemiplegia, brain stem symptoms (vertigo, etc.), transient blindness, ophthalmoplegia, not present in this woman. Giant cell arteritis is associated with unilateral temporal headache, jaw claudication, and possible symptoms of retinal ischemia, not manifested in this woman. Cerebral vasospasm is a complication of SAH leading to neurologic decline in the days following the SAH.

Reference:

Ingall, T, Asplund, K, Mahonen, M, Bonita, R. A multinational comparison of subarachnoid hemorrhage epidemiology in the WHO MONICA stroke study. Stroke 2000; 31:1054 Edlow, JA, Caplan, LR. Avoiding pitfalls in the diagnosis of subarachnoid hemorrhage. N Engl J Med 2000; 342:29 Lipton, RB, Bigal, ME, Steiner, TJ, et al. Classification of primary headaches. Neurology 2004; 63:427 Hunder, GG. Giant cell arteritis and polymyalgia rheumatica. In: Textbook of Rheumatology, 6th ed, Kelly, WN, Harris, ED, Ruddy, S, et al (Eds), WB Saunders Company, Philadelphia, 2001

Page 157: Anaesthesia TMJ Pain Orthognathic Pathology Reconstruction

2007 Oral and Maxillofacial Surgery Self Assessment Tool (OMSSAT)

The American Board of Oral and Maxillofacial Surgery 157

147. The diagnosis of sympathetically maintained pain caused by injury to the inferior alveolar nerve can be confirmed by which test?

A. Regional nerve block with local anesthesia

B. Somatosensory evoked potential recording

C. Magnetic resonance neurography of the mandible

D. Stellate ganglion blockade of the superior stellate ganglion

Answer: D Rationale:

Sympathetic maintained pain (SMP) results from tonic activity in myelinated mechanoreceptor afferents, whose activity is induced by sympathetic efferent actions on sensory receptors, and this afferent input causes tonic firing in previously sensitized wide-dynamic (WDR) or mulitreceptive neurons that are part of a central nociceptive pathway. A burning painful sensation results from this chain of actions. Insert

Reference:

Gregg JM: Studies of traumatic neuralgia in the maxillofacial region: symptom complexes and response to microsurgery. J Oral Maxillofac Surg 48:135, 1990 Roberts W: A hypothesis on the physiological basis for causalgia and related pain. Pain 24:297, 1986 Raja SN. Turnquist JL. Meleka S. Campbell JN. Monitoring adequacy of alpha-adrenoceptor blockade following systemic phentolamine administration. [Clinical Trial. Controlled Clinical Trial. Journal Article] Pain. 1996;64):197-204, 1996

Page 158: Anaesthesia TMJ Pain Orthognathic Pathology Reconstruction

2007 Oral and Maxillofacial Surgery Self Assessment Tool (OMSSAT)

The American Board of Oral and Maxillofacial Surgery 158

148. A 38 year-old female with a long history of periodic migraine headache has, for the past 3 months, experienced an insidious increase in headache frequency to near daily headache and frequent awakening with headache. For the past 4 months she has tried to manage her headache with daily over-the-counter acetaminophen, aspirin, and caffeine containing analgesics. Her present condition is best described by:

A. variant migraine. B. rebound-type transformed migraine. C. cluster headache. D. hemiplegic migraine. Answer: B Rationale:

Variant migraine refers anyone of several sub-classifications of migraine such as retinal migraine, hemiplegic migraine, etc. Cluster headache is a distinct clinical syndrome affecting men more than woman, with attacks accompanied by ipsilateral lacrimation, nasal discharge, ptosis, conjunctival injection, and pupillary change. Hemiplegic migraine is a migraine variant characterized by motor and sensory symptoms that are unilateral. Symptoms can last longer than the headache itself and complete recovery may take weeks in some cases. Transformed headache is a transformation from intermittent migraine headache attacks to daily or almost daily headache. The excessive and frequent use of symptomatic medications including ergot derivatives, analgesics and high-dose NSAID's can result in the insidious transformation to daily headaches. This has been also referred to as medication-overuse headache where daily headache is induced and maintained by the medications used to relieve such pain.

Reference:

Dodick DW. Chronic daily headache. N Engl J Med 2006;354:158-65 Saper JR. Headache disorders. Med Clin North Am 1999; 83:663-90 Corbett, JJ. Neuro-ophthalmic complications of migraine and cluster headaches. Neurol Clin 1983; 1:973

Page 159: Anaesthesia TMJ Pain Orthognathic Pathology Reconstruction

2007 Oral and Maxillofacial Surgery Self Assessment Tool (OMSSAT)

The American Board of Oral and Maxillofacial Surgery 159

149. Excruciating unilateral periorbital burning or piercing pain episodes lasting 30 – 60 minutes, associated with ipsilateral lacrimation are characteristic of:

A. migraine headache. B. trigeminal neuralgia. C. tension headache. D. cluster headache. Answer: D Rationale:

Cluster headache has a 6:1 male predilection, is most commonly periorbital and excruciating, and is strictly unilateral. Episodes last from 15 – 180 minutes, but most commonly from 30 – 60 minutes. Occurrence is from one every other day to eight per day, and they tend to “cluster” over time; each cluster lasts from 1 – 2 months, with one or two clusters per year. Pain is commonly described as burning, piercing, or neuralgic, and ipsilateral autonomic symptoms such as lacrimation, rhinorrhea, miosis, and/or ptosis occur. Migraine typically exhibits throbbing unilateral pain lasting from 4 – 72 hours, nausea, photophobia, and exacerbation by physical activity. Trigeminal neuralgia can be differentiated from cluster headache because pain is transient and not associated with autonomic signs/symptoms. Tension headache is commonly bandlike and bilateral, lasts 30 minutes to seven days, of mild to moderate intensity that does not prohibit activity, and not associated with nausea/vomiting or photophobia.

Reference:

Smetana, GW. The diagnostic value of historical features in primary headache syndromes: a comprehensive review. Arch Int Med 200;160:2729-37.

Page 160: Anaesthesia TMJ Pain Orthognathic Pathology Reconstruction

2007 Oral and Maxillofacial Surgery Self Assessment Tool (OMSSAT)

The American Board of Oral and Maxillofacial Surgery 160

150. A 40 year-old male is referred for evaluation of facial pain and headache. He describes a long history of temporal/periorbital region headaches. His headaches generally last 30 minutes, occur 3-4 times per day over a 2 month period, followed by several headache-free months. Other symptoms characteristic of this man’s headache diagnosis are:

A. photophobia, phonophobia. B. lacrimation and rhinorrhea. C. meningismus. D. nausea and vomiting. Answer: B Rationale:

The four categories of primary headache include migraine, tension-type, cluster headache, and other trigeminal autonomic cephalgias and other primary headaches. This man's history is most compatible with cluster headache. Cluster headache is a distinct clinical syndrome affecting men more than woman, with attacks accompanied by ipsilateral lacrimation, nasal discharge, ptosis, conjunctival injection, and pupillary change. The features most predictive of migraine are nausea, photophobia, phonophobia, exacerbation with physical activity. Meningismus is associated such acute events as subarachnoid hemorrhage.

Reference:

Saper JR. Headache disorders. Med Clin North Am 1999; 83:663-90 Smetana, GW. The diagnostic value of historical features in primary headache syndromes. A comprehensive review. Arch Intern Med 2000; 160:2729. Bahra, A, May, A, Goadsby, PJ. Cluster headache: A prospective clinical study with diagnostic implications. Neurology 2002; 58:354. Lipton, RB, Bigal, ME, Steiner, TJ, et al. Classification of primary headaches. Neurology 2004; 63:427.

Page 161: Anaesthesia TMJ Pain Orthognathic Pathology Reconstruction

2007 Oral and Maxillofacial Surgery Self Assessment Tool (OMSSAT)

The American Board of Oral and Maxillofacial Surgery 161

151. A healthy 65 year-old male is referred with a 3 week history of severe, “shock-like”, paroxysmal pain in the right tonsillar and ear region associated with swallowing cold liquids. He recently experienced a syncopal event where he was noted to be profoundly bradycardic with one of the attacks. This man’s history is most compatible with which diagnosis:

A. carotodynia. B. Eagle’s syndrome. C. glossopharyngeal neuralgia. D. sphenopalatine neuralgia. Answer: C Rationale:

Glossopharyngeal neuralgia is defined as paroxysmal pain in areas innervated by cranial nerves IX and X. Pain may be experienced in the ear, larynx, tonsillar region, and tongue. It is almost always unilateral. Triggers include chewing, swallowing, coughing, speaking, and yawning. Severe attacks have been associated with bradycardia/syncope through the vagal motor nucleus. Carotodynia is pain that originates in the carotid artery. The pain is usually unilateral, provoked by swallowing, coughing, or neck movement, and typically lasts for several days to months. Eagle's syndrome is loosely associated with ossification of the stylohyoid ligament. Dull pharyngeal and neck pain possibly provoked with swallowing and head turning is described. Palpation of the tonsillar fossa and/or hyoid may illicit pain. Sphenopalatine (greater superficial petrosal) neuralgia presents as unilateral, episodic pain in the perinasal region, associated with nasal congestion. Some case may be attributable to cluster headache and may not represent a unique diagnostic entity.

Reference:

Rushton, JG, Stevens, JC, Miller, RH. Glossopharyngeal (vagoglossopharyngeal) neuralgia: a study of 217 cases. Arch Neurol 1981; 38:201 Elias, J, Kuniyoshi, R, Valadao, W, et al. Glossopharyngeal neuralgia associated with cardiac syncope. Arq Bras Cardiol 2002; 78:515 Fini, G, Gasparini, G, Filippini, F, et al. The long styloid process syndrome or Eagle's syndrome. J Craniomaxillofac Surg 2000; 28:123

Page 162: Anaesthesia TMJ Pain Orthognathic Pathology Reconstruction

2007 Oral and Maxillofacial Surgery Self Assessment Tool (OMSSAT)

The American Board of Oral and Maxillofacial Surgery 162

Bruyn, G. Sphenopalatine neuralgia (Slyder). In: Vinken, PJ, Bruyn, GW, Klawanus, HC, Rose, FC (eds.), Handbook of Clinical Neurology, Amsterdam, Elselvier, 1986, p 475-482 Raskin, NH. Headache, 2d ed, New York, Churchill Livingstone, 1988, p 353-357

Page 163: Anaesthesia TMJ Pain Orthognathic Pathology Reconstruction

2007 Oral and Maxillofacial Surgery Self Assessment Tool (OMSSAT)

The American Board of Oral and Maxillofacial Surgery 163

152. A healthy 75 year-old female presents with new temporal region headache and trismus. On examination she is mildly febrile (38oC), with focal tenderness in the temporal and occipital regions. Examination of her visual fields reveals a deficit in the temporal field of her right eye. What is the most likely diagnosis?

A. Polymyalgia rheumatica B. Polyarteritis nodosa C. Giant cell arteritis D. Variant (retinal) migraine Answer: C Rationale:

Giant cell arteritis(GCA) is a vasculitis of large and medium size vessels that most prominently involves the cranial arteries. Symptoms in this patient compatible with GCA are fever, temporal head pain and trismus. Polymyalgia rheumatica (PMR) is closely linked to GCA but characterized by aching and morning stiffness in the shoulder, hip girdles, neck and torso. Polyarteritis nodosa (PAN) is a systemic necrotizing vasculitis effecting small and medium size vessels. Patients with PAN present with fatigue, weakness, fever, arthralgias, and polyneuropathy. Systems affected include neurologic, skin, renal, and gastrointestinal. Retinal or ocular migraine is a rare variant migraine characterized by repeated attacks of monocular scotomata or blindness lasting less than one hour, associated with or followed by headache.

Reference:

Saper JR. Headache disorders. Med Clin North Am 1999; 83:663-90 Corbett, JJ. Neuro-ophthalmic complications of migraine and cluster headaches. Neurol Clin 1983; 1:973 Hunder, GG. Giant cell arteritis and polymyalgia rheumatica. In: Textbook of Rheumatology, 6th ed, Kelly, WN, Harris, ED, Ruddy, S, et al (Eds), WB Saunders Company, Philadelphia, 2001 Conn, DL. Polyarteritis. In: Rheumatology, Klippel, JH, Dieppe, PA (Eds), Mosby, St Louis, 1994

Page 164: Anaesthesia TMJ Pain Orthognathic Pathology Reconstruction

2007 Oral and Maxillofacial Surgery Self Assessment Tool (OMSSAT)

The American Board of Oral and Maxillofacial Surgery 164

153. A neuropathic pain condition that is characterized by summation, delayed onset, overshoot and aftersensation to a mechanical stimulus represents which condition? A. allodynia B. anesthesia dolorosa C. hyperalgsia D. hyperpathia Answer: D Rationale:

Neuropathic pain is described by the patient as pain that is severe and “out of proportion” to the stimulus or pain that is often spontaneous, continuous, or lingering in nature. Stimulus induced neuropathic pains are: 1. Allodynia: painful response to normally non-painful stimulus. For example, moving a brush across the zone of injury causes a very painful stimulation of the area. Normally a brush moving across the adjacent uninjured area is non-painful. Axons of cutaneous afferent neurons become sensitive to mechanical stimulus. Local inflammatory mediators at or around the injured nerve terminals or axons decrease the threshold for activation and the response magnitude to constant stimuli may increase. This sensitization, which is caused by endogenous bradykinin, prostaglandin and others may enhance mechanosensitivity of afferent neurons. 2. Anesthesia Dolorosa: painful response in a zone of complete anesthesia. For example, pain is still reported in a zone that has no response to tactile, thermal, painful stimuli and not resolved with peripheral nerve blocks of the same zone. These findings indicate a central origin of the pain response. 3. Hyperalgesia: enhanced pain response. For example, thermal pain tolerance over the mental fold in a healthy adult is between 48o and 50o C. If the recorded pain tolerance in the same area is now 45o C, then the patient has hyperalgesia. They have pain response and sensitivity, but it is abnormally low in threshold and therefore previously non-painful levels of a painful stimulus (cold, hot, pressure, etc.) is now painful and normal just detectable painful stimulus is enhanced. 4. Hyperpathia: a painful syndrome characterized by an abnormally painful reaction to a stimulus, especially a repetitive stimulus, as well as an increased threshold. For example, in applying a neurosensory test protocol; a series of about ten tactile pulses with Semmes Weinstein monofilaments slowly administered (1/sec) followed by a 1 minute rest period. This protocol is used to identify delayed onset in pain. The pain may alternatively begin during the stimulation but continue for appreciable durations exceeding 1minute after the stimulation is terminated and is an example of aftersensation. If the pain begins during the stimulation, the approximate number of pulses required for its elicitation should be recorded to describe the summation. If the

Page 165: Anaesthesia TMJ Pain Orthognathic Pathology Reconstruction

2007 Oral and Maxillofacial Surgery Self Assessment Tool (OMSSAT)

The American Board of Oral and Maxillofacial Surgery 165

intensity of the pain is reported to increase with continued stimulation, overshoot is present. One or more observations of delayed onset, aftersensation, overshoot, or summation are sufficient for a diagnosis of a hyperpathic pain condition. 5. Sympathetic Maintained Pain (SMP): Sympathetic maintained pain results from tonic activity in myelinated mechanoreceptor afferents, whose activity is induced by sympathetic efferent actions on sensory receptors, and this afferent input causes tonic firing in previously sensitized wide-dynamic (WDR) or mulitreceptive neurons that are part of a central nociceptive pathway. A burning painful sensation results from this chain of actions. A regional nerve block with local anesthesia would result in the reduction of the mechanical sensitivity but not the tonic firing of WDR neuron generated impulses. Brush-stoke directional testing of the involved area would demonstrated mechanical allodynia but would not be able to distinguish peripheral mechanical allodynia from SMP pain.

Reference:

Dworkin RH: An overview of neuropathic pain: syndromes, symptoms, signs, and several mechanisms. Clin J Pain 2002;18:343. Coderre TJ, Melzack R: Cutaneous hyperalgesia: contributions of the peripheral and central nervous systems to the increase in pain sensitivity after injury. Brain Res 1987;404:95. Ochs G, Schenk M, Struppler A: Painful dysaesthesia following peripheral nerve injury: a clinical and electrophysiological study. Brain Res 1987;496:228. Michaelis M, Vogel C, Blenk K-H, Arnarson A, Janig W: Inflammatory mediators sensitize acutely axotomized nerve fibers to mechanical stimulation in the rat. J Neurosci 1998;18:7581. Zuniga JR, Essick GK: A contemporary approach to the clinical evaluation of trigeminal nerve injuries. Oral Maxillofac Surg Clinic North Am 1992;4:353.

Page 166: Anaesthesia TMJ Pain Orthognathic Pathology Reconstruction

2007 Oral and Maxillofacial Surgery Self Assessment Tool (OMSSAT)

The American Board of Oral and Maxillofacial Surgery 166

154. After injury to the mental nerve, a painful neuroma has formed. Surgical exploration, reveals no viable nerve distal to the neuroma. The recommended surgical treatment of a painful neuroma when there is no distal nerve element is resection of the neuroma and:

A. intraneural steroid application. B. intraneural alcohol injection. C. fasciculectomy with lateralization of the inferior alveolar nerve. D. fasciculectomy with closure using an epineurial sleeve. Answer: D Rationale:

Painful neuromas form following injuries associated with third, fourth and fifth degree nerve injuries and following inadequate surgical repair of severed nerves. Surgical treatment of painful neuromas should include the exposure and separation of the neuroma from adjacent tissues so that the proximal and distal nerves can be adequately exposed for microscopic examination. After the neuroma is resected, the surgical treatment chosen depends upon the presence or absence of distal nerves. In the absence of adequate distal nerve tissue to repair, the treatment should be to reduce or eliminate the recurrence of a painful neuroma. Intraneural steroid applications may induce a chemical neuritis and increase the pain associated with the neuroma and is largely contraindicated for treatment of neuroma Resection of the neuroma will leave the proximal end intact so that regrowth of the neuroma is highly likely. Neuromas form when amputated axons develop sprouting axons (neurites) in such large numbers that they collectively form a mass of unorganized unmyelinated axons that are sensitive to environmental conditions, both physical and chemical. Resection of the neuroma and transposition and implantation of the amputated end into muscle tissue is one surgical option in this case. Muscle tissue reduces neurite formation, in part, due to acetylcholine receptor or negative RATIONALE to neurite growth. Resection of the neuroma and lateralization of the inferior alveolar nerve does not accomplish the end goal of implantation into muscle tissue. In a funiculectomy procedure of a peripheral nerve, the fascicles are exposed and sectioned and allowed to retract within the epineurial sleeve. The epineurium is then brought out to full length, and a double or triple suture ligature is placed around the epineurial sleeve so that the cut nerve is within its epineurial space. The proposed mechanism of action of funiculectomy and epineurial sleeve method is to reduce

Page 167: Anaesthesia TMJ Pain Orthognathic Pathology Reconstruction

2007 Oral and Maxillofacial Surgery Self Assessment Tool (OMSSAT)

The American Board of Oral and Maxillofacial Surgery 167

fibroproliferation of nerve and supporting tissue and reduce the size of a resulting neuroma. (Figure 2)

Reference:

Williams HB: The painful stump neuroma and its treatment. In Microreconstruction of Nerve Injuries. 1987 WB Saunders, Philadelphia, pp 165-171 Zuniga JR: Surgical management of trigeminal neuropathic pain. Atlas Oral Maxillofac Surg Clinic North Am 2001;9:59.

Page 168: Anaesthesia TMJ Pain Orthognathic Pathology Reconstruction

2007 Oral and Maxillofacial Surgery Self Assessment Tool (OMSSAT)

The American Board of Oral and Maxillofacial Surgery 168

155. Therapy for chronic central neuropathic pain after injury to the inferior alveolar nerve following third molar surgery includes:

A. external neurolysis with primary neurorrhaphy. B. anti-convulsants. C. non-steroidal antiinflammatory medication. D. low energy laser application. Answer: B Rationale:

Pharmacological interventions for the management of neuropathic pain very widely in terms of their safety, efficacy, tolerability, titration requirements and cost. Opioids were previously considered ineffective but recent clinical trials demonstrate they offer effective pain relief in many instances but can be subject to diversion and abuse. Tricyclic antidepressants (amitriptyline) also have proven efficacy but tolerability concerns and titration complexities can limit their practicality for use. Selected anti-convulsants agents have proven effectiveness in neuropathic pain (Clonazepam and gabapentin). Finally, nonsteroidal anti-inflammatory agents are inexpensive, well tolerated, and easy to administer, but lack efficacy in neuropathic pain. Peripheral nerve surgery has little role in the management of centrally mediated pain.

Reference:

Mendell JR, Sahenk Z: Painful sensory neuropathy. N Engl J Med 2003;348:1243. Rowbotham MC, Twilling L, Davies PS, Reisner L, Taylor K, Mohr D: Oral opioid therapy for chronic peripheral and central neuropathic pain. N Eng J Med 2003;l J Med 348:1223. Gilron I, Bailey JM, Tu D, Holden RR, Weaver DF, Houlden RL: Morphine, gabapentin, or their combination for neuropathic pain. N Eng J Med 2005;352:1324. Gregg JM: Medical management of traumatic neurpathies. Oral Maxillofac Surg Clinic North Am 2001;13:343.

Page 169: Anaesthesia TMJ Pain Orthognathic Pathology Reconstruction

2007 Oral and Maxillofacial Surgery Self Assessment Tool (OMSSAT)

The American Board of Oral and Maxillofacial Surgery 169

156. 3 months after third molar extraction, a patient is experiencing dysesthesia in the distribution of the inferior alveolar nerve. Which neuropathic pain characteristics and clinical findings would support microsurgical intervention?

A. Mechanical allodynia that is resolved with a proximal local anesthetic nerve block B. Burning hyperpathia that is resolved with a stellate ganglion block C. Mechanical allodynia that is not resolved with a proximal local anesthetic nerve block D. Burning hyperpathia in which only the hyperpathia is resolved with a proximal local anesthetic nerve block. Answer: A Rationale:

Diagnostic nerve blocks are employed to weigh the relative contributions of peripheral and central factors to the neuropathic pain condition. Should peripheral nerve blocks proximal to the site of injury significantly decrease the pain complaint, then peripheral factors are accepted as responsible for the discomfort. Should the peripheral nerve block fail to significantly decrease the pain, central factors are likely responsible for the pain and discomfort. For sympathetic maintained pain conditions, blockade of sympathetic afferent nerves results in elimination of the pain. When SMP involves the cranial nerves, a stellate ganglion block is utilized to relieve SMP pain. The literature supports the concept that invasive treatments (microsurgery of the inferior alveolar nerve) are more frequently helpful (slightly better than chance improvement) in peripheral neuropathic pain states than in central or sympathetic maintained pain states.

Reference:

Campbell JN, Raja SN, Meyer RA: Painful sequelae of nerve injury. In Dubner R, Gebhart GF, Bond MR (eds): Proceedings of the Fifth World Congress on Pain. 1988 Elsevier, Amsterdam, p 135 Gregg JM: Studies of traumatic neuralgia in the maxillofacial region: symptom complexes and response to microsurgery. J Oral Maxillofac Surg 1990;48:135. Zuniga JR: Surgical management of trigeminal neuropathic pain. Atlas Oral Maxillofac Surg Clinic North Am 2001;9:59.

Page 170: Anaesthesia TMJ Pain Orthognathic Pathology Reconstruction

2007 Oral and Maxillofacial Surgery Self Assessment Tool (OMSSAT)

The American Board of Oral and Maxillofacial Surgery 170

157. A neonatologist requests an evaluation of a week old baby who has an oral mass that was present at birth. On examination the mass measures 2 x 1.5 cm, is pink in color and is attached to the anterior maxillary gingiva by a narrow stalk. What is the most likely diagnosis for this lesion?

A. Congenital granular cell tumor B. Infantile hemangioma C. Arteriovenous malformation D. Melanotic neuroectodermal tumor of infancy Answer: A

Rationale:

The primary clinical features that are consistent with a congenital granular cell tumor and rule out the other possibilities are presence at birth and that the lesion is attached by a stalk. A hemangioma and arteriovenous malformation can be present at birth but would not be attached by a stalk. A melanotic neuroectodermal tumor of infancy is not present at birth.

Reference: Kaban LB, Troulis MJ: Pediatric Oral and Maxillofacial Surgery, 2004, p 260-262; Marx RE, Stern D: Oral and Maxillofacial Pathology, 2003, p 427.

Page 171: Anaesthesia TMJ Pain Orthognathic Pathology Reconstruction

2007 Oral and Maxillofacial Surgery Self Assessment Tool (OMSSAT)

The American Board of Oral and Maxillofacial Surgery 171

158. Appropriate soft tissue margins for resection of squamous cell carcinoma of the oral cavity are:

A. 0.25 – 0.75 cm. B. 1.0 – 1.5 cm. C. 2.0 – 2.5 cm. D. 3.0 – 3.5 cm. Answer: B Rationale:

Resection of at least 1.0 cm beyond the clinical or microscopic disease is indicated in the treatment of squamous cell carcinoma of the oral cavity. 0.25 cm is an inadequate margin and while some authors have suggested resections of 1.0-1.5 cm, a 2.0 cm margin or beyond for this disease process is not indicated.

Reference:

MacIntosh R. Oral Malignant Disease: Management and Investigational Directions. Fonseca Oral and Maxillofacial Surgery. Williams Stewart, 152-235, 2000.

Page 172: Anaesthesia TMJ Pain Orthognathic Pathology Reconstruction

2007 Oral and Maxillofacial Surgery Self Assessment Tool (OMSSAT)

The American Board of Oral and Maxillofacial Surgery 172

159. Which of the following clinical findings is most consistent with a diagnosis of chronic, diffuse sclerosing osteomyelitis of the mandible?

A. Pain B. Lack of expansion of the mandible C. The presence of an oro-cutaneous fistula D. Sclerotic bone located at the alveolus of the anterior mandible Answer: A

Rationale:

Chronic diffuse sclerosing osteomyelitis (CDSO) of the mandible is a poorly understood clinical entity that is most often confused with florid osseous dysplasia. Typically, CDSO is very painful, associated with expansion of the mandible, located diffusely within the mandible from alveolus to the inferior border, and is unassociated with an oro-cutaneous fistula.

Reference:

Marx R.E., Carlson E.R., Smith B.R. and Torayo N.: Isolation of Actinomyces Species and Eikenella Corrodens from Patients with Chronic Diffuse Sclerosing Osteomyelitis. J Oral Maxillofac Surg, 1994; 52:26-33.

Page 173: Anaesthesia TMJ Pain Orthognathic Pathology Reconstruction

2007 Oral and Maxillofacial Surgery Self Assessment Tool (OMSSAT)

The American Board of Oral and Maxillofacial Surgery 173

160. A 15 year-old female presents with an expansile mass of the anterior mandible. The CT scan reveals calcifications and a well corticated lesion. The incisional biopsy demonstrates the presence of the microcalcifications and shows epithelial-lined duct-like structures in a background of swirling spindle-shaped epithelial cells. Which of the following diagnoses is supported by this description?

161.

A. Ameloblastoma, multilocular B. Ameloblastoma, unilocular C. Adenomatoid odontogenic tumor D. Odontogenic myxoma Answer: C Rationale:

The clinical history and histopathology is characteristic of adenomatoid odontogenic tumor that is the correct answer. By definition ameloblastomas are odontogenic tumors that do not produce calcifications and do not have duct-like structures (epithelial lined structures with lumin). In addition, the age of patient, site of lesion and imaging is not characteristic of both ameloblastomas. Therefore options A & B are incorrect. Option D is also incorrect since odontogenic myxoma is a mesenchymal tumor.

Page 174: Anaesthesia TMJ Pain Orthognathic Pathology Reconstruction

2007 Oral and Maxillofacial Surgery Self Assessment Tool (OMSSAT)

The American Board of Oral and Maxillofacial Surgery 174

Reference:

Abaza, NA: Ultrastructural features and biologic aspects of odontogenic cysts and tumors. In: Oral and Maxillofac Surg Clin of North Amer, L. Gold, (editor), 1994, P 523-577. W.B. Saunders, Philadelphia. Rick CM: Adenomatoid odontogenic tumor. In: Oral & Maxillofac Surg Clin North Amer., Odontogenic Tumors, DK White (editor), Saunders, Philadelphia; 2004, p333-354.

Page 175: Anaesthesia TMJ Pain Orthognathic Pathology Reconstruction

2007 Oral and Maxillofacial Surgery Self Assessment Tool (OMSSAT)

The American Board of Oral and Maxillofacial Surgery 175

162. This photomicrograph is illustrative of which of the following diagnoses?

A. Calcifying odontogenic cyst. B. Squamous cell carcinoma C. Desmoplastic ameloblastoma D. Parakeratotic odontogenic cyst Answer: A Rationale:

The photomicrograph reveals well illustrated “ghost cells” (sheets of epithelial cell that only reveals shadow outlines of cell membranes and shadow of the cell nuclei). There are also cuboidal epithelium and stellate-like cells. This is characteristic histology of calcifying odontogenic cysts. Option B is incorrect since there is no sign of cellular atypia and the “ghost cells” should not be confused with keratin pearls. Option C is incorrect since “ghost cells” are not reported as components of the histopathology of desmoplastic ameloblastoma. In addition, the glassy and dense stroma of desmoplastic ameloblastoma is lacking from the micrograph. Option D is incorrect; there is not a 7-8 cell layer thick, corrugated and parakeratinized luminal epithelial lining illustrated.

Page 176: Anaesthesia TMJ Pain Orthognathic Pathology Reconstruction

2007 Oral and Maxillofacial Surgery Self Assessment Tool (OMSSAT)

The American Board of Oral and Maxillofacial Surgery 176

Reference: Kramer IRH, Pindborg JJ, Shear M: Histological typing of odontogenic tumors, 2nd edit, Berlin, 1992, Springer-Verlag. Tomich CE: Calcifying odontogenic cyst and dentinogenic ghost cell tumor. In: Oral Maxillofac Surg Clin North Amer; Odontogenic Tumors, DK White (editor) Saunders, Philadelphia, 2004, p391-397

Page 177: Anaesthesia TMJ Pain Orthognathic Pathology Reconstruction

2007 Oral and Maxillofacial Surgery Self Assessment Tool (OMSSAT)

The American Board of Oral and Maxillofacial Surgery 177

163. This photomicrograph represents a lesion that is best described as which of the following?

A. Benign tumor with no recurrence after curettage B. Benign tumor with high recurrence rate after curettage C. Resection is not required for cure D. Benign tumor with high frequency of malignant transformation Answer: B Rationale:

The photomicrograph is a classic picture of odontogenic myxoma with its stellate and spindle myxoblasts with marked myxomatous stroma. The examinee is expected to know that odontogenic myxoma is a benign tumor, that is aggressive with high recurrence rates after curettage, and resection is required for a cure. The tumor is also reported to have a very low incidence of malignant transformation as myxosarcoma. Therefore, the best answer is option B. Options A, C & D are, therefore incorrect.

Reference:

Abaza, NA: Ultrastructural features and biologic aspects of odontogenic cysts and tumors. In: Oral Maxillofac Surg Clin North Amer, L. Gold (editor), WB Saunders, Philadelphia, 1994, p 523-577. Marx RE, and Stern D: Oral and Maxillofac Pathol: A RATIONALE for diagnosis and treatment. Quintessence Pub Comp, Chicago, 2003, p637-703.

Page 178: Anaesthesia TMJ Pain Orthognathic Pathology Reconstruction

2007 Oral and Maxillofacial Surgery Self Assessment Tool (OMSSAT)

The American Board of Oral and Maxillofacial Surgery 178

164. This photomicrograph is from a biopsy of a lesion in the angle of the mandible of a 17 year-old patient. Which of the following represents the correct diagnosis?

A. Follicular ameloblastoma B. Desmoplastic ameloblastoma C. Ameloblastic fibroma D. Metastatic squamous cell carcinoma Answer: C Rationale:

The correct answer is option C because the photomicrograph shows two actively proliferating tissue types: 1. cords and nests of hyperplastic odontogenic epithelium in 2: a hypercellular stroma. The ameloblastic fibroma is a mixed tumor. Option A is incorrect: there are no well developed follicles of odontogenic epithelium with peripheral palisading columnar cells and inner stellate-reticulum cells and the stroma is usually hypocellular in follicular ameloblastoma. Option B is incorrect because in this lesion, the stroma is characteristically acellular with the characteristic and marked wide bands of glassy collagenous tissue. Option D is incorrect since there is no cellular atypia of both the epithelium and connective tissue. In rare instances only the connective component of the tumor may have malignant transformation and the lesion is called ameloblastic fibrosarcoma.

Reference:

Page 179: Anaesthesia TMJ Pain Orthognathic Pathology Reconstruction

2007 Oral and Maxillofacial Surgery Self Assessment Tool (OMSSAT)

The American Board of Oral and Maxillofacial Surgery 179

Gardner DG: The mixed odontogenic tumors. Oral Surg Oral Med Pathol 58:166, 1984. Cohen DH: Ameloblastic fibroma ameloblastic fibro-odontoma and odontoma. In: Oral Maxillofac Surg North Amer: Odontogenic Tumors, DW White: Saunders, Philadelphia, 2004, 16: 375-384.

Page 180: Anaesthesia TMJ Pain Orthognathic Pathology Reconstruction

2007 Oral and Maxillofacial Surgery Self Assessment Tool (OMSSAT)

The American Board of Oral and Maxillofacial Surgery 180

165. Which of the following is true regarding the clinical presentation of a majority of central giant cell lesions?

A. Present in the maxilla in infants B. Present in the mandible in young adults C. Present in the mandible in elderly adults D. Present as multiple maxillary and mandibular lesions in adults Answer: B

Rationale:

Central giant cell lesions occur primarily in young adults. Occurrence in infants and the elderly is rare. The most common site of involvement is the mandible (70%). Central giant cell lesions are almost always solitary lesions.

Reference:

Oral and Maxillofacial Surgery Clinics of North America, November 1997, p 657.

Page 181: Anaesthesia TMJ Pain Orthognathic Pathology Reconstruction

2007 Oral and Maxillofacial Surgery Self Assessment Tool (OMSSAT)

The American Board of Oral and Maxillofacial Surgery 181

166. The incidence of which of the following neoplasms is increased in a patient with multiple cysts of the jaws and skin cancer?

A. Liposarcoma B. Renal cell carcinoma C. Osteosarcoma D. Medulloblastoma Answer: D Rationale:

The question deals with the basal cell nevus syndrome (Gorlin's) with its known systemic manifestations as an autosomal dominant with 100% penetrance and located on chromosome 9. Several structural anomalies and diseases have been associated with the syndrome and their frequencies have been tabulated. Options A, B & C are not recorded as part of the syndrome. Option D is correct, although the frequency has been low; it is clinically significant and, therefore, should be recognized in working-up patients with the syndrome.

Reference:

Rogerson KC: Gorlin's syndrome: an update on diagnosis and management, Oral Maxillofac Surg Clin North Amer 3:155-164, 1991. Marx RE, and Stern D: Oral and Maxillofac Pathol: A RATIONALE for diagnosis and treatment. Quintessence Pub Comp, Chicago, 2003, p602-606.

Page 182: Anaesthesia TMJ Pain Orthognathic Pathology Reconstruction

2007 Oral and Maxillofacial Surgery Self Assessment Tool (OMSSAT)

The American Board of Oral and Maxillofacial Surgery 182

167. This photomicrograph is from a mass of the anterior maxilla of a 41 year old patient. The CT scan revealed a mixed radiolucent/radiopaque lesion with ill-defined margins. Which of the following is the correct diagnosis?

A. Desmoplastic ameloblastoma. B. Ameloblastic fibroma C. Odontogenic myxoma D. Ameloblastic fibrosarcoma Answer: A Rationale:

The correct option is A. The most significant histologic feature of desmoplastic ameloblastoma is the hypocellular and markedly hyalinized collagenous stroma that compresses island of odontogenic epithelium. Option B is incorrect; the stroma should be hypercellular and actively proliferating. Option C is incorrect because the odontogenic myxoma is a connective tissue tumor where the cells are stellate and spindle shaped with mucoid or mucinous stroma, occasional fibroblast may be present. Occasional inert islands of odontogenic epithelium may be present. Option D is also incorrect. Although malignant transformation of the actively proliferating hypercellular stroma may occur, its incidence is very low. The photomicrograph does not show any atypical changes of these cells suggesting any malignant transformation.

Page 183: Anaesthesia TMJ Pain Orthognathic Pathology Reconstruction

2007 Oral and Maxillofacial Surgery Self Assessment Tool (OMSSAT)

The American Board of Oral and Maxillofacial Surgery 183

Reference: Yoshimura Y, Saito H: Desmoplastic variant of ameloblastoma: report of a case and review of the literature, J Oral Maxillofac Surg 84:1231, 1990.

Page 184: Anaesthesia TMJ Pain Orthognathic Pathology Reconstruction

2007 Oral and Maxillofacial Surgery Self Assessment Tool (OMSSAT)

The American Board of Oral and Maxillofacial Surgery 184

168. Which of the following best describes the histopathologic features of the lining of the glandular odontogenic cyst?

A. Non-keratinized stratified squamous epithelium with palisading basal cell layer and

parakeratotic luminal surface. B. Non-keratinized stratified squamous epithelium with mucus cells and cilia. C. Keratinized stratified squamous epithelium with ghost cells and stellate-like epithelial

cells. D. Keratinized stratified squamous epithelium with keratohyaline granules and luminal

fibrillar keratin. Answer: B Rationale:

Option B is the correct answer. The stem of the question diagnoses the cyst as glandular and the only option containing cells that can produce “glands” is option B where epithelium contains mucus cells and cilia. Option A describes parakeratotic odontogenic cyst; Option C describes calcifying odontogenic cyst; and option D is that of an orthokeratinized odontogenic cyst.

Reference:

Hussain K, Edmondson HB, Browne RM: Glandular odontogenic cysts: Diagnosis and treatment. Oral Surg Oral Med Oral Pathol, 1995: 79:593-602.

Page 185: Anaesthesia TMJ Pain Orthognathic Pathology Reconstruction

2007 Oral and Maxillofacial Surgery Self Assessment Tool (OMSSAT)

The American Board of Oral and Maxillofacial Surgery 185

169. A 13 year-old female is referred for evaluation of an expanding mass involving the maxilla. The parents report that the mass has been slowly expanding over the last few years and has now recently become more noticeable. Upon reviewing her medical history it is discovered that the patient started menstruating at the age of 9. Physical examination reveals an area of pigmentation with irregular borders involving the patient’s lower back and buttock. A panoramic radiograph demonstrates a mixed radiolucent/radiopaque lesion of the maxilla, as well as a smaller lesion involving the mandible. You obtain a chest radiograph that demonstrates a lesion involving the left third and fourth ribs. Which of the following is the most likely diagnosis?

A. Jaffe-Lichtenstein syndrome B. Cleidocranial dysplasia C. McCune-Albright syndrome D. Fragile X syndrome Answer: C Rationale:

The patient's clinical and radiographic examinations demonstrate multiple skeletal lesions suggestive of polyostotic fibrous dysplasia. The presence of café-au-lait pigmentation is consistent with either Jaffe-Lichtenstein syndrome or McCune-Albright syndrome. Endocrine abnormalities, such as precocious puberty as observed in this patient, are only observed in McCune-Albright syndrome.

Reference:

Kaban LB, Troulis MJ: Pediatric Oral and Maxillofacial Surgery, 2004, p 222; Marx RE, Stern D: Oral and Maxillofacial Pathology, 2003, p 749-750; Oral and Maxillofacial Surgery Clinics of North America, November 1997, p 731.

Page 186: Anaesthesia TMJ Pain Orthognathic Pathology Reconstruction

2007 Oral and Maxillofacial Surgery Self Assessment Tool (OMSSAT)

The American Board of Oral and Maxillofacial Surgery 186

170. A 45 year-old female is referred for evaluation of multiple loose teeth. A review of her history reveals recent onset depression. She also reports increased thirst and bone pain. The panoramic radiograph demonstrates multiple mandibular radiolucencies. The most likely diagnosis based on her history and radiographic findings is:

A. Langerhans cell disease. B. multiple odontogenic keratocysts associated with nevoid basal cell carcinoma syndrome. C. multiple myeloma. D. hyperparathyroidism. Answer: D

Rationale:

The differential diagnosis for multiple mandibular radiolucencies should include all of the above. However, the history of depression, thirst and bone pain is clearly consistent with hyperparathyroidism.

Reference:

Marx RE, Stern D: Oral and Maxillofacial Pathology, 2003, p 739.

Page 187: Anaesthesia TMJ Pain Orthognathic Pathology Reconstruction

2007 Oral and Maxillofacial Surgery Self Assessment Tool (OMSSAT)

The American Board of Oral and Maxillofacial Surgery 187

171. Which of the following represents the most appropriate treatment of an ameloblastic fibroma?

A. Resection with preservation of inferior border of mandible B. En-bloc resection including the inferior border of mandible C. Enucleation and curettage D. Marsupialization Answer: C Rationale:

Option C is correct since the lesion is benign with very low or no recurrence after adequate enucleation. Options A, B & D are incorrect. Option A may be selected for follicular ameloblastoma; Option B may be selected for desmoplastic ameloblastoma; and Option D is not appropriate for a diagnosis of ameloblastic fibroma which represents a true odontogenic tumor that will not respond to such therapy.

Reference:

Gardner, DB: the mixed odontogenic tumors. Oral Surg Oral Med, Oral Pathol, 58:166-1984. Marx RE, and Stern D: Oral and Maxillofac Patho: A Rationale for diagnosis and treatment. Quintessence Pub Comp, Chicago, 2003, p 637-703.

Page 188: Anaesthesia TMJ Pain Orthognathic Pathology Reconstruction

2007 Oral and Maxillofacial Surgery Self Assessment Tool (OMSSAT)

The American Board of Oral and Maxillofacial Surgery 188

172. A 40 year-old African American female presents for evaluation of mixed radiolucent/radiopaque, round, non-corticated lesions involving the apices of the anterior mandibular teeth. The patient is asymptomatic. All of the mandibular anterior teeth test vital. Which of the following would be the recommended management?

A. Obtain a CT scan to further evaluate the extent of the lesion. B. Obtain a tissue biopsy to confirm the diagnosis. C. No treatment is necessary. D. Perform curettage of the lesions and send tissue for pathologic diagnosis. Answer: C Rationale:

The clinical presentation is entirely consistent with periapical cementodysplasia. The patient is asymptomatic. No treatment is necessary.

Reference::

Oral and Maxillofacial Surgery Clinics of North America, November 1997, p 644-646.

Page 189: Anaesthesia TMJ Pain Orthognathic Pathology Reconstruction

2007 Oral and Maxillofacial Surgery Self Assessment Tool (OMSSAT)

The American Board of Oral and Maxillofacial Surgery 189

173. This panoramic radiograph is that of an 18-year-old patient with an incisional biopsy diagnosis of intraluminal unicystic ameloblastoma. Which of the following is the best surgical treatment?

A. Hemimandibulectomy with disarticulation and reconstruction with titanium plate and

metal condyle B. Hemimandibulectomy with disarticulation and reconstruction with iliac crest and rib

grafting C. Enucleation of the lesion D. Enucleation and curettage of the lesion Answer: D Rationale:

The panogram and the stem of the question emphasize the specific subtype of the ameloblastoma in an 18 year old patient which clearly should render itself to option D of the treatment which has the marked advantage of decreased morbidity with predictable healing of the bony defect and the low recurrence rates. Options A and B are incorrect and are reserved for solid or multicystic ameloblastomas. The intraluminal subtype of the unicystic ameloblastoma is best treated with enucleation and curettage (option D) rather than just an enucleation surgery (Option C).

Page 190: Anaesthesia TMJ Pain Orthognathic Pathology Reconstruction

2007 Oral and Maxillofacial Surgery Self Assessment Tool (OMSSAT)

The American Board of Oral and Maxillofacial Surgery 190

Reference: Gold L.: Biologic Behavior of Ameloblastoma. Oral Maxillofac Surg Clin North Amer. 1991; 3:21-71 Nakamura N, Higuchi Y, Mitsuyasu T, et al, Oral Surg Oral Med Oral Pathol Oral Rad Endod 2002; 93:13-20. Kessler HP: Intraosseous ameloblastoma. In: Oral Maxillofac Surg Clinic North Amer, Odontogenic Tumors DK, White L (editor), Saunders, Philadelphia, 2004, 16:309-322.

Page 191: Anaesthesia TMJ Pain Orthognathic Pathology Reconstruction

2007 Oral and Maxillofacial Surgery Self Assessment Tool (OMSSAT)

The American Board of Oral and Maxillofacial Surgery 191

174. Which of the following represents the most appropriate surgical management of an adenomatoid odontogenic tumor?

A. En-bloc resection with delayed reconstruction B. A two-stage surgery: decompression followed by enucleation C. Enucleation and curettage D. En-bloc resection and excision of surrounding involved soft tissue Answer: C Rationale:

Option A is incorrect since it is the recommended treatment for solid or multicystic ameloblastoma that has not perforated the cortical bone. Option B is also incorrect as this surgical modality is recommended for non-neoplastic entities. Option C is the correct answer as the adenomatoid odontogenic tumor is benign and enucleation and curettage is curative. Option D is incorrect and is the treatment of choice for odontogenic myxoma and other aggressive odontogenic tumors.

Reference:

Philipsen JP, Reichart PA, Zhang KH, et al.: Adenomatoid odontogenic tumor: biologic profile based on 499 cases. J. Oral Pathol Med 1991; 20:149-158. Carlson ER: Odontogenic cysts and tumors. In: Peterson's Principles of Oral & Maxillofacial Surgery, M Miloro, GE Ghali, PE Larson, and PD White (editors), Vol. 1, BC Decker, Inc., 2004, Hamilton, p575-596.

Page 192: Anaesthesia TMJ Pain Orthognathic Pathology Reconstruction

2007 Oral and Maxillofacial Surgery Self Assessment Tool (OMSSAT)

The American Board of Oral and Maxillofacial Surgery 192

175. An ameloblastoma exhibiting malignant cytologic features that is found in the lung is best classified as which of the following diagnoses?

A. Malignant ameloblastoma B. Ameloblastic carcinoma C. Metastatic ameloblastoma D. Clear cell odontogenic carcinoma Answer: B Rationale:

Ameloblastic carcinoma. The most important discriminating issue in the correct answer is the fact that the tumor exhibits malignant cytologic features. As such, the most accurate description of this process is ameloblastic carcinoma, and not malignant ameloblastoma, which is a benign tumor by definition. While this tumor is a metastatic ameloblastoma, this mere designation is incomplete. Also, a clear cell odontogenic carcinoma is possible, however, there is not enough information to support this answer.

Reference:

Carlson ER: Odontogenic cysts and tumors. In: Miloro M (ed): Peterson's Principles of Oral and Maxillofacial Surgery. Chapter 30, Hamilton, BC Decker, 575-596, 2004.

Page 193: Anaesthesia TMJ Pain Orthognathic Pathology Reconstruction

2007 Oral and Maxillofacial Surgery Self Assessment Tool (OMSSAT)

The American Board of Oral and Maxillofacial Surgery 193

176. A metastatic tumor of the mandible in a 65 year-old woman is most likely to originate in which of the following anatomic sites?

A. Colon B. Kidney C. Lung D. Breast Answer: D Rationale:

In adults, metastases to the jaws most commonly originate from primary carcinomas of the breast in women and of the lung in men. Other common primary sites in decreasing order of frequency are the prostate, gastrointestinal tract, kidney, colon, and rectum.

Reference:

Zarbo RJ, Carlson ER: Malignancies of the Jaws. In: Regezi J, Sciubba J, Jordan R (eds), Oral Pathology, Clinical Pathologic Correlations, 4th edition, chapter 14, St. Louis, Elsevier 336, 2003.

Page 194: Anaesthesia TMJ Pain Orthognathic Pathology Reconstruction

2007 Oral and Maxillofacial Surgery Self Assessment Tool (OMSSAT)

The American Board of Oral and Maxillofacial Surgery 194

177. Which of the following features is most consistent with the ameloblastic fibrosarcoma of the jaws?

A. More commonly located in the maxilla than the mandible B. More common in seventh decade adults than third decade adults C. More common in females than males D. Associated with the development of a painful mass. Answer: D Rationale:

The ameloblastic fibrosarcoma is a very rare odontogenic tumor that is most commonly located in the mandible. It is found with peak incidence in third decade with a mean age of 25 years. Males are affected more commonly than females with a ratio of about 2:1. The most common presenting symptoms are pain and a swelling.

Reference:

Reichart PA, Philipsen HP (eds): Odontogenic Tumors and Allied Lesions. Chapter 29, Ameloblastic Fibrosarcoma, London, Quintessence Publishing Co. Ltd. 2004, 255-262.

Page 195: Anaesthesia TMJ Pain Orthognathic Pathology Reconstruction

2007 Oral and Maxillofacial Surgery Self Assessment Tool (OMSSAT)

The American Board of Oral and Maxillofacial Surgery 195

178. A metastatic tumor of the mandible in a 70-year-old man is most likely to originate in which of the following anatomic sites?

A. Colon B. Kidney C. Lung D. Breast Answer: C Rationale:

In adults, metastases to the jaws most commonly originate from primary carcinomas of the breast in women and of the lung in men. Other common primary sites in decreasing order of frequency are the prostate, gastrointestinal tract, kidney, colon, and rectum.

Reference:

Zarbo RJ, Carlson ER: Malignancies of the Jaws. In: Regezi J, Sciubba J, Jordan R (eds), Oral Pathology, Clinical Pathologic Correlations, 4th edition, chapter 14, St. Louis, Elsevier 336, 2003.

Page 196: Anaesthesia TMJ Pain Orthognathic Pathology Reconstruction

2007 Oral and Maxillofacial Surgery Self Assessment Tool (OMSSAT)

The American Board of Oral and Maxillofacial Surgery 196

179. Which of the following represents the most common site for oral squamous cell carcinoma in the United States? A. Tongue B. Palate C. Alveolar Gingiva D. Buccal Mucosa Answer: A Rationale:

While exact site-specific incidence is variably reported, general consensus is that the oral tongue is the most commonly affected site in the oral cavity. This seems to be especially true of younger patients without significant risk factors where an increased index of suspicion is warranted.

Reference:

Neville, Damm, Allen, Bouquot. Oral and Maxillofacial Pathology. W.B. Saunders. 1995

Page 197: Anaesthesia TMJ Pain Orthognathic Pathology Reconstruction

2007 Oral and Maxillofacial Surgery Self Assessment Tool (OMSSAT)

The American Board of Oral and Maxillofacial Surgery 197

180. Information that may guide the decision for elective neck dissection in patients with T1 and T2 oral cavity squamous cell carcinoma includes which of the following?

A. Degree of differentiation B. Margin STATUS at the time of diagnostic biopsy C. Length of time that the cancer has been present D. Tumor depth of invasion Answer: D Rationale:

Treatment of the N0 neck with elective neck dissection is indicated when an approximate 20-30% chance of spread to lymph nodes exists. Length of time, degree of differentiation, and margin STATUS at diagnostic biopsy are not reliable indicators of spread to neck lymph nodes. Tumor depth of invasion has been correlated with risk of positive nodes in a large number of reported studies and can be used as a guide in making clinical decisions for elective neck dissection in T1 and T2 lesions.

Reference:

Holmes J, Dierks, E. Oral Cancer Treatment. Peterson's Principles of Oral and Maxillofacial Surgery 2nd Ed. BC Decker, 631-657, 2004

Page 198: Anaesthesia TMJ Pain Orthognathic Pathology Reconstruction

2007 Oral and Maxillofacial Surgery Self Assessment Tool (OMSSAT)

The American Board of Oral and Maxillofacial Surgery 198

181. The presence of a single positive neck lymph node without extracapsular spread after dissection for head and neck squamous cell carcinoma is equated to a decrease in 5-year survival by what percent?

A. 10% B. 25% C. 50% D. 75% Answer: C Rationale:

A single positive neck lymph node well contained within the capsule decreases 5-year survival to 50%. If the patient had extracapsular spread associated with the node then this would have decreased survival by an additional 50% to about 25% 5-year survival.

Reference:

Ghali G, Connor M. Oral Cancer: Classification, Staging, and Diagnosis. Peterson's Principles of Oral and Maxillofacial Surgery 2nd Ed. BC Decker, 618-630, 2004

Page 199: Anaesthesia TMJ Pain Orthognathic Pathology Reconstruction

2007 Oral and Maxillofacial Surgery Self Assessment Tool (OMSSAT)

The American Board of Oral and Maxillofacial Surgery 199

182. Which of the following features distinguishes lentigo maligna from lentigo maligna melanoma? A. Lentigo maligna is a nodular melanoma while lentigo maligna melanoma is a superficial

spreading melanoma. B. Lentigo maligna is a melanoma in situ while lentigo maligna melanoma develops from a

lentigo maligna. C. Lentigo maligna is a superficial spreading melanoma while lentigo maligna melanoma

is a nodular melanoma. D. Lentigo maligna is a primary melanoma while lentigo maligna melanoma is a

previously treated recurrent melanoma. Answer: B Rationale:

Lentigo maligna is a melanoma in situ. It is, by definition, a Clark's level I lesion that is confined to the epithelium. There is no breach of the basement membrane, similar to a carcinoma in situ. The former name of this entity was a melanotic freckle of Hutchison. A lentigo maligna melanoma is an invasive melanoma that arose from a lentigo maligna. As such, the lentigo maligna exhibits radial growth confined to the epithelium while the lentigo maligna melanoma exhibits vertical growth within the lamina propria.

Reference:

Carlson ER: Melanoma of the Oral and Maxillofacial Region. Oral and Maxillofacial Surgery Knowledge Update. Rosemont, American Association of Oral and Maxillofacial Surgeons, Volume 4, PTH 57-69, 2006.

Page 200: Anaesthesia TMJ Pain Orthognathic Pathology Reconstruction

2007 Oral and Maxillofacial Surgery Self Assessment Tool (OMSSAT)

The American Board of Oral and Maxillofacial Surgery 200

183. Which of the following tumors is least likely to be found in the parotid gland?

A. Acinic cell carcinoma

B. Adenoid cystic carcinoma

C. Carcinoma ex-pleomorphic adenoma

D. Polymorphous low-grade adenocarcinoma

Answer: D Rationale:

PLGA is almost exclusively found in the minor salivary glands. This is a characteristic feature of this salivary gland tumor. All of the other options occur with reasonable frequency in the parotid gland.

Reference:

Castle JT, Thompson LDR, from melt RA, Ewing BM, Kessler HP. Polymorphous low-grade adenocarcinoma: a clinicopathologic study of 164 cases. Cancer 1999;86:207-19

Page 201: Anaesthesia TMJ Pain Orthognathic Pathology Reconstruction

2007 Oral and Maxillofacial Surgery Self Assessment Tool (OMSSAT)

The American Board of Oral and Maxillofacial Surgery 201

184. A 30 year-old female presents to your office with a large mass of her right parotid gland. On examination you note that the buccal and frontal branches of her facial nerve are not functioning. Her MRI reveals a large mass with ill-defined borders occupying the parotid gland. You wish to procure tissue for histologic diagnosis. What is the most appropriate next step?

A. Incisional biopsy

B. Fine Needle aspiration (FNA)

C. Nerve sparing superficial parotidectomy

D. Total parotidectomy with nerve sacrifice

Answer: B Rationale:

A parotid incisional biopsy is the method of choice for the initial microscopic diagnosis of a diffuse, rather than a discrete mass of the parotid gland. It is contraindicated to perform an incisional biopsy when a neoplasm is suspected due to the seeding of the overlying skin. The only exception to this general rule is when the parotid neoplasm has already perforated through skin. The FNAB permits cytologic analysis of a parotid neoplasm without seeding the skin, which otherwise would require sacrifice at the time of parotidectomy.

Reference:

Marx RE: Incisional parotid biopsy for diagnosis of systemic disease. Oral and Maxillofacial Surgery Clinics of North America 7: 505-517, 1995.

Page 202: Anaesthesia TMJ Pain Orthognathic Pathology Reconstruction

2007 Oral and Maxillofacial Surgery Self Assessment Tool (OMSSAT)

The American Board of Oral and Maxillofacial Surgery 202

185. Which of the salivary glands is more prone to development of sialolithiasis?

A. Parotid gland B. Submandibular gland C. Sublingual gland D. Minor salivary glands Answer: B Rationale:

Due to the tortuosity of the Wharton's duct, as well as the character of the submandibular saliva, the submandibular gland is the most common salivary gland to develop stones.

Reference:

Levy BM, ReMine WH, Devine KD: Salivary gland calculi: Pain, swelling associated with eating. JAMA 1962; 181:1115. Berry RL: Sialadenitis and sialolithiasis. Diagnosis and management. Oral and Maxillofacial Surgery Clinics of North America 7: 479-503, 1995.

Page 203: Anaesthesia TMJ Pain Orthognathic Pathology Reconstruction

2007 Oral and Maxillofacial Surgery Self Assessment Tool (OMSSAT)

The American Board of Oral and Maxillofacial Surgery 203

186. A 69 year-old patient presents to your office with a complaint of an enlarging mass of his palate. He states that the mass has been present for the past year and has recently ulcerated and became painful. On clinical examination you note a 2 cm raised ulcerated mass at the junction of the right hard and soft palates. Which of the following diagnoses is most likely in this case?

A. Acinic cell carcinoma

B. Mucoepidermoid Carcinoma

C. Polymorphous Low-grade Adenocarcinoma

D. Carcinoma ex-pleomorphic adenoma

Answer: B Rationale:

The scenario points to a malignant diagnosis. Statistically, the most common minor salivary gland malignancy of the palate is the mucoepidermoid carcinoma. Of the 692 malignant palatal tumors in the AFIP data, 305 were mucoepidermoid carcinoma. The second most common malignancy in this location is the adenoid cystic carcinoma.

Reference:

Ellis GL, Auclair PL, Gnepp DR (eds): Surgical pathology of the salivary glands. Philadelphia, WB Saunders, 144-145, 1991.

Page 204: Anaesthesia TMJ Pain Orthognathic Pathology Reconstruction

2007 Oral and Maxillofacial Surgery Self Assessment Tool (OMSSAT)

The American Board of Oral and Maxillofacial Surgery 204

187. What is the most common salivary gland tumor of the upper lip?

A. Canalicular Adenoma

B. Pleomorphic Adenoma

C. Acinic Cell Carcinoma

D. Mucoepidermoid Carcinoma Answer: B Rationale:

The largest database of salivary gland tumors (AFIP) showed the pleomorphic adenoma to comprise 147 of 413 upper lip tumors, canalicular adenoma 89/413, acinic cell carcinoma 24/413 and mucoepidermoid carcinoma 12 of 413 tumors.

Reference:

Ellis GL, Auclair PL, Gnepp DR (eds): Surgical pathology of the salivary glands. Philadelphia, WB Saunders, 1991, pp 144-145.

Page 205: Anaesthesia TMJ Pain Orthognathic Pathology Reconstruction

2007 Oral and Maxillofacial Surgery Self Assessment Tool (OMSSAT)

The American Board of Oral and Maxillofacial Surgery 205

188. A 9 year-old patient presents with a mass on her left parotid gland. Your clinical examination reveals a 2 cm lesion on the left parotid gland. The lesion is attached to the overlying skin and the skin is erythematous. Her facial nerve is intact with exception of a slight buccal branch weakness. Which of the following tumors is the most likely diagnosis?

A. Adenoid cystic carcinoma

B. Acinic cell carcinoma

C. Lymphoma

D. Mucoepidermoid carcinoma

Answer: D Rationale:

The scenario depicts a malignant neoplasm in the parotid gland of this child. Statistically, the most likely diagnosis is mucoepidermoid carcinoma.

Reference:

Bentz BG, Hughes A, Ludrmann JP, Maddalozzo J Masses of the salivary gland region in children. Arch Otolaryngol Head Neck Surg 2000;126:1435-1439.

Page 206: Anaesthesia TMJ Pain Orthognathic Pathology Reconstruction

2007 Oral and Maxillofacial Surgery Self Assessment Tool (OMSSAT)

The American Board of Oral and Maxillofacial Surgery 206

189. What is the most common malignant submandibular gland tumor?

A. Mucoepidermoid carcinoma

B. Carcinoma ex-pleomorphic adenona

C. Adenoid Cystic Carcinoma

D. Acinic Cell Carcinoma Answer: C Rationale:

The AFIP data showed 144 adenoid cystic carcinomas of a total of 510 malignant tumors. The second most common malignancy of the submandibular gland was the mucoepidermoid carcinoma, accounting for 112 cases.

Reference:

Ellis GL, Auclair PL, Gnepp DR (eds): Surgical pathology of the salivary glands. Philadelphia, WB Saunders, 1991, pp 144-145.

Page 207: Anaesthesia TMJ Pain Orthognathic Pathology Reconstruction

2007 Oral and Maxillofacial Surgery Self Assessment Tool (OMSSAT)

The American Board of Oral and Maxillofacial Surgery 207

190. A 12 year-old patient is brought to your office for evaluation of a swelling on the left anterior floor of mouth. The examination reveals a dome-shaped, translucent, fluid filled elevation on the left anterior floor of mouth. On further examination of the head and neck you note that the swelling extends to the submental region on the same side. Based on your clinical diagnosis, what is the best treatment to insure that the lesion does not recur?

A. Incision and drainage

B. Marsupialization

C. Partial/total extirpation of pseudocyst

D. Sublingual gland excision Answer: D Rationale:

The scientific literature points to the sublingual gland excision, option D, to be the most reliably and effective form of treatment to ensure that a ranula does not recur. Incision and drainage will not resolve a ranula as saliva will continue to collect in the submucosal space. Marsupialization is a procedure reviewed in the literature; however, insufficient evidence exists to suggest that this procedure is more effective in managing the ranula than the sublingual gland excision. Since the gland is the culprit in this pathologic entity, excision of the pseudocyst will not cure the problem.

Reference:

Catone GA: Sublingual gland mucous escape. Pseudocysts of the oro-cervical region. Oral and Maxillofacial Surgery Clinics of North America 7: 431-477, 1995.

Page 208: Anaesthesia TMJ Pain Orthognathic Pathology Reconstruction

2007 Oral and Maxillofacial Surgery Self Assessment Tool (OMSSAT)

The American Board of Oral and Maxillofacial Surgery 208

191. A 40 year-old patient presents to the clinic one week after a left nerve sparing superficial parotidectomy for a pleomorphic adenoma. On exam you note an area of fluctuant swelling directly underneath the skin overlying the residual parotid bed. You explain to her that she has a salivary fistula. Which of the following represents the next step? A. Carryout a wound exploration.

B. Prescribe an anticholinergic agent.

C. Have her return to clinic in 2 weeks for reexamination.

D. Aspirate the area. Answer: C Rationale:

Most salivary fistulas resolve within 2 to 3 weeks without intervention. As such, a wound exploration is not necessary. Aspiration of the area will not resolve the fistula, as saliva will continue to be formed. Prescribing an anticholinergic agent such as Robinol is not initially indicated, but may play a role in the management of a chronic salivary fistula.

Reference:

Lee KJ (7th ed) Essential Otolaryngology, McGraw Hill 1998; pg 523.

Page 209: Anaesthesia TMJ Pain Orthognathic Pathology Reconstruction

2007 Oral and Maxillofacial Surgery Self Assessment Tool (OMSSAT)

The American Board of Oral and Maxillofacial Surgery 209

192. Which of the following represents an advantage of using platelet rich plasma?

A. Increased concentration of growth factors B. Increased clotting potential C. Decreased tissue viscosity D. Decreased immune responses Answer: A Rationale:

PRP is an increase in platelets at a wound or recipient bone graft site. Platelets are involved in hemostasis and contain multiple growth factors in their alpha granules. Yes, the clotting potential is increased with the use of PRP, but this is not the significant advantage since even the PPP can increase hemostasis. An increased concentration of growth factors is the significant advantage with the use of PRP. Tissue viscosity and immune responses are not changed by the use of PRP.

Reference:

Weibrich et al., Growth factor levels in PRP and correlations with donor age, sex, and platelet count. Int J Oral Maxillofac Impl 16(5):693-9, 2001. Weibrich et al. Growth factor levels in the platelet-rich plasma produced by 2 different methods: curasan-type PRP kit versus PCCS PRP system. Int J Oral Maxillofac Impl 17(2):184-90, 2002.

Page 210: Anaesthesia TMJ Pain Orthognathic Pathology Reconstruction

2007 Oral and Maxillofacial Surgery Self Assessment Tool (OMSSAT)

The American Board of Oral and Maxillofacial Surgery 210

193. Which of the following statements is true regarding full thickness and split thickness skin grafts?

A. A full thickness skin graft can be expected to contract approximately 80–90 % of its

initial size. B. Split thickness skin graft donor sites heal primarily by epithelial migration from adnexal

structures. C. Survival of split thickness skin grafts beyond 72 hours is dependent upon plasmatic

imbibition. D. Survival of a split thickness or full thickness graft in the initial 24-48 hours is dependent

upon inosculation. Answer: B Rationale:

Full thickness skin grafts can be expected to contract 15 -20% and should be harvested with that in mind. Healing of the donor site in split thickness skin grafting is similar to burns, and occurs primarily from the contribution of adnexal skin structures. Therefore, “b” is the best choice. Survival of skin grafts in the initial 48-72 hours is dependent upon plasmatic circulation or imbibition. Beyond that, neovascularization and inosculation take over.

Reference:

Ghali et al. Peterson's Principles of Oral and Maxillofacial Surgery. BC Decker 2004. Baker et al. Local Flaps in Facial Reconstruction. Mosby 1994.

Page 211: Anaesthesia TMJ Pain Orthognathic Pathology Reconstruction

2007 Oral and Maxillofacial Surgery Self Assessment Tool (OMSSAT)

The American Board of Oral and Maxillofacial Surgery 211

194. Which of the following is true regarding the lateral arm vascularized flap?

A. One advantage over the radial forearm flap is the likelihood of direct closure of the donor site of the lateral arm flap.

B. It is possible that harvest of the lateral arm flap can result in vascular compromise to the

atershed area of the lateral forearm and result in skin loss. C. The arterial blood supply to the flap is provided by the anterior radial collateral artery. D. Harvest of the flap should include preservation of the posterior cutaneous nerve of the

arm. Answer: A Rationale:

The lateral arm flap serves as a useful alternative to the radial forearm flap for reconstruction of oral cavity defects. Direct linear closure of the donor site is almost always possible, and results in a more esthetic site than the radial forearm. Sacrifice of the posterior radial collateral artery dose not pose any risk to the vascular supply to the distal arm. Harvest of the flap invariably results in sacrifice of the posterior cutaneous nerve of the arm, and it can be used for re-innervation.

Reference:

Urken ML, et al. Atlas of regional and free flaps for head and neck reconstruction. Raven Press 1995. Wolff KD, Holzle F. Raising of Microvascular flaps: A Systemic Approach. Springer-Verlag. 2005.

Page 212: Anaesthesia TMJ Pain Orthognathic Pathology Reconstruction

2007 Oral and Maxillofacial Surgery Self Assessment Tool (OMSSAT)

The American Board of Oral and Maxillofacial Surgery 212

195. Which of the following statements is true regarding the radial forearm osseocutaneous free flap?

A. The amount of bone thickness available is limited to 60 % of the radius’ circumference. B. The amount of bone length available is limited to 5 cm. C. Prophylactic plating of the donor site allows harvest of full thickness segments of the

radius. D. It is primarily indicated for defects of the mandibular angle and ramus.

Answer: D Rationale:

The radial forearm osseocutaneous flap has recently enjoyed a resurgence in popularity based on Neal Futran’s work. He has found that prophylactic plating of the donor site has basically eliminated pathologic fractures of the residual radius. Harvest is still limited to 40 % of the radius’ circumference and 10-12 cm of length. It is primarily indicated for non-tooth bearing areas of the mandible, such as the angle and ramus.

Reference:

Villaret DB, Futran NA. The indications and outcomes in the use of osteocutaneous radial forearm free flap. Head Neck 2003 Jun 25(6):475-481. Farwell DG, Futran ND. Oromandibular Reconstruction. Facial Plastic Surgery Vol 16 2000.

Page 213: Anaesthesia TMJ Pain Orthognathic Pathology Reconstruction

2007 Oral and Maxillofacial Surgery Self Assessment Tool (OMSSAT)

The American Board of Oral and Maxillofacial Surgery 213

196. A superiorly based platysma flap receives its dominant blood supply from which of the following vessels?

A. The occipital artery B. Submental branch of the facial artery C. Transverse cervical artery D. Superior thyroid artery Answer: B Rationale:

Three different variations of the platysma flap are available based on the dominant blood supply. The inferiorly based flap, with no real application in oral/facial reconstruction, receives its arterial supply from the transverse cervical artery. The posteriorly based platysma flap receives its blood supply primarily from branches of the occipital artery. The superiorly based platysma flap receives its blood supply from the submental branch of the facial artery at or near the inferior border of the mandible. The superior thyroid artery does not provide blood supply to the flap.

Reference:

Baur DA: The Platysma Myocutaneous Flap. Oral and Maxillofacial Surgery Clinics of North America. Nov, 2003, 559-564.

Page 214: Anaesthesia TMJ Pain Orthognathic Pathology Reconstruction

2007 Oral and Maxillofacial Surgery Self Assessment Tool (OMSSAT)

The American Board of Oral and Maxillofacial Surgery 214

197. Which of the following is the most important determinant in survival of a local random pattern skin flap of the face?

A. Width of the flap base B. Length of the flap C. Perfusion pressure of nutrient vessels to the flap D. Amount of reactive oxygen intermediates Answer: C Rationale:

It now recognized that the length to width ratio of facial skin flaps is only a rough guideline and can vary with different flaps. Generally a 3-1 ratio is felt to be optimal. However, the survival of a skin flap is primarily dependent on the intravascular resistance and perfusion pressure of the nutrient vessels. If the perfusion pressure drops below a critical closing pressure of the arterioles in the subdermal plexus, nutritional blood flow will cease and necrosis will occur. A flap with a wider base will only include additional subdermal vessels that have the same perfusion pressure since they are all based upon the same feeding vessel.

Reference:

Baker SR, Swanson NA; Local Flaps in Facial Reconstruction. Mosby, 15-30, 1995

Page 215: Anaesthesia TMJ Pain Orthognathic Pathology Reconstruction

2007 Oral and Maxillofacial Surgery Self Assessment Tool (OMSSAT)

The American Board of Oral and Maxillofacial Surgery 215

198. Which of the following statements is true regarding the superiorly based sternocleidomastoid myocutaneous flap?

A. The spinal accessory nerve can be sacrificed during elevation of the flap without

morbidity. B. Survival of the skin paddle requires that the blood supply must traverse an intervening

muscular unit . C. It can successfully and reliably reconstruct anterior floor of mouth defects. D. The muscle is relatively resistant to fibrosis following radiation treatment to the neck. Answer: B Rationale:

The blood supply to the skin paddle of the sternocleidomastoid myocutaneous flap must traverse through an intermediate muscular layer, the platysma muscle. The spinal accessory nerve should not be sacrificed as it also innervates the trapezius muscle and to do so would cause shoulder dysfunction. The arc of rotation of the SCM flap is inadequate to reconstruct anterior floor of mouth defects. Radiation treatment to the neck can cause fibrosis and reduction of the muscular blood supply, making dissection more difficult and compromising flap survivability.

Reference:

Urkin ML, Cheney ML, Sullivan MJ, Biller HF; Atlas of Regional and Free Flaps for Head and Neck Reconstruction. Raven Press, New York, NY. 1995, 49-64 Carlson ER; Regional Flaps in Maxillofacial Reconstruction. Oral and Maxillofacial Surgery Clinics of North America, 5(4), 667-685, 1993

Page 216: Anaesthesia TMJ Pain Orthognathic Pathology Reconstruction

2007 Oral and Maxillofacial Surgery Self Assessment Tool (OMSSAT)

The American Board of Oral and Maxillofacial Surgery 216

199. The dominant blood supply of a trapezius myocutaneous flap is provided by which of the following vessels? A. Dorsal scapular artery B. Transverse cervical artery C. Paraspinous perforators D. Occipital artery Answer: B Rationale:

While in situ, the trapezius muscle receives minor contributions from the dorsal scapular artery, paraspinous perforators, and the occipital artery. However, once the flap is elevated, the transverse cervical artery is the primary nutrient vessel of this axial pattern flap. The transverse cervical artery is often sacrificed in patients who have undergone a neck dissection. If a trapezius myocutaneous flap is considered in one of these patients, the presence of the vessel must be confirmed by angiography.

Reference:

Urkin ML, Cheney ML, Sullivan MJ, Biller HF; Atlas of Regional and Free Flaps for Head and Neck Reconstruction. Raven Press, New York, NY. 1995, 29-48. Carlson ER; Regional Flaps in Maxillofacial Reconstruction. Oral and Maxillofacial Surgery Clinics of North America, 5(4), 667-685, 1993

Page 217: Anaesthesia TMJ Pain Orthognathic Pathology Reconstruction

2007 Oral and Maxillofacial Surgery Self Assessment Tool (OMSSAT)

The American Board of Oral and Maxillofacial Surgery 217

200. Which of the following enzymes is most responsible for mediating a reperfusion injury in an ischemic random pattern skin flap?

A. Acetylaldehyde dehydrogenase B. Xanthine oxidase C. Dopa decarboxylase D. Cystathione synthetase Answer: B Rationale:

In ischemic tissue, purines are oxidized by xanthine oxidase to xanthine and uric acid. The highly toxic superoxide free radical is formed as a byproduct. When oxygen is reintroduced suddenly during reperfusion, large amounts of superoxide are formed causing cellular damage. Acetylaldehyde dehydrogenase plays a role in the metabolism of ethanol. Dopa decarboxylase metabolizes L-dopa to dopamine. Cystathione synthetase is an enzyme involved in homocysteine metabolism.

Reference:

Baker SR, Swanson NA; Local Flaps in Facial Reconstruction. Mosby, 15-30, 1995

Page 218: Anaesthesia TMJ Pain Orthognathic Pathology Reconstruction

2007 Oral and Maxillofacial Surgery Self Assessment Tool (OMSSAT)

The American Board of Oral and Maxillofacial Surgery 218

201. Which of the following is true regarding the temporalis muscle flap in mid-face reconstruction?

A. Cannot be used as a carrier of calvarial bone B. Cannot transfer overlying temporal skin C. The muscle flap cannot be skin grafted D. Cannot be safely elevated without damaging the frontal branch of the facial nerve Answer: B Rationale:

The temporal muscle flap cannot be used to transfer overlying temporal skin due to a lack of myocutaneous perforators over the muscle. The muscle flap can successfully be grafted with a split thickness skin graft. The muscle flap can be used as a carrier of vascularized outer table calvarial bone for orbital or palatal reconstruction. The frontal branch of the facial nerve is located in the temporal parietal fascia, superficial to the temporal muscle fascia. This location is predictable and the flap can be elevated without damaging the frontal branch.

Reference:

Urkin ML, Cheney ML, Sullivan MJ, Biller HF; Atlas of Regional and Free Flaps for Head and Neck Reconstruction. Raven Press, New York, NY. 1995, 65-76.

Page 219: Anaesthesia TMJ Pain Orthognathic Pathology Reconstruction

2007 Oral and Maxillofacial Surgery Self Assessment Tool (OMSSAT)

The American Board of Oral and Maxillofacial Surgery 219

202. Sixty percent of the lower lip is missing following tumor ablation. Frozen sections have verified tumor free margins. Of the following choices, which is the best option for obtaining an adequate functional and esthetic result in reconstructing the lower lip?

A. The pectoralis major flap B. The Karapandzic flap C. The Abbe-Estlander flap D. The deltopectoral flap Answer: B Rationale:

In most instances of reconstructing one-half to two-thirds of the lower lip, the Karapandzic flap provides a good functional and esthetic result. The Karapandzic flap re-approximates the orbicularis oris muscle providing for adequate sphincter function of the lips. In addition, since the sensory and motor nerves are preserved, the flap allows for a sensate and functional flap. The pectoralis major flap or the deltopectoral flap could be used to reconstruct the lower lip. However, these flaps would not provide adequate sensation or function, and the patient would have poor oral competence. In addition, the esthetic result would be less than optimal. The Abbe-Estlander flap does not provide sufficient soft tissue for reconstruction of 60% of the lower lip, and its indication is for reconstruction of the oral commissure.

Reference:

Baker SR, Swanson NA; Local Flaps in Facial Reconstruction. Mosby, 345-396, 1995 Carlson ER; Regional Flaps in Maxillofacial Reconstruction. Oral and Maxillofacial Surgery Clinics of North America, 5(4), 667-685, 1993

Page 220: Anaesthesia TMJ Pain Orthognathic Pathology Reconstruction

2007 Oral and Maxillofacial Surgery Self Assessment Tool (OMSSAT)

The American Board of Oral and Maxillofacial Surgery 220

203. When medicinal leeches are used in salvaging flaps suffering from venous congestion, antibiotic prophylaxis against which of the following organisms is indicated?

A. Pseudomonas aeruginosa B. Pasteurella multocida C. Aeromonas hydrophilia D. Bacteroides species Answer: C Rationale:

The leech gut contains an endosymbiotic bacterium Aeromonas hydrophilia. The patient is at risk of developing an infection from this organism. Therefore, prophylaxis against A. hydrophilia is recommended in patients receiving leech therapy. Trimethoprim/sulfa, fluoroquinolones, or third generation cephalosporins are the antibiotics of choice. P. multocida is associated with cat bites. Bacteroides species are associated with odontogenic infections. P. aeruginosa is responsible for a myriad of infectious processes.

Reference:

Utley, DS: The Failing Flap in Facial Plastic and Reconstructive Surgery. Laryngoscope 108:1129-35. 1998